You are on page 1of 406

CA Intermediate

STRATEGIC MANAGEMENT

MCQ Question Bank

By-
CA Rachendra Mundada
(CA,ISA,M.Com)

“Doing Business without strategy is slowest route to


success”
INDEX

Ch Particulars Page No. of


No No MCQ
1 Introduction to Strategic 1 30
Management
2 Dynamics of Competitive 7 80
Strategy
3 Strategic Management Process 21 70
4 Corporate Level Strategies 31 80
5 Business Level Strategies 45 60
6 Functional Level Strategies 56 60
7 Organisation & Strategic 67 60
Leadership
8 Strategy Implementation & 78 70
Control

CA Rachendra Mundada Best Faculty for EIS-SM all over India 9422972000/7020400972
Buy Pendrive from learnkart.in

Chapter – 1 : Introduction to Strategic Management

1. Which of these seeks to relate the goals of organization to the means of achieving
them ?
a) Strategy
b) Execution
c) Monitoring
d) Management

2. When market & competitive conditions take an unexpected turn then required strategy is
.
Proactive
Reactive
Both
None

3. In which level there are no general managers ?


Corporate Level
Business Level
Functional Level
None of these
4. Organization can be classified as on the basis of interest they have.
Market Oriented & Prize Oriented
General & Specific
Broad & Narrow
Commercial & Non-Commercial

5. When did integrative course introduced in Harvard Business


School? a) 1865
b) 1911
c) 2001
d) 2015

6. What is a unified, comprehensive & integrated plan designed to assure that


basic objectives
of enterprise are achieved ?
a) Strategy

CA Rachendra Mundada Best Faculty for EIS-SM all over India Page 1
Buy Pendrive from learnkart.in

b) Execution
c) Monitoring
d) Management

7. What is required by companies to meet changing condition in their industries ?


a) Far-sighted
b) Visionary
c) Develop long term strategies
d) All the above

8. Corporate level of management does consists of _.


Board of directors
Chief Executive Officer
Corporate staff
All the above

9. Who said that ‘I believe we do a far better job of strategic management than any company I
know?
Richard Cyert
Igor H. Ansoff
William F. Glueck
Michal Porter

10. Strategy is not a for sound, alert & responsible management ?


Reason
Necessity
Substitute
Platform

11. Business policy tends to emphasize on which aspect of strategic management?


a) System Analytical
b) Portfolio Analytical
c) Credit Analytical
d) Rational Analytical

12. In large organization strategies are formulated at which level ?


a) Corporate levels
b) Divisional Levels
Buy Pendrive from learnkart.in

c) Functional levels
d) All of the above

13. Which of these serves as a corporate defense mechanism against mistakes & pitfalls?
a) Strategic Management
b) Marketing Techniques
c) Strategic Awareness
d) Competitive Analysis

14. Which is a self-contained division?


a) Strategic business unit
b) Divisional structure
c) Corporate division
d) Managerial division

15. Successful hospital strategy for future will require renewed &
deepened collaboration with
.
a) Physicians
b) Ambulance Providers
c) CCTV Providers
d) Nurses

16. Which is used with reference to a key group in an organization in-charge of


its affairs?
a) Strategy
b) Management
c) Execution
d) Monitoring

17. Which is a planned strategy ?


a) Proactive
b) Reactive
c) Adaptive
d) None of these

18. Which is not an advantage of strategic management?


a) Helps organizations to be proactive.
b) Control their own destiny in better manner.
Buy Pendrive from learnkart.in

c) Identify available opportunity


d) None of these

19. Who is the head of the division?


a) Corporate level manager
b) Business level managers
c) Functional level manager
d) None of these

20. Whole Strategic landscape of healthcare is changing because of .


a) Doctors
b) Nurses
c) Internet
d) Patients

21. Which is a set of interrelated functions & processes carried out by management
of an organization to attain its objective ?
a) Strategy
b) Execution
c) Monitoring
d) Management

22. Which is adaptive reaction to changing business environment ?


a) Proactive
b) Reactive
c) Adaptive
d) None of these

23. Which is not the limitation of strategic management ?


a) Time Consuming Process
b) Difficult Estimation of competitive responses
c) Costly Process
d) Understandable complex environment

24. Functional level managers are concerned with .


a) Strategies responsible for Specific business operations
b) Strategies that span individual business
c) Strategies that are specific to particular business
d) None of these
Buy Pendrive from learnkart.in

25. Public enterprise generally cannot diversify into .


a) Unrelated businesses
b) Profitable businesses
c) Private businesses
d) All of these

26. Which of these is something that has to do with war & ways to win over enemy ?
a) Strategy
b) Management
c) Execution
d) Monitoring

27. Which strategy is used by companies to cope up with uncertain


business environment ?
a) Proactive
b) Reactive
c) Both
d) None of these

28. In competitive environment it is difficult to clearly estimate _ about firm’s


strategies.
a) Competitive Responses
b) Operating Process
c) Control System
d) Reasonable Outcomes

29. Who is more closer to the customers?


a) Corporate level manager
b) Business level managers
c) Functional level manager
d) None of these

30. Who enjoys more freedom in strategic decision making?


a) Government Strategists
b) Non-Government Strategists
c) Both (a) & (b)
d) None of these
Buy Pendrive from learnkart.in

1 a 7 d 13 a 19 b 25 a
2 b 8 d 14 a 20 c 26 a
3 c 9 a 15 a 21 d 27 c
4 d 10 c 16 b 22 a 28 a
5 b 11 d 17 a 23 d 29 c
6 a 12 d 18 d 24 a 30 a
Buy Pendrive from learnkart.in

Chapter – 2: Dynamics of competitive Strategy

1. Which of these require a lot of cash to hold their share & need heavy investment
with low growth potential?
a) Star
b) Cash-Cow
c) Question-Mark
d) Dog

2. Which of these refers to a strategy where the business seeks to sell its
existing products into new market & this can be achieved by new product
packaging, distribution channel etc?
a) Market Penetration
b) Market Development
c) Product Development
d) Diversification

3. In which position of ADL matrix companies are generally vulnerable in the face
of increased competition from stronger & more proactive companies in market?
a) Dominant
b) Tenable
c) Favorable
d) Strong

4. Firm is successful in achieving only after other firm’s effort to duplicate or


imitate it are failed.
a) Strategic Leadership
b) Competitive Advantage
c) Strategic Intent
d) Competitive Landscape

5. A company which has gone global is called


a) MNC
b) TNC
c) Both (a) & (b)
d) None of these

6. Determine the flow of creating the competitive advantage.


Buy Pendrive from learnkart.in

i) Resources, ii) Competitive advantage, iii) capabilities


a) (i),(iii),(ii)
b) (ii),(i),(iii)
c) (iii),(ii),(i)
d) (i),(ii),(iii)

7. In industries where rate of is fast, are quite likely to became obsolete.


a) Product Innovation, Product Patents
b) Market Innovation, Firm’s Name
c) Product Innovation, Firm’s Name
d) Market Innovation, Product Patents

8. The TOWS matrix is a tool of generating strategic .


a) Leadership
b) Environment
c) Option
d) Control

9. Which of these is useful analytical tool for comparing market position of each firm
separately when an industry has so many competitors that it is not practical to
examine each of them?
a) Strategic Group Mapping
b) Scenario Analysis
c) Strategic Core Analysis
d) PESTEL Analysis

10. Which is not the area identified in major core competencies?


a) Competitor differentiation
b) Customer Value
c) Profitability of industry
d) Application to other market

11. Which of these refers to process of integration of world economy into one
huge market.
a) Globalization
b) Privatization
c) Stratification
d) None Of these
Buy Pendrive from learnkart.in

12. Resources are foundation of & unique bundle of generate


competitive advantage leading to wealth creation.
a) Resource, Plan
b) Strategy, Resource
c) Policies, Procedure
d) Resources, Capabilities

13. Which of these is a widely used means of describing activities within & around
an organization & relating them to an assessment of competitive strength of an
organization.
a) Accounting Analysis
b) Portfolio Analysis
c) Controls Analysis
d) System Analysis

14. Which concept is also known as ‚S-Shaped‛ Curve?


a) PLC
b) BCG
c) ADL
d) SWOT

15. Which is nit the example of driving forces?


a) Increasing Globalization
b) Product Innovation
c) Changes in cost/efficiency
d) None of these

16. is defined as a combination of skills & techniques rather than individual skill
or separate technique.
a) Competency
b) Driving Force
c) Core Identity Force
d) Concurrent Filter

17. Which area of value chain transform various inputs into the final product or service?
a) Marketing & Sales
b) Procurement
c) Infrastructure
d) Operation
Buy Pendrive from learnkart.in

18. Which is achieved advantage over rivals when a company’s profitability is


greater than average profitability of firms in its industry?
a) Competitive Advantage
b) Market Position
c) Strategic Choice
d) Value Creation

19. Which of these is a simplest way to portray a corporation’s portfolio of investment?


a) Ansoff’s Growth Matrix
b) Product Life Cycle
c) SWOT Analysis
d) BCG Growth Share Matrix

20. Which is meant about analyze competitors & at the same time, it permits the
comprehension of their vision, mission, core values, niche market, strength &
weakness?
a) Strategic Analysis
b) Core Competence
c) Competitive Landscape
d) Competitive Strategy

21. In which stage of product life cycle the sales & profit falls down sharply due to
some new product replaces the existing product?
a) Introduction
b) Growth
c) Maturity
d) Decline

22. are capabilities that serves as a source of competitive advantage for a firm
over its rivals.
a) Concurrent Filters
b) Core competencies
c) Driving Forces
d) Core Identity Forces

23. Capabilities that do not have strategic equivalents are known as _


Capabilities.
a) Non-Substitutable
b) Valuable
c) Rare
CA Rachendra Mundada Best Faculty for EIS-SM all over India Page 10
CA Rachendra Mundada Best Faculty for EIS-SM all over India Page 10
Buy Pendrive from learnkart.in

d) Non-Imitable

24. Which of these is a unique features of a company & its products that are
perceived by the target market as significant & superior to the competition?
a) Strategic Leadership
b) Competitive Advantage
c) Strategic Intent
d) Competitive Landscape

25. Which approach has an advantage that it can be used to diagnose a portfolio
of products in order to establish stage at which each of them exist?
a) Experience Curve
b) Product Life Cycle
c) SWOT Analysis
d) Growth Share Matrix

26. is a business analysis which identifies competitors, either direct or indirect.


a) Competitive Landscape
b) Strategic Analysis
c) Core Competence
d) Competitive Strategy

27. Which of the following is also known as ‚Problem Child‛ or ‚Wild Cats‛ ?
a) Star
b) Cash-Cow
c) Question-Mark
d) Dog

28. Which of these refers to a growth strategy where he business focuses on


selling existing products into existing markets?
a) Market Penetration
b) Market Development
c) Product Development
d) Diversification

29. Which position in ADL matrix generally comes about when industry is fragmented
& no one competitor stand out clearly, result in market leaders a reasonable degree of
freedom?
a) Dominant

CA Rachendra Mundada Best Faculty for EIS-SM all over India Page 11
Buy Pendrive from learnkart.in

b) Tenable
c) Favorable
d) Strong

30. Which of the following is unique feature of company & its products that
are perceived by target market?
a) Strategic Leadership
b) Competitive Advantage
c) Strategic Intent
d) Globalization

31. Which of the following is based on commonly observed phenomenon that unit
costs decline as a firm accumulates experience in terms of a cumulative volume of
production?
a) Experience Curve
b) Product Life Cycle
c) SWOT Analysis
d) Growth Share Matrix

32. comes from a firm’s ability to perform activities more effectively that its
rivals.
a) Competitive Landscape
b) Competitive Advantage
c) Core Competence
d) Strategic Change

33. is ‚a group of firms whose products have same & similar attributes such
that they compete for same buyers‛.
a) Industry
b) Value Chain
c) Competitive Landscape
d) Strategy Analysis

34. Which strategy has its objective to sell or liquidate the business because
resources can be better used elsewhere?
a) Build
b) Hold
c) Harvest
d) Divest
Buy Pendrive from learnkart.in

35. In which position of ADL, a firm has considerable degree of freedom over its
choice of strategies & often able to act without its market position?
a) Dominant
b) Tenable
c) Favorable
d) Strong

36. Most dominant forces are called because they have biggest influence.
a) Driving Forces
b) Dominant economic feature
c) Strategic Moves
d) Competitive Landscape

37. Which of these is a measurement through which value can be measured?


a) Price
b) Willing to pay
c) Cost
d) All the above

38. Due to centralized system which of the following will face barrier in decision making?
a) MNC
b) TNC
c) Both (a) & (b)
d) None of these

39. Which of the following is not the characteristic of competitive advantage?


a) appropriability
b) Transferability
c) Imitability
d) Tractability

40. The concept of the core competency was developed by .


a) H.N. Prashad & F.W. Taylor
b) R.K. Narayan & Arthur D. Little
c) C.K. Prahalad & Gary Hamel
d) Ansoff & Willium F. Gluek

41. Which is a particularly important area of supportive activities in value chain


that transcends all primary activities?
Buy Pendrive from learnkart.in

a) Procurement
b) Technology Development
c) Human Resources Manager
d) Infrastructure

42. Which is not the factor on which company’s profitability depends?


a) Value customers place for product/service
b) Prize that company charges for its product
c) Profit charged by the company
d) Costs of creating those product

43. Which of these are products or SBU that are growing rapidly & also need
heavy investment to maintain their position & finance their rapid growth
potential?
a) Star
b) Cash-Cow
c) Question-Mark
d) Dog

44. Determine the correct stage of competitive landscape.


i) Put all information together ii) Determine Strength & Weakness of competitors iii)
Understand the competitors, iv) Identify the competitors
a) (ii),(iv),(i),(iii)
b) (iii),(iv),(ii),(i)
c) (i),(iii),(ii),(iv)
d) (iv),(iii),(ii),(i)

45. is ‚a group of firms whose products have same & similar attributes such
that they compete for same buyers‛.
a) Industry
b) Value Chain
c) Competitive Landscape
d) Strategy Analysis

46. Which of these are low-growth, low-share businesses & products that may
generate enough cash to maintain themselves but do not have much future?
a) Star
b) Cash-Cow
c) Question-Mark
d) Dog
Buy Pendrive from learnkart.in

47. Which of these refers to a growth strategy where new products is market in
new markets?
a) Market Penetration
b) Market Development
c) Product Development
d) Diversification

48. General Electric Matrix does not known as .


a) Business Planning Matrix
b) Nine-Cell Matrix
c) Electric Model
d) S-Shaped Curve

49. Nine-Cell Matrix is developed on the basis of _.


a) Traffic Control Lights
b) Electricity Cable Tools
c) General Electronics Functions
d) Power Failure Concept

50. Which is the factor used by Business Planning Matrix ?


a) Only Market Growth Rate
b) Only Relative Market Share
c) Market Attractiveness & Business Strength
d) Only Market Growth Rate & Relative Market Share

51. Which position of ADL matrix is comparatively rare position & in many cases
is attributable to a monopoly?
a) Dominant
b) Tenable
c) Favourable
d) Strong

52. Industry’s economic feature & competitive structure revealed about its
fundamental character & about ways in which its environment may be changing.
a) A Lot, Little
b) Little, A lot
c) Nothing, A lot
d) A Lot, Nothing
Buy Pendrive from learnkart.in

53. Competitive strategy consist of moves to .


a) Attract Customer
b) Withstand Competitive pressure
c) Strengthen market position
d) All the above

54. Which of the following gain more interest in local market?


a) MNC
b) TNC
c) Both (a) & (b)
d) None of these
55. Companies often set to reduce high transformational cost in globalization.
Bearer Plant
Overseas plant
Domestic Plant
All the above

56. The concept ADL Matrix is developed by .


Igor H. Ansoff
Willium F. Gluek
Heinz Weihrich
Arthur D. Little

57. Which is not the factor influencing market position of an organization?


Organization’s Correct market position
Nature of opportunity and threat
Capability to capitalize opportunity
Capability to stratify situation

58. Which of these are the rules that shape whether a company will be financially
& competitively successful?
a) Core Identity Forces
b) Driving Forces
c) Key Success Factors
d) Concurrent Filters

59. Which criteria of core competency is related with the capabilities that allows firm
to exploit opportunity or avert threats in its external environment?
Buy Pendrive from learnkart.in

a) Valuable
b) Rare
c) Costly to Imitate
d) Non-substitutable

60. ‚If you don’t have a competitive advantage, don’t compete‛, it is said by .
a) Igor H. Ansoff
b) Willium F. Gluek
c) Jack Welch
d) Arthur D. Little

61. Which of the following does not have centralized management system?
MNC
TNC
Both (a) & (b)
None of these

62. Which of these will not be considered as primary activity?


Inbound Logistic
Marketing & Sales
Procurement
Service & Operation

63. The concept of TOWS matrix was developed by


Igor H. Ansoff
Willium F. Gluek
Heinz Weihrich
Arthur D. Little

64. Which is not the characteristic of SWOT analysis?


a) Provides Logical framework
b) Helps in crafting strategy
c) Present a cooperative account
d) Guides in strategy identification

65. Which of these are those things that most affect industry member’s ability to
prosper in marketplace?
a) Key Success Factors
b) Driving Forces
Buy Pendrive from learnkart.in

c) Core Identity Forces


d) Concurrent Filters

66. Capabilities that are valuable, rare, costly to imitate, & non-substitutable are
.
a) Core Competency
b) Driving Forces
c) Key Success Factors
d) Concurrent Filters

ese under linkages seeks to improve performance through closer working relationship between specialists within the
anagement
ement
ion Management
ment Module

nit of company that has a separate mission & objectives and which can be planned independently from other compa

e objective of competitive strategy?


etitive Advantage
t Share
on

70. Which strategy has its objective to increase short term cash flows regardless of
long term earning ?
a) Build
b) Hold
c) Harvest
d) Divest

71. Determine the flow of creating the competitive advantage.


Buy Pendrive from learnkart.in

i) Assign the firms, ii) Plot firms on a two-variable map, iii) Identify competitive
characteristic, (iv) Draw circle around each strategic group
a) (i),(iii),(ii),(iv)
b) (ii),(i),(iv),(iii)
c) (iii),(ii),(i),(iv)
d) (iv),(i),(ii),(iii)

72. A for a firm is whatever it does Best.


a) Core Competency
b) Driving Forces
c) Key Success Factors
d) Concurrent Filters

73. Which integration attempts to improve performance through ownership of


more parts of the value system, making linkages internal to organization?
a) Vertical
b) Horizontal
c) Co-Centric
d) Stratified

74. Which can be defined as a technique that helps strategists in taking


strategic decisions with regard to individual products or businesses in a firm’s
portfolio?
a) Strategic Analysis
b) Portfolio Analysis
c) Market Analysis
d) System Analysis

75. of a firm evolves out of consideration of several factors that are external to
it.
a) Competitive Landscape
b) Strategic Analysis
c) Core Competence
d) Competitive Strategy

76. Industry & competitive analysis begins with .


a) Trigger of change
b) Dominant Economic Feature
c) Nature & strength of competition
d) Prospect of Industry
Buy Pendrive from learnkart.in

77. Which strategy has its objective of preserve market share?


a) Build
b) Hold
c) Harvest
d) Divest

78. Which of these is a portfolio analysis technique that is based on the product
life cycle?
a) BCG Growth Matrix
b) Ansoff Growth Matrix
c) Arthur D. Little Matrix
d) General Electric Matrix

79. If a company’s strategies result in superior performance it is said to have .


a) Core Competency
b) Strategic Analysis
c) Competitive Advantage
d) Value Creation

80. The objective competitive strategy is to generate


a) Generate Competitive Advantage
b) Increase Market Share
c) Beat Competition
d) All the above

1 c 11 a 21 d 31 a 41 c 51 a 61 b 71 c
2 b 12 b 22 b 32 b 42 c 52 a 62 c 72 c
3 b 13 a 23 a 33 a 43 a 53 d 63 c 73 a
4 b 14 a 24 b 34 d 44 d 54 b 64 b 74 b
5 c 15 d 25 a 35 d 45 a 55 b 65 a 75 d
6 a 16 a 26 a 36 a 46 d 56 d 66 c 76 b
7 a 17 d 27 c 37 b 47 d 57 d 67 a 77 b
8 c 18 a 28 a 38 a 48 d 58 c 68 a 78 c
9 a 19 d 29 c 39 d 49 a 59 a 69 d 79 c
10 c 20 c 30 b 40 c 50 c 60 c 70 c 80 d

CA Rachendra Mundada Best Faculty for EIS-SM all over India Page 20
Buy Pendrive from learnkart.in

CHAPTER 3 : Strategic Management Process

1. Which is a managerial process of selecting best course of action out of several


alternative courses for purpose of accomplishment of organization goals?
a) Decision Making
b) Strategy Implementation
c) Programming
d) Monitoring strategy

2. Which is not the element of strategic intent ?


Business Definition
Vision
Goals & Objectives
None of these

3. Rivals firms, operating in the same industry rely on different due to their strategic choice.
Mission
Goals & Objective
Business Definition
Business Model

4. Forming a strategic vision is an exercise in intelligent .


Execution
Monitoring
Planning
Entrepreneurship

5. Who was the first to agitate the issue of mission statement through writings?
a) Peter Drucker
b) Theodore Levitt
c) ‚Both (a) & (b)
d) None of these

6. Which is more specific & translate goals to both short-term & long-term goals?
a) Vision
b) Mission
c) Objectives

CA Rachendra Mundada Best Faculty for EIS-SM all over India Page 21
Buy Pendrive from learnkart.in

d) Business Model

7. Strategy formulating is at which stage of strategic management.


a) First
b) Second
c) Third
d) Fourth

8. Which of these is foundation & from this the network of goals is built ?
a) Objectives
b) Business Model
c) Mission
d) Business Definition

9. Which stage is diagnostic phase of strategic analysis?


a) Strategy Implementation
b) Environmental & Organizational Analysis
c) Strategic Evolution & Control
d) Formulating Strategy

10. Which is not the major dimension of strategic decision making?


a) Top management involvement
b) Commitment of organizational resources
c) Future Oriented
d) Impact on short term prosperity of firm

11. Which can be employed when information gathering & additional are not able
to reduce uncertainty?
a) Report Analysis
b) Market Analysis
c) Risk Analysis
d) Scenario Analysis

12. Depicts the organization’s aspirations and provides a glimpse of what the
organization would like to become in the future?
a) Vision
b) Mission
c) Strategic management
d) None of these
Buy Pendrive from learnkart.in

13. Which of these is not related with business definition ?


a) Customers needs
b) Target Markets
c) Alternative Technologies
d) Organization Aspiration

14. Which is not the element of strategic vision?


a) Coming up with a mission statement
b) Using mission statement
c) Communicating strategic vision
d) Modifying strategic vision

15. Why an organization need mission statement ?


a) To have its own special identity
b) To Reduce Rivalry in industry
c) To guide resources in proper way
d) To change Ethical environment

16. Which of these are close-ended?


a) Goals
b) Objectives
c) Both (a) & (b)
d) None of these

17. Strategy formulation, implementation, & evaluation activities should be performed


?
a) At end of year
b) Semi Annually
c) At beginning of year
d) Continual Basis

18. Which of these is not the type of strategic analysis?


a) Environmental scanning
b) Organizational analysis
c) Ratio Analysis
d) None of these

19. Which of these communicates management aspirations to stakeholders &


helps steer the energies of company personnel in a common direction?
Buy Pendrive from learnkart.in

a) Corporate Mission
b) Business Model
c) Strategic Vision
d) Goals & Objectives

20. Which is not the final stage of strategic management ?


a) Evaluating company’s progress
b) Assessing impact of external development
c) Making corrective adjustment
d) Setting Vision, Mission & Objectives

1. Which of these is made by senior management ?


rategic Plan
perational Plan
oth (a) & (b)
one of these

2. Strategic intent indicates the market position.


ong term
hort term
oth
one of these

3. Which seeks to explain business undertaken by firm, with respect to customer needs, target markets & technologie
ission
oals & Objective
usiness Definition
usiness Model

24. Which of these point out a particular direction, charts a strategic path to followed
in future ?
a) Vision
b) Mission
c) Goals
d) Objectives

25. Which is not the parameter to be assessed & controlled by mission statement ?
a) Risk
Buy Pendrive from learnkart.in

b) Cost
c) Time
d) Performance

26. Which of these are open-ended?


a) Goals
b) Objectives
c) Both (a) & (b)
d) None of these

trategic management process is .


amic
inuous
(a) & (b)
e of these

nvironmental and organizational analysis is at which stage of strategic management.

nd
d
th

Which of these delineates management’s aspirations for organization & highlights a particular direction, or strategic p
orate Mission
ness Model
egic Vision
s & Objectives

30. Which is not the principal aspect in implementation of strategy ?


a) Developing Budget
b) Staffing the organization
c) Creating Company Culture
d) Uninstalling Information system

31. Which of these involves determination of the course of action to attain


the predetermined objectives?
a) Implementation
Buy Pendrive from learnkart.in

b) Monitoring
c) Planning
d) Programing

32. can be understand as the philosophical base of strategic management.


a) Goals and objectives
b) Mission
c) Vision
d) Strategic intent

33. Which of these explain the reason for existence of firm & identify what business company undertakes?
Vision
Mission
Goals & Objectives
Business Model
34. Which of these does not explain the philosophy of organization ?
Vision
Mission
Goals & Objective
Business Model

35. Which is not the reason for having mission statement in organization ?
Ensure unanimity of purpose
Develop basis for allocating resources
Establish organizational climate
Helpful in lawful Strategy implementation

36. Business organization translate their vision & mission into ?


a) Strategy planning
b) Business Definition
c) Goals & Objectives
d) Business Model

37. What should be considered before setting long-term objectives?


a) Profitability
b) Productivity
c) Employee Relation
Buy Pendrive from learnkart.in

d) All the above


38. Strategic management process is cleanly divided and neatly performed in practice as
suggests.
a) Business Environment
b) Market Condition
c) Risk Appetite
d) Strategic Management Model

39. Top management views & conclusion about company’s direction &
product- customer market- technology focus constitute for company.
a) Corporate Mission
b) Business Model
c) Strategic Vision
d) Goals & Objectives

40. Which of these is operation-oriented, activity aimed at shaping performance of


core business activities?
a) Environmental analysis
b) Formulation
c) Implementation
d) Evolution & Control

41. means deciding what need to done in future & generating blueprint for
action ?
a) Implementation
b) Monitoring
c) Planning
d) Programing

42. Refers to purposes of what the organization strives for?


a) Strategic management
b) Strategic intent
c) Vision
d) Goals and objectives

43. Which of these delineates the firm’s business, its goals & ways to reach goals ?
a) Vision
b) Mission
c) Goals & Objectives
Buy Pendrive from learnkart.in

d) Business Model
44. Which is time-based measurable targets, which help in accomplishment of goals ?
a) Business Model
b) Objectives
c) Vision
d) Mission

45. Which of these are central to strategic planning ?


a) Vision & Mission
b) Mission & Business Definition
c) Goals & Objectives
d) Business Model & Definition

46. Which of these serves as justification for the firm’s very presence & existence ?
a) Strategic Vision
b) Corporate Mission
c) Business Definition
d) Goals & Objective

47. By spelling out management indicates speed at which longer-range targets


are to be approached?
a) Annual Performance Targets
b) Measurable short-term targets
c) Qualitative Long-term targets
d) Quantitative time-based targets

48. Which of these provides the basis for major decision of the firm & also said
the organizational performance to be realized at each level ?
a) Objectives
b) Mission
c) Business Model
d) Strategic vision

49. Which is not the stage of strategic management ?


a) Strategic Vision, Mission & Objectives
b) Environmental & Organizational Analysis
c) Strategic Evolution & Control
d) Analysis of Risk & Control
Buy Pendrive from learnkart.in

50. Which is not involved in organizational analysis ?


a) Technological resources
b) Productive Capacity
c) Social Effect
d) Research & development

51. Which strategy actually steers the firms towards success?


a) Corporate Strategy
b) Business Strategy
c) Functional Strategy
d) None of these

52. Generally, result of strategic implementation are seen on which basis?


a) Short-term
b) Long term
c) Both (a) & (b)
d) None of these

53. Which of these implies blueprint of the company’s future position &
despite organization’s aspirations?
a) Vision
b) Mission
c) Goals & Objectives
d) Business Model

54. Which is the end result that organization want to achieve ?


a) Goals
b) Objectives
c) Vision
d) Mission

55. ‚To be a world class corporate constantly furthering interest of all its stakeholders’’
is vision of .
a) Tata Motors
b) Reliance Industry
c) Microsoft
d) Amazon

56. Which of these is the expression of growth ambition of the firm ?


Buy Pendrive from learnkart.in

a) Strategic Vision
b) Corporate Mission
c) Business Definition
d) Goals & Objective

57. Which is not the characteristic of objectives ?


a) Understandable
b) Challenging
c) Measurable
d) Uncontrollable

pose of setting is to convert the strategic vision into specific performance targets – results and outcomes the manage

& soul of managing a business enterprise?


rategy
ol

ess

namic & uncertain?

1 a 11 d 21 a 31 c 51 c 61 a
2 d 12 a 22 a 32 d 52 b 62 b
3 d 13 d 23 c 33 b 53 b 63 a
4 d 14 d 24 a 34 c 54 b 64 a
5 c 15 a 25 a 35 d 55 b 65 a
6 c 16 b 26 a 36 c 56 b 66 b
7 c 17 d 27 c 37 d 57 a 67 d
8 c 18 c 28 b 38 d 58 a 68 b
9 b 19 c 29 c 39 c 59 d 69 a
10 d 20 d 30 d 40 c 60 c 70 a

CA Rachendra Mundada Best Faculty for EIS-SM all over India Page 30
Buy Pendrive from learnkart.in

Chapter – 4: Corporate Level Strategies

1. Which type of merger has no linkages with respect to customer groups, customer
functions technologies being used?
a) Vertical
b) Horizontal
c) Co-generic
d) Conglomerate

2. Strategic alliance may also be useful to create a competitive advantage by the pooling of resources & skills.
Organizational
Economic
Political
Strategic

3. Which is the first stage of turnaround strategy?


Assessment of current problems
Implementation of emergency action plan
Restructuring the business
Returning to normal

4. Which is the reason for adoption of divestment strategy?


Business acquired proves to be mismatch
Persistent negative cashflows
Severity of competition
All the above

5. At which stage currently Patanjali is & which strategy it is following ?


a) Introduction , Expansion
b) Growth , Competitive
c) Maturity , Stability
d) Decline , Turnaround

6. In which strategy firm maintains the existing level of effort, & is satisfied
with incremental growth?
a) Stability
b) Expansion

CA Rachendra Mundada Best Faculty for EIS-SM all over India Page 31
Buy Pendrive from learnkart.in

c) Retrenchment
d) Combination

7. Which is not the major reasons for stability strategy?


a) Product has reached maturity stage
b) Less risky as it involves less changes
c) Expansion may be perceived as being threatening
d) Environment faced is relatively unstable

8. Which strategy have greater control over market as well as competitor ?


a) Stability Strategy
b) Expansion Strategy
c) Retrenchment strategy
d) Combination Strategy

9. Which of these is not the type of diversification through diversification ?


a) Vertical integrated
b) Horizontally integrated
c) Co-generic
d) Conglomerate

10. When a coffee bean manufacturer may choose to merge with a coffee bean.
a) Forward Integration
b) Backward Integration
c) Concentric
d) Conglomerate

11. What is required to be focused for successful turnaround strategy?


a) Long-term
b) Short-term
c) Strategic issue
d) All the above

12. Which of these is usually a part of rehabilitation or restructuring plan & is


adopted when a turnaround has been attempted but has proved to be unsuccessful?
a) Divestment
b) Expansion
c) Stability
d) Stratification
Buy Pendrive from learnkart.in

13. Which is not the strategy to be followed at startup ?


a) Competitive Strategy
b) Collaborative Strategy
c) Expansion Strategy
d) None of these

14. Stability, Expansion, Retrenchment & Combination strategies are related with
.
a) Corporate Level
b) Business Level
c) Functional Level
d) None of these

15. Which strategy involves minor improvement with status quo oriented strategy?
a) Stability
b) Expansion
c) Retrenchment
d) Combination

16. Which is not the major reason for expansion strategy ?


a) Environment demand decrease in pace of activity
b) Organization perceived to be growth oriented
c) Greater control over market & competitor
d) Experience curve & scale of operation may accrue

17. Which of these consists marketing present products, to customer in related


market areas by adding different channels of distribution ?
a) Market Development
b) Market Penetration
c) Product Development
d) None of these

18. When firm is entering in business lines that use existing products & also when
firm enters into business of distribution channels.
a) Forward Integration
b) Backward Integration
c) Concentric
d) Conglomerate
Buy Pendrive from learnkart.in

19. When a cement manufacturer diversifies in the manufacture of steel and


rubber products.
a) Vertical Integrated
b) Horizontally integrated
c) Concentric
d) Conglomerate

20. Which of these is not the type of merger?


a) Vertical
b) Horizontal
c) Concentric
d) Conglomerate

21. The nature of strategic decisions tends to be .


a) Time Oriented
b) Risk Oriented
c) Control Oriented
d) Value Oriented

22. In how many categories, corporate level strategies can be classified ?


a) Two
b) Three
c) Four
d) Five

23. Which is not the characteristics of stability strategy ?


a) Less Risk
b) Involve minor improvement
c) Safety Oriented strategy
d) Involves redefinition of business

24. Which is not the route of expansion strategy ?


a) Intensification
b) Diversification
c) Stratification
d) None of these

25. When firm directs its resources to the profitable growth of its existing product
in existing market.
Buy Pendrive from learnkart.in

a) Market Penetration
b) Market Development
c) Product Development
d) None of these

26. When a large supermarket chain considers to purchase a number of farms


that would provide it a significant amount of fresh produce.
a) Forward Integration
b) Backward Integration
c) Concentric
d) Conglomerate

27. In which type of diversification there is no linkages exist & new product
are disjointed from existing product in every way?
a) Vertical Integrated
b) Horizontally integrated
c) Concentric
d) Conglomerate

28. Which of these often happen during recession in economy or during declining
profit margins?
a) Acquisition
b) Merger
c) Diversification
d) Stratification

29. Which of these is a combination of organizations that are unrelated to each other ?
a) Vertical
b) Horizontal
c) Co-generic
d) Conglomerate

30. When rivals can join together instead of compete, it is advantage of strategic
alliance.
a) Organizational
b) Economic
c) Political
d) Strategic
Buy Pendrive from learnkart.in

31. Which strategy is most suitable for those firm who are in the maturity stage
of product life cycle?
a) Stability
b) Expansion
c) Retrenchment
d) Combination

32. A firm with mammoth growth ambition can meet its objectives only through
.
a) Stability Strategy
b) Expansion Strategy
c) Retrenchment strategy
d) All The Above

33. Which is highly common intensification strategy ?


a) Market Penetration
b) Market Development
c) Product Development
d) None of these

34. Which of these is a step towards, creation of effective supply by entering business
of input providers?
a) Forward Integration
b) Backward Integration
c) Concentric
d) Conglomerate

35. In which type of diversification new product is a spin-off from existing facilities
& products/processes?
a) Vertical Integrated
b) Horizontally integrated
c) Concentric
d) Conglomerate

36. When organization takes over other organization & control all its
business operations, it is known as .
a) Acquisition
b) Merger
c) Penetration
Buy Pendrive from learnkart.in

d) Stratification

37. Which merger include extension of product-line or acquiring components that


are required in daily operations?
a) Vertical
b) Horizontal
c) Co-generic
d) Conglomerate

38. Finally, partners can take advantage of co-specialization, creating additional value.
a) Organizational
b) Economic
c) Political
d) Strategic

39. Which is not the danger signal for turnaround strategy?


a) Decline market share
b) Mismanagement
c) Over staffing with low moral
d) Competitive product or services

40. Which is not an important element of turnaround strategy?


a) Changes in top management
b) Initial credibility-building actions
c) Identifying quick payoff activities
d) Slow rate of cost reduction

41. Which of these is the utilization of firm’s existing facilities & capabilities in
more effective & efficient manner ?
a) Diversification
b) Penetration
c) Stratification
d) Merger & Acquisition

42. How many parts of vertical integrated diversification ?


a) Two
b) Three
c) Four
d) Five
Buy Pendrive from learnkart.in

43. In which type of diversification the new business is linked to existing


businesses through process, technology or marketing ?
a) Vertical Integrated
b) Horizontally integrated
c) Concentric
d) Conglomerate

44. When two organization combine to increase their strength & financial gains
along with breaking the trade barriers.
a) Acquisition
b) Merger
c) Penetration
d) Stratification

45. In which type of merger organizations are associated in some way or related
to production processes, business markets, or basis required technologies?
a) Vertical
b) Horizontal
c) Co-generic
d) Conglomerate

46. Strategic alliancehelp to learn necessary skill and obtain certain capabilities
from strategic partners?
a) Organizational
b) Economic
c) Political
d) Strategic

47. Which strategy is adopted by organization if it cuts off loss-making units, division
or SBUs, curtails its product line or reduces functions performed?
a) Divestment
b) Expansion
c) Stability
d) Stratification

48. Which of these is the next stage after implementing emergency action plan
for turnaround?
a) Assessment of current problems
b) Implementation of emergency action plan
Buy Pendrive from learnkart.in

c) Restructuring the business


d) Returning to normal

49. Which of these occupy highest level of strategic decision making & cover
actions dealing with objective of firm ?
a) Corporate Level Strategy
b) Business Level Strategy
c) Functional Level Strategy
d) None of these

these is formulated for each business areas like production, marketing etc.?
vel Strategies
el Strategies
evel Strategies
e

als get finalized on friendly terms & both organizations share profits in the newly created entity.

these helps to create an advantageous position by restricting supply of inputs to other players or providing input at h

53. Which is not the field of advantage with strategic alliance?


a) Organizational
b) Economic
c) Political
d) None of these

54. Which strategy is adopted by organization when if it chooses to focus on ways


& means to reserve process of decline?
a) Turnaround
Buy Pendrive from learnkart.in

b) Expansion
c) Stability
d) Stratification

55. Which of these is the next stage after implementing emergency action plan
for turnaround?
a) Assessment of current problems
b) Implementation of emergency action plan
c) Restructuring the business
d) Returning to normal

56. Which is the major reason for retrenchment/turnaround strategy?


Continuous losses & unviability
Persistent negative cashflows
Severity of competition
All the above

57. We can classify different types of strategies on the basis of .


Levels of organization
Stages of business cycle
Competition
All the above

58. Which of these is formulated for each product division known as strategic business unit?
Corporate level Strategies
Business Level Strategies
Functional Level Strategies
None of these

59. Which strategy involves keeping track of new developments to ensure that
strategy continues to make sense?
a) Stability
b) Expansion
c) Retrenchment
d) Combination

60. Which is not the characteristic of expansion strategy ?


a) Redefinition of business

CA Rachendra Mundada Best Faculty for EIS-SM all over India Page 40
Buy Pendrive from learnkart.in

b) Opposite of stability strategy


c) Less Versatile strategy
d) Fresh investment

61. Which of these is not the disadvantage of strategic alliance?


a) Sharing knowledge & skill
b) Long-term court procedures
c) Create a potential competitor
d) None of these

62. Which is the third stage of turnaround strategy to stop bleeding & enable organization to survive?
Assessment of current problems
Implementation of emergency action plan
Restructuring the business
Returning to normal

63. ‘When dead business is worth more than alive’ then which strategy should be opted by entity?
Divestment
Liquidation
Expansion
Stratification

64. Which strategy is not suggested by Michael Porter ?


Cost Leadership Strategy
Focused Strategy
Differentiation Strategy
None of these

65. Which is also known as grand strategies ?


a) Corporate level Strategies
b) Business Level Strategies
c) Functional Level Strategies
d) None of these

66. When the stability strategy is pursued by firm ?


a) Serve in Similar Market
b) Deals in similar product/service

CA Rachendra Mundada Best Faculty for EIS-SM all over India Page 41
Buy Pendrive from learnkart.in

c) Focus on incremental improvement


d) All the above

67. Which is a popular strategy that tends to be equated with dynamism, vigor,
promise & success?
a) Stability
b) Expansion
c) Retrenchment
d) Combination

ve entry with substantially various skill, technology & knowledge

rsification firm moves forward or backward in chain & enters specific process steps with intention of making them int

of related diversification?
duct portfolios
uct portfolios
bids
g brand name

71. Stability, Growth, Retrenchment & Combination are .


a) Initial Strategies
b) Environmental Strategy
c) Electric Model Strategy
d) Directional Strategy

72. Business conglomerates having multiple product folios formulate strategies at


.
a) Corporate level
Buy Pendrive from learnkart.in

b) Business Level
c) Functional Level
d) Different Levels

73. In which strategy firm seeks significant growth-maybe within current, new, related
or unrelated business ?
a) Stability
b) Expansion
c) Retrenchment
d) Combination

74. Which strategy is implemented by redefining business & enlarging scope of business
?
a) Stability
b) Expansion
c) Retrenchment
d) Combination

75. Which is not the type of expansion strategy ?


a) Through diversification
b) Through mergers & acquisition
c) Through Strategic alliance
d) Through Tough Rivalry

76. In which diversification, firms opt to engage in businesses that are not related
the existing business of firm?
a) Vertical integrated
b) Horizontally integrated
c) Concentric
d) Conglomerate

77. Acquisition of one or more similar business operating at same stage of


production- marketing chain that is going into complementary or by or competitors
product.
a) Vertical integrated
b) Horizontally integrated
c) Concentric
d) Conglomerate

78. Which is a process of combining two or more organization together?


Buy Pendrive from learnkart.in

a) Diversification
b) Acquisition or Merger
c) Strategic Alliance
d) None of these

79. Which of these is a merger of two organization that are operating in same
industry but at different stages of production or distribution system?
a) Vertical
b) Horizontal
c) Co-generic
d) Conglomerate
p between two or more businesses that enables each to achieve certain strategic objectives which neither would be ab

1 d 21 d 41 a 61 d
2 d 22 c 42 a 62 b
3 a 23 d 43 c 63 a
4 b 24 c 44 b 64 d
5 d 25 a 45 c 65 a
6 a 26 b 46 a 66 d
7 d 27 d 47 a 67 b
8 b 28 a 48 c 68 a
9 c 29 d 49 a 69 a
10 a 30 d 50 c 70 d
11 d 31 a 51 a 71 d
12 a 32 b 52 a 72 d
13 c 33 a 53 d 73 b
14 a 34 b 54 a 74 b
15 a 35 c 55 c 75 d
16 a 36 a 56 d 76 d
17 a 37 c 57 d 77 b
18 a 38 b 58 b 78 b
19 d 39 d 59 a 79 a
20 c 40 d 60 c 80 c
Buy Pendrive from learnkart.in

Chapter – 5 : Business Level Strategies


1. Which is not the basis for differentiation ?
a) Product
b) Prize
c) Organization
d) Economic of scale

2. An organization using strategy may concentrated on a particular group of


customer geographic markets or particular product line segment.
a) Cost Leadership
b) Differentiation
c) Focused
d) None of these

3. Determine whether the collective strength of five competitive forces is conducive


to earning attractive profit.
a) First step of five forces model
b) Second step of five forces model
c) Third step of five forces model

4. Which refers to decline in the per unit cost of production as volume grows?
a) Product Differentiation
b) Economic of scale
c) Switching Cost
d) Brand Identity

5. Introduction of products by new firm may lead incumbents firms to reduce


their product prices & increase their advertising budget.
a) Possibility of aggressive retaliation
b) Economic of scale
c) Access to distribution channels
d) Brand Identity

6. When rivals feel strong motivation to utilize their capacity & therefore are inclined
to cut prices when they have access capacity.
a) Slow Growth
b) Exit Barriers
c) Fixed Cost
Buy Pendrive from learnkart.in

d) Industry Leader

7. Which is not the generic strategy of Michael Porter?


a) Cost Leadership
b) Growth Share Matrix
c) Differentiation
d) Focused strategy

8. Which of these has a basic idea to underprice competitors & thereby gain
market share and sales, driving some competitors out of the market entirely?
a) Differentiation
b) Cost Leadership
c) Focused
d) None of these

9. Which of these can fluctuate based on its supply & demand and also be influence
but customer’s ideal value for product?
a) Product
b) Prize
c) Organization
d) None of these

10. Rolls Royce sells limited number of high-end, custom-built cars.


a) Only Cost Leadership
b) Only Differentiation
c) Focused Cost Leadership
d) Focused Cost Differentiation

11. Who is the foundation of an organization’s business level strategies?


a) Cost
b) Control
c) Prize
d) Customer

12. Sharing activities & resources enhances competitive advantage by _ .


a) Lowering costs
b) Raising Differentiation
c) Both (a) & (b)
d) None of these
Buy Pendrive from learnkart.in

13. Which allows a firm when it is successful, to charge a higher price for its product
& to customer loyalty?
a) Cost Leadership
b) Differentiation
c) Focused
d) None of these

14. Which strategy is most effective when consumers have distinctive preferences
or requirement & rival firms are not attempting to specialize in same target
segment?
a) Cost Leadership
b) Differentiation
c) Focused strategy
d) None of these

15. Evaluate how strong the pressure comprising each of the five forces are?
a) First step of five forces model
b) Second step of five forces model
c) Third step of five forces model
d) Fourth step of five forces model

16. Firms lacking of funds are effectively barred from the industry, thus
enhancing profitability of existing firms in industry.
a) Capital Requirement Barrier
b) Economic of scale Barrier
c) Switching Cost Barrier
d) Brand Identity Barrier

17. Which is the result of committing substantial resources for long period & it
is important for infrequently purchased products that carry a high cost to buyer?
a) Brand Identity
b) Switching Cost
c) Product Differentiation
d) Economic of scale

18. Even when an industry leader exists, leader’s ability to exert pricing
discipline diminishes with .
a) Exit barriers
b) Fixed Cost
c) Slow Growth
Buy Pendrive from learnkart.in

d) Number of competitors

19. Issue about which business level strategies are concerned with is .
a) Meeting needs of key customers
b) Achieving advantage over competitors
c) Avoiding competitive disadvantage
d) All the above

20. Which is a primary reason for pursuing forward, backward, & horizontal
integration strategies?
a) Differentiation
b) Cost Leadership
c) Focused
d) None of these

21. Rivalry among competitors tends to be cutthroat & industry profitability low when
.
a) Industry has no clear leader
b) Numerous competitors
c) Face High exit barriers
d) All the above

22. Which of these is concerned with firm’s position in an industry, relative


to competitors & to the five forces of competition?
a) Corporate level strategy
b) Business level strategies
c) Functional level strategies
d) None of these

23. Smaller firms often compete using strategy.


a) Cost Leadership
b) Differentiation
c) Focused
d) None of these

24. Product development is example of strategy that offers the advantage of .


a) Cost Leadership
b) Differentiation
c) Focused
Buy Pendrive from learnkart.in

d) None of these

25. Which is depend on an industry segment that is of sufficient size, has good
growth potential & is not crucial to success of other major competitors?
a) Cost Leadership
b) Differentiation
c) Focused Strategy
d) None of these

26. Which is not the way of best cost provider strategy?


a) Lower prize with comparable feature
b) Similar prize with more feature
c) Higher prize with same feature
d) None of these

27. Identify the specific competitive pressures associated with each of five forces.
a) First step of five forces model
b) Second step of five forces model
c) Third step of five forces model
d) Fourth step of five forces model

28. Which is not the barrier for new entrance?


a) Capital Requirement
b) Product Differentiation
c) Switching Cost
d) Access to telecom channel

29. When switching are high, buyers are often .


a) Reluctant to change
b) Ready to Change
c) Moderate about Change
d) None of these

30. Who can discourage prize wars by disciplining initiators of such activity?
a) Functional level Managers
b) Industry Leader
c) Workman
d) Strategists
Buy Pendrive from learnkart.in

31. Which refers the cost of customer to change the product & move to
another’s product?
a) Handling Cost
b) Emergency Cost
c) Switching Cost
d) Durability Cost

32. The more intensive the , the less attractive is the industry.
a) Rivalry
b) Controls
c) Strategy
d) Management

33. Which has detailed actions taken to provide value to customer & gain
competitive advantage by exploiting core competencies in specific, individual
products/services.
a) Corporate level strategy
b) Business level strategies
c) Functional level strategies
d) None of these

34. Michael Porter’s strategy imply .


a) Organizational arrangements
b) Control Procedure
c) Incentive system
d) All the above

35. Which is he disadvantage of cost leadership strategy?


a) Technological changes
b) May not be remaining for longtime
c) Depend upon high sales volume
d) All the above

36. Which is not the disadvantage of differentiation?


a) Uniqueness is difficult to sustain
b) Charging Higher prize
c) Not valued by customers
d) All the above

37. Which is the disadvantage of focused differentiation strategy?


CA Rachendra Mundada Best Faculty for EIS-SM all over India Page 50
CA Rachendra Mundada Best Faculty for EIS-SM all over India Page 50
Buy Pendrive from learnkart.in

a) Lacking in distinctive competencies


b) Higher cost of product/services
c) Niche could be Disappear
d) All the above

38. Which of these is not the element of five forces?


a) Threats of new entrance
b) Bargaining power of buyer
c) Threats of complimentary
d) Nature of industry rivalry

39. What is not the effect of new entrance in the industry?


a) Reduce industry profitability
b) Increased supply of product
c) Higher prize of product
d) Erode market share prize

40. Buyers may need to test new firm’s product, negotiable new purchase contracts,
& train personnel to use new equipment.
a) Handling Cost
b) Emergency Cost
c) Switching Cost
d) Durability Cost

41. The interrelationship among gives each industry its own particular competitive
environment.
a) Strategies
b) Resources
c) Controls
d) Porter’s Five Forces

42. Which refers to physical or enhancements, that make a product special or unique
in the eyes of customers?
a) Product Differentiation
b) Economic of scale
c) Switching Cost
d) Brand Identity

43. Suppliers bargaining power has leverage when .

CA Rachendra Mundada Best Faculty for EIS-SM all over India Page 51
Buy Pendrive from learnkart.in

a) Substitutes are not available


b) High Switching Cost
c) Sellers are more concentrated than buyers
d) All the above

44. Firms must search for products that perform the same, or nearly the same,
function as their existing products.
a) Threats of new entrance
b) Bargaining power of buyer
c) Threats of Substitutes
d) Nature of industry rivalry

45. Which means producing products & services that fulfill the needs of small groups
of consumers who are relatively price-sensitive?
a) Cost Leadership
b) Differentiation
c) Focused strategy
d) None of these

46. Which action can be taken for achieving cost leadership strategy?
a) Forecast the demand promptly
b) Optimum utilization of resources
c) Resistance to differentiation
d) All the above

47. Which is measure for achieving differentiation strategy?


a) Elevate the performance of product
b) Rapid product innovation
c) Offer utility for customers
d) All the above

48. Which is not the advantage of focused strategy?


a) Premium Prize
b) Difficulty for competitors
c) Both (a) & (b)
d) None of these

49. Which of these is a powerful & widely used tool for systematically
diagnosing significant competitive pressure in a market?
Buy Pendrive from learnkart.in

a) Porter’s five forces


b) Generic Strategy
c) Ansoff growth share matrix
d) General Electric Matrix

50. A firm’s profitability tends to be when other firms are blocked from entering
the industry.
a) Lower
b) Medium
c) Higher
d) Moderate

51.Which strategies are the course of action adopted by organization to serve


identified customer group & provide value to customer by satisfaction of their needs?
a) Corporate Level Strategy
b) Business Level Strategy
c) Functional Level Strategy
d) None of these

52. Which is one of most effective & enduring conceptual frameworks used to
assess nature of competitive environment & to describe an industry’s structure?
a) Porter’s five forces
b) Generic Strategy
c) Ansoff growth share matrix
d) General Electric Matrix

53. A large firm can produce high volumes of good at successively lower costs.
a) Capital Requirement Barrier
b) Economic of scale Barrier
c) Switching Cost Barrier
d) Brand Identity Barrier

54. Buyer bargaining power has leverage when .


a) Buyers have full knowledge
b) Buyers have invested a lot of money
c) Buyers are more concentrated than firms
d) All the above
Buy Pendrive from learnkart.in

55. Assets of firm considering maybe highly specialized & therefore of little value to
any other firm.
a) Fixed Cost
b) Slow Growth
c) Exit Barrier
d) Industry Leader

56. Which strategy aimed at producing products & services considered


unique industrywide & directed at consumers who are relatively price
incentive?
a) Cost Leadership
b) Differentiation
c) Focused strategy
d) None of these

57. Which is not the risk of pursuing cost leadership strategy?


a) Competitors may imitate the strategy
b) Technological breakthrough in industry
c) Buyers interest may swing to differentiating feature
d) Higher prize may lead to less demand of product

58. Which of these maximize the power of brand, or using specific advantages that
the entity possesses can be instrumental to a company’s success?
a) Product
b) Prize
c) Organization
d) None of these

59. Which is the measure that could be adopted by an organization?


a) Selecting specific niches
b) Creating superior skills
c) Developing innovating ways
d) All the above

60. Who believes that basic unit of analysis for understanding is a group of
competitors producing goods & services that compete directly with each other?
a) Michal Porter
b) Igor H. Ansoff
c) William F. Glueck
d) Richard Cyert
Buy Pendrive from learnkart.in

1 d 11 d 21 d 31 c 41 d 51 b
2 c 12 c 22 b 32 a 42 a 52 a
3 c 13 b 23 c 33 b 43 d 53 b
4 b 14 c 24 b 34 d 44 c 54 d
5 a 15 b 25 c 35 d 45 c 55 c
6 c 16 a 26 c 36 d 46 d 56 b
7 b 17 a 27 a 37 d 47 d 57 d
8 b 18 d 28 d 38 c 48 d 58 c
9 b 19 d 29 a 39 c 49 a 59 d
10 d 20 b 30 b 40 c 50 c 60 a
Buy Pendrive from learnkart.in

Chapter – 6: Functional level Strategies

1. Which of these includes marketing strategies to reduce demand temporarily or


permanently?
a) Concentrated Marketing
b) Differential Marketing
c) Demarketing
d) Synchro Marketing

In second approach of determining business worth times of current annual profit or years average profit can be used
ve, Five
ve, Four
hree, Two
our, Four

Delivery of products to customers is the in a marketing channel?


eginning Position
enter Position
ast Position
one of these

including human resource planning & job analysis.


re-Selection Practices
election Practices
ost Selection Practices
one of these

5. means authorizing every member of an organization to take up his/her own


destiny his/her full potential.
a) Empowerment
b) Workforce Diversity
c) Redesigned Ethics
d) Vibrant Culture

6. Which is a social & managerial process by which individual & groups obtain what
they need and want through creating, offering & exchanging products of value with
others?
a) Production
Buy Pendrive from learnkart.in

b) Marketing
c) Financing
d) Research & Development

7. Product stand for combination of that the company offers to target market.
a) Goods
b) Services
c) Both (a) & (b)
d) None of these

8. Which of these stands for company activities that make the product available to target customers?
Product
Promotion
Place
Prize

9. Which of these is Non-personal, Highly flexible & dynamic promotional method?


Personal Selling
Advertisement
Publicity
Sales Promotion

10. Marketing by environment in which market offering is delivered & where the firm & customer interact.
People
Process
Physical Evidence
Publicity

11. Which is not the approach for determining the worth of business?
a) Outstanding share method
b) Selling price Method
c) Price Earning Ratio Method
d) Trigger of Change Method

12. Which of these requires careful resource planning, quality issue, identifying
sources, negotiation, order placement, inbound transportation and storage?
a) Physical Distribution
Buy Pendrive from learnkart.in

b) Procurement
c) Performance Management
d) Product Development

13. The workforce will be more competent if a firm can successfully identify, attracts
& select the most competent applicants.
a) Recruitment & Selection
b) Appraisal of performance
c) Compensation
d) Training

14. can be observed in terms of male & female workers, young & old workers, etc.
Empowerment
Workforce Diversity
Redesigned Ethics
Vibrant Culture

15. Which of these operate bellow the business level strategies or SBU?
Corporate Level Strategy
Business Level Strategy
Functional Level Strategy
None of these

16. Which is not a ‘4P’ variable of the market mix ?


Product
Promotion
Place
Portfolio

17. In skimming the prize of product are set at a very level.


a) Low
b) Moderate
c) High
d) None of these

18. Which of these is not a cost effective way of reaching a large number of customer?
a) Personal Selling
b) Advertisement
Buy Pendrive from learnkart.in

c) Publicity
d) Sales Promotion

19. Marketing by actual procedure, mechanisms & flow of activities by


which product/service is delivered.
a) People
b) Process
c) Physical Evidence
d) Publicity

20. includes catalogue selling, e-mail, telecomputing, electronic marketing, shopping & TV shopping.
Augmented Marketing
Direct Marketing
Enlightened Marketing
Social Marketing

21. Which of these is more broader concept ?


Supply Chain Management
Product Development
Logistic Management
Performance Management

22. must develop performance incentives that clearly link performance and pay to strategies.
Human Resource Development
Product Development
Logistic Management
Manufacturing Department

23. The Human Resource manager will be more concentrated on rather than
.
a) Substance, Form
b) Accomplishment, Activities
c) Practice, Theory
d) All The Above

24. Which is not the reason why functional strategies are needed?
a) Lay down clearly what is to be done
Buy Pendrive from learnkart.in

b) Provide senses of direction to staff


c) Help in bringing harmony
d) None of these

25. Which is a set of controllable marketing variables that the firm blends to produce
the response it want in target market?
a) Market Technique
b) Market Mix
c) Market Zone
d) Market Condition

26. Which is not the objective for new product pricing strategy?
Product is acceptable to customer
Higher margin over cost
Catering to the market
None of these

27. Where oral communication is made with potential buyers of a product wit the intention of making sale?
Personal Selling
Publicity
Sales Promotion
Advertisement

28. Which is not the component of marketing?


Planning
Implementation
Control
Analysis

29. Which of these is a provision of additional customer services & benefits built
around core & actual products that relate to introduction of high-tech services?
a) Augmented Marketing
b) Direct Marketing
c) Enlightened Marketing
d) Social Marketing

30. Which of these is a market-coverage strategy in which a firm goes after a large
share of one or few sun market?

CA Rachendra Mundada Best Faculty for EIS-SM all over India Page 60
Buy Pendrive from learnkart.in

a) Concentrated Marketing
b) Augmented Marketing
c) Enlightened Marketing
d) Synchro Marketing

31. Which approach of research & development is glamorous & exciting strategy
but also a dangerous one?
a) Market new technological product
b) Innovator imitator of successful products
c) To be a low cost provider
d) None of these

32. Which is not the prominent area where human resource manager can play
strategic role?
a) Development of works ethics
b) Empowerment of human resources
c) Managing workforce Diversity
d) Restricting the change

33. Which is not the functional area of the organization?


a) Marketing
b) Financial
c) Production
d) None of these

34. Once corporate strategy has defined company’s overall mission & objectives,
plays
a role in carrying out these objectives.
a) Production
b) Marketing
c) Financing
d) Research & Development

35. Which of these is composite expression of product’s value & utility to customer,
its demand, quality etc.
a) Product
b) Promotion
c) Place
d) Prize

CA Rachendra Mundada Best Faculty for EIS-SM all over India Page 61
Buy Pendrive from learnkart.in

36. Which of these involve face to face interaction with consumers & provides a
high degree of attention to them?
a) Personal Selling
b) Publicity
c) Sales Promotion
d) Advertisement

37. Which is not included in the expanded market mix?


a) Physical Evidence
b) Promotion
c) Process
d) People

38. Publicity campaign for prohibition of smoking in Delhi explained place where
one can & can’t smoke.
a) Augmented Marketing
b) Direct Marketing
c) Enlightened Marketing
d) Social Marketing

39. Which marketing can be used when demand for product is irregular due to
season, some parts of the day, or on hour basis, causing overworked capacity?
a) Concentrated Marketing
b) Differential Marketing
c) Enlightened Marketing
d) Synchro Marketing

40. How many approaches are there to determining a business’s worth?


a) Two
b) Three
c) Four
d) Five

41. The Prominent area where the human resources manager can play strategic role
.
a) Providing purposeful direction
b) Building Core Competency
c) Managing Workforce Diversity
d) All the above
Buy Pendrive from learnkart.in

42. Which of these provide details to business strategy & govern as to how key
activities of business are to be managed?
a) Corporate Level Strategy
b) Business Level Strategy
c) Functional Level Strategy
d) None of these

43. Which of these induces or helps in moving people closer to making a decision
to purchase and facilitate a sale?
a) Production
b) Marketing
c) Financing
d) Research & Development

44. Organization formalize product differentiation through designating to


their respective products.
a) Prize Tag
b) Cost Component
c) Profit percentage
d) Brand Names

45. Which is the oldest form of promotion?


a) Personal Selling
b) Publicity
c) Sales Promotion
d) Advertisement

46. Discount, contests, money refund, instalments etc. are the various tool of .
a) Personal Selling
b) Promotion
c) Advertisement
d) Sales Promotion

47. Which of these involves monitoring & measuring of results & their evaluation?
a) Strategic Change
b) Strategic Intent
c) Strategic Control
d) None of these
Buy Pendrive from learnkart.in

48. is market coverage strategy in which a firm decides to target several market
segments & design separate offer for each.
a) Concentrated Marketing
b) Differential Marketing
c) Enlightened Marketing
d) Synchro Marketing

49. Implementation of project in pursuance of typically results in increase in


capital work in progress & current assets.
a) Stability Strategy
b) Expansion Strategy
c) Turnaround Strategy
d) Divestment Strategy

50. Which refers to the linkages between suppliers, manufacturers & customers ?
a) Supply Chain Management
b) Business process Reengineering
c) Benchmarking
d) Strategic Change

51. Which are designed to help in implementation of corporate & business unit
level strategies?
a) Corporate Level Strategy
b) Business Level Strategy
c) Functional Level Strategy
d) None of these

52. In it is more important to do what is strategically right than what is


immediately profitable.
a) Production
b) Marketing
c) Financing
d) HR

53. To succeed in today’s competitive marketplace, companies must be


centered.
a) Profit
b) Cost
c) Customer
Buy Pendrive from learnkart.in

d) Prize

54. Modern marketing is highly _ oriented.


a) Place
b) Price
c) Product
d) Promotion

55. In which form of promotion there is no requirement of payment by the organization?


a) Personal Selling
b) Advertisement
c) Publicity
d) Sales Promotion

56. Which is not included in an important marketing situation section?


a) Market Description
b) Product Review
c) Analysis of competition
d) Risk of decreased demand

57. Which of these is a marketing philosophy holding that a company’s


marketing should support the best long-run performance of marketing system?
a) Concentrated Marketing
b) Differential Marketing
c) Enlightened Marketing
d) Synchro Marketing

58. should not be thought of as a tool for limiting expenditures but rather than
as a method for obtaining most productive & profitable use of organization’s
resources.
a) Entity’s fund Analysis
b) Research & Development
c) Logistic Management
d) Financial Budget

59. Which is not the advantage of logistic management ?


a) Cost saving
b) Improved delivery time
c) Increased inventory
d) Competitive advantage
Buy Pendrive from learnkart.in

60. Which is not the R&D approach for implementing strategy?


a) Market new technological product
b) Innovator imitator of successful products
c) To be a low cost provider
d) None of these

1 c 11 d 21 a 31 a 41 d 51 c
2 a 12 b 22 a 32 d 42 c 52 b
3 d 13 a 23 b 33 d 43 b 53 c
4 a 14 b 24 b 34 b 44 d 54 d
5 b 15 c 25 d 35 d 45 a 55 c
6 b 16 d 26 d 36 a 46 d 56 d
7 c 17 c 27 a 37 b 47 c 57 c
8 c 18 a 28 d 38 d 48 b 58 d
9 b 19 b 29 a 39 d 49 b 59 c
10 c 20 b 30 a 40 b 50 a 60 d
Buy Pendrive from learnkart.in

Chapter – 7: Organization and Strategic Leadership

1. Which is an organizational form in which owner-manager makes all major


decisions directly & monitors all activities?
a) Network Structure
b) Hourglass Structure
c) Matrix Structure
d) Simple Structure

2. A by geographic area is appropriate for organizations whose strategies are


formulated to fit particular needs & characteristics of customer in different
geographic areas.
a) Network Structure
b) Hourglass Structure
c) Divisional Structure
d) Matrix Structure

3. Multi-divisional structure calls for .


a) Creating separate division, representing separate business
b) Each division would house its functional hierarchy.
c) Division Manager have responsibility to manage day-today operations
d) All the above

4. Which of these helps a multi business corporation in scientifically grouping


its businesses into a few distinct business unit?
a) Divisional Structure
b) Functional Structure
c) Strategic business unit
d) Hour Glass Structure

5. In which structure employees have two superiors, a product or project manager &
a functional manager?
a) Matrix Structure
b) Network Structure
c) Hourglass Structure
d) M-Form Structure
Buy Pendrive from learnkart.in

6. Which phase involves true dual-authority structure, both functional &


product structure are permanent?
a) Cross Functional Task Force
b) Product/Brand Management
c) Mature Matrix
d) None of theses

7. In which structure the promotion opportunity for lower levels diminish significantly?
a) M-Form Structure
b) Network Structure
c) Hourglass Structure
d) Matrix Structure

8. uses the authority of its office to exchange rewards, such as pay and statue.
a) Transformational Leadership
b) Transactional Leadership
c) Both (a) & (b)
d) None of these

9. is the attempt to create value through recognition of business


opportunity.
a) Entrepreneurship
b) Intrapreneurship
c) Both (a) & (b)
d) None of these

10. refers to company’s value, beliefs, business principles, tradition, ways of


operating and internal work environment.
a) Strategic Intent
b) Corporate culture
c) Functional Structure
d) All the above

11. Medium sized firms tend to be structured.


a) Divisionally
b) Matrix
c) Hourglass
d) Network
Buy Pendrive from learnkart.in

12. can be costly because it requires an elaborate, headquarters-driven


control system.
a) Network Structure
b) Divisional Structure
c) Hourglass Structure
d) Matrix Structure

13. Which is not the disadvantage of M-form structure?


a) Impossible to assess individual products profit contribution
b) Top managers become over involved in solving short-run problems
c) Optimal Allocation of firm resources between products was difficult
d) None of these

14. The principal underlying the grouping is that all related products-related from
the standpoint of -should fall under one SBU.
a) Control
b) Process
c) Function
d) System

15. In which structure functional & products forms are combined simultaneously at
the same level of organization?
a) Network Structure
b) Matrix Structure
c) Hourglass Structure
d) M-Form Structure

16. Which of these are initially used when a new product line is being introduced?
a) Cross Functional Task Force
b) Product/Brand Management
c) Mature Matrix
d) None of these

17. has obvious benefit of reduced costs & it also helps in enhancing
responsiveness by simplifying decision making.
a) M-Form Structure
b) Network Structure
c) Hourglass Structure
d) Matrix Structure
Buy Pendrive from learnkart.in

18. Which Leadership style motivates followers to do more originally affected to do


by stretching their abilities & increasing their shelf confidence?
a) Transformational Leadership
b) Transactional Leadership
c) Both (a) & (b)
d) None of these

19. Creating & sustaining a strategy supportive culture is a job for the team.
a) Whole Management
b) Functional Level Managers
c) Supportive Staff
d) None of these

20. Work of is extremely challenging & they get recognition & reward for success
in form of profit.
a) Entrepreneurship
b) Intrapreneurship
c) Both (a) & (b)
d) None of these

21. Changes to lead to change in organizational structure.


a) System
b) Process
c) Strategy
d) Controls

22. Which is not the advantage of divisional structure?


a) Accounting is clear
b) Performance Evaluation of employees
c) Career development opportunity
d) It is less costly structure

23. In response to co-coordination & control related problems in large firs


which structure is developed?
a) Network Structure
b) Hourglass Structure
c) M-Form Structure
d) Matrix Structure

CA Rachendra Mundada Best Faculty for EIS-SM all over India Page 70
Buy Pendrive from learnkart.in

24. Which of these facilitate comparison between divisions, improving allocation


of resources, & can be used to stimulate managers of poorly performing
divisions?
a) Strategic business unit
b) Divisional Structure
c) Functional Structure
d) M-Form Structure

25. When most organization find that organizing around either functions or around
products & geography provides an appropriate organizational structure, it is known
as
.
a) Network Structure
b) Hourglass Structure
c) M-Form Structure
d) Matrix Structure

26. Which is not the phase of Matrix structure?


a) Cross Functional Task Force
b) Product/Brand Management
c) Mature Matrix
d) Process Matrix Management

27. In which structure roll played by middle management is diminishing as the


tasks performed by them are increasingly being replaced by the technological
tools?
a) M-Form Structure
b) Network Structure
c) Hourglass Structure
d) Matrix Structure

28. Which Leadership Offers excitement, vision, intellectual stimulation and


personal satisfaction?
a) Transformational
b) Transactional
c) Both (a) & (b)
d) None of these

29. Changing a is very difficult because of the heavy anchor of deeply held values
and habits-people Cling emotionally to the old & familiar.
CA Rachendra Mundada Best Faculty for EIS-SM all over India Page 71
a) Problem Culture
b) Strategic Control

CA Rachendra Mundada Best Faculty for EIS-SM all over India Page 72
Buy Pendrive from learnkart.in

c) Support process
d) None of these

30. An is nothing but an who operates within the boundaries of


organization.
a) Entrepreneur, Intrapreneur
b) Intrapreneur, Entrepreneur
c) Both (a) & (b)
d) None of these

31. dictates how resources will be allocated to achieve strategic objective.


a) Controls
b) Structure
c) System
d) Process

32. Which is not the way by which divisional structure can be organized?
a) By Geographic area
b) By Controls
c) By Customer
d) By Process

33. Multi-Divisional or M-form structure is developed in .


a) 1890
b) 1920
c) 1980
d) 2000

34. Which of these has a manager who has responsibility for strategic planning &
profit performance?
a) Divisional Structure
b) Strategic business unit
c) Functional Structure
d) M-Form Structure

35. The identification of SBU convenient point for


a) Planning
b) Risk Analysis
c) Defining Controls
Buy Pendrive from learnkart.in

d) Monitoring

36. Which is not the condition for having matrix-structure?


a) Ideas needs to be cross fertilized
b) Resources are Enough
c) Abilities to make decision need to be improved
d) Process of information need to be improved

37. Which of these is the disadvantage of Network Structure?


a) Availability of numerous potential partners
b) Keep firm from discovering synergy
c) Choosing wrong function thus becoming Non-competitive
d) All the Above

38. use charisma & enthusiasm to inspire people to exert them for good of the
organization.
a) Transformational Leadership
b) Transactional Leadership
c) Both (a) & (b)
d) None of these

39. It is strategy maker’s responsibility to select strategy compatible with parts of


prevailing corporate culture.
a) Changeable
b) Unchangeable
c) Similar
d) None of these

40. Innovation is function of


a) Entrepreneurship
b) Intrapreneurship
c) Both (a) & (b)
d) None of these

41. largely dictates how operational objectives and policies will be established
to achieve the strategic objectives.
a) Controls
b) Structure
c) System
Buy Pendrive from learnkart.in

d) Process

42. Which of these consists of a chief executive officer or a managing director


and supported by corporate staff with manager of production, financial,
accounting department?
a) Network Structure
b) functional Structure
c) Hourglass Structure
d) Matrix Structure

divisional structure is similar to, functional structure because activities are organized according to way is actually perfo
ographic area
ct/Services
tomer
cess

hich is the characteristic of strategic business unit?


tion of related business
et of competitors
nsible for strategic planning & profit performance
above

Us might be serving & provide similar shorts of product & market.


r Market
ent market
a) & (b)
of these

46. Which structure is very useful when the external environment is very complex
& changeable?
a) Network Structure
b) Hourglass Structure
c) Matrix Structure
d) Simple Structure

47. Which of these provides an organization with increased flexibility & adaptability
tp cope up with rapid technological change & shifting patterns of international trade
&competition?
Buy Pendrive from learnkart.in

a) Simple Structure
b) Divisional Structure
c) Network Structure
d) Functional Structure

48. use charisma & enthusiasm to inspire people to exert them for good of the
organization.
a) Transformational Leadership
b) Transactional Leadership
c) Both (a) & (b)
d) None of these

49. A sizable and prop0nged conflict weakness & may even defeat managerial
efforts to make the strategy work.
a) Strategic Change
b) Strategic Intent
c) Strategic Culture
d) None of these

50. Who is not the entrepreneur ?


a) Initiates & innovates a new concept
b) Establishes a startup company
c) Doesn’t face risk & uncertainty
d) Recognizes & utilizes opportunity

51. To act and contribute as a manager and employee in today’s emerging


business scenario, skills related to process is necessary.
a) Strategic
b) Organizational
c) Leadership
d) All the above

52. To coordinate more complex organizational functions, companies should abandon


in favor of the functional structure.
a) Network Structure
b) Hourglass Structure
c) Simple Structure
d) Matrix Structure
Buy Pendrive from learnkart.in

53. A divisional structure by process is similar to , because activities are


organized according to way is actually performed.
a) Network Structure
b) Hourglass Structure
c) Functional Structure
d) Matrix Structure

54. Which of these is relevant to multi-product, multi-business enterprise?


a) Strategic business unit
b) Divisional Structure
c) Functional Structure
d) M-Form Structure

55. In strategic business unit products/businesses in any group are separated.


a) Unrelated
b) Related
c) Similar
d) None of these

56. Which structure is developed to combine the stability of the functional structure
with the flexibility of the product form?
a) Network Structure
b) Hourglass Structure
c) M-Form Structure
d) Matrix Structure

57. Which structure is most useful when environment of a firm is unstable & is
expected to retain so?
a) Simple Structure
b) Network Structure
c) Divisional Structure
d) Functional Structure

58. is a change agent to initiates strategic change in the organization and


ensure that changes successfully implemented.
a) Entrepreneurship
b) Intrapreneurship
c) Strategic Leader
d) None of these
Buy Pendrive from learnkart.in

59. Every organization has organizational culture.


a) Similar
b) Related
c) Unique
d) None of these

60. The person who perceives the business idea & take steps to implement the idea
is known
as
a) Entrepreneur
b) Strategists
c) Consultant
d) Manager

1 d 11 a 21 c 31 b 41 b 51 d
2 c 12 b 22 d 32 b 42 b 52 c
3 d 13 d 23 c 33 b 43 d 53 c
4 c 14 c 24 a 34 b 44 d 54 a
5 a 15 b 25 d 35 a 45 c 55 a
6 c 16 a 26 d 36 b 46 c 56 d
7 c 17 c 27 c 37 d 47 c 57 b
8 b 18 a 28 a 38 a 48 a 58 c
9 a 19 a 29 a 39 b 49 c 59 c
10 b 20 a 30 b 40 c 50 c 60 a
Buy Pendrive from learnkart.in

Chapter – 8: Strategy Implementation & Control

1. Sudden change in government, natural calamities, terrorist attacks, industrial disasters


etc.
a) Special Alert Control
b) Strategic Surveillance
c) Premise Control
d) Implementation Control

2. refers to the need for strategists to examine sets of trends, as well as individual trends, in auditing strategy.
Consistency
Consonance
Feasibility
Disharmony

3. A set of interconnected processes comprise a .


Business System
Analytical Program
Market Policy
Strategic Control

4. Process involving interface(s) with customer is _.


Manufacturing
Procurement
Advertising
Reporting

5. IT-initiatives, thus, provide business values in form of Innovation _.


a) By way of increased productivity
b) By way of better management
c) By way of improved products/services
d) None of these

6. Which of these requires combination among many people?


a) Strategy Formulation
b) Strategy Implementation
c) Strategy Planning
Buy Pendrive from learnkart.in

d) None of these

7. Strategies should lead to formulation of different kind of .


a) Process
b) Control
c) Project
d) Programmes

8. Determine the correct sequence of strategic change. i) Institutionalize the change, ii)
Create a shared vision to manage change, iii) Recognize need for change
a) (i),(iii),(ii)
b) (iii),(ii), (i)
c) (ii),(iii),(i)
d) (i),(ii),(iii)

9. occurs when members are psychologically impressed upon to identify them


selves with some given role models whose behavior they would like to adopt & try
to become like them.
a) Compliance
b) Identification
c) Internalization
d) None of these

10. How many types of strategic controls are there?


a) Two
b) Three
c) Four
d) Five

11. Which is not the criteria of Richard Rumelt for strategic audit?
a) Consistency
b) Consonance
c) Feasibility
d) Disharmony

12. Which is a collection of activities that creates an output of value to the customer
& often transcends departmental or functional boundaries?
a) Strategic Controls
b) Business Process
Buy Pendrive from learnkart.in

c) Analytical Programs
d) Market Policies

13. Process pertaining to development & delivery of product(s) and/or services does
not include
.
a) Manufacturing
b) Procurement
c) Engineering
d) Advertising

14. IT-initiatives, thus, provide business values in form of effectiveness .


By way of increased productivity
By way of better management
By way of improved products/services
None of these

15. The concept of is much broader than mare as there are major strategic dimensions involved.
Controlling, Benchmarking
Procedure, Programme
Benchmarking, Controlling
All the Above

16. Strategy Formulation is primarily an process.


Intellectual
Irrational
Impudent
None of these

17. Programmes lead to the formulation of .


a) Process
b) Control
c) Projects
d) Planning

18. is a complex process that involves a corporate strategy focused on new


markets, products, services and new ways of doing business.
a) Strategic Change

CA Rachendra Mundada Best Faculty for EIS-SM all over India Page 80
Buy Pendrive from learnkart.in

b) Strategic Intent
c) Strategic Control
d) None of these

19. Which of these is achieved by strictly enforcing the reward & punishment
strategy for good or bad behavior?
a) Compliance
b) Identification
c) Internalization
d) None of these

20. Which control is more inclusive & more aggregative control?


a) Management Control
b) Operational Control
c) Supportive Control
d) Processing Control

21. Which of these is a set of logically related tasks or activities oriented


towards achieving a specified outcome?
a) Strategic Controls
b) Analytical Programs
c) Market Policies
d) Business Process

22. is meant for replacement of the old process by altogether new one to
achieve dramatic improvement in the performance.
a) Benchmarking
b) Strategic Change
c) Re-Engineering
d) Strategic Leadership

23. IT-initiatives, thus, provide business values in form of efficiency .


a) By way of increased productivity
b) By way of better management
c) By way of improved products/services
d) None of these

24. Which step of benchmarking involve compiling information & data on


performance & includes mapping process?

CA Rachendra Mundada Best Faculty for EIS-SM all over India Page 81
Buy Pendrive from learnkart.in

a) Identifying the need for benchmarking


b) Identify best processes
c) Evaluation
d) Clearly understanding existing business processes

25. Which of these focuses on effectiveness ?


a) Strategy Formulation
b) Strategy Implementation
c) Strategy Planning
d) None of these

26. Strategy formulation is primarily an activity, based on strategic decision


making.
a) Entrepreneurial
b) Promotional
c) Virtualize
d) None of these

27. It is essential that managers be involved as much as possible in the strategy


formulation process.
a) Strategy Formulation
b) Strategy Implementation
c) Both (a) & (b)
d) None of these

28. Which is not the method given by H.C. Kalman for reassigning new patterns
of behavior?
a) Compliance
b) Identification
c) Internalization
d) Stratification

29. Which is not the type of organizational control?


a) Organizational Control
b) Management Control
c) Qualitative Control
d) Strategic Control
Buy Pendrive from learnkart.in

30. involves casual environmental browsing, reading financial & other


newspapers, magazines etc.
a) Special Alert Control
b) Strategic Surveillance
c) Premise Control
d) Implementation Control

31. means going to the root of the problem areas and not attempting to make
any superficial changes.
a) Fundamental Re-thinking
b) Radical Redesigning
c) Dramatic Improvement
d) All the above

32. Impact of IT system on BPR are identified by


a) Compression of time
b) Overcoming restrictions of geography
c) Restriction of relationship
d) All of these

33. Which step involves selecting the type of benchmarking & by which
organizations identify realistic opportunity for improvements?
a) Identifying the need for benchmarking
b) Identify best processes
c) Evaluation
d) Clearly understanding existing business processes

34. Which of these is manage forces during the action?


a) Strategy Formulation
b) Strategy Implementation
c) Strategy Planning
d) None of these

35. In forward linkages organizational structure has to undergo a change in the light
of the requirements of .
a) Modified Strategy
b) Old Strategy
c) Classic Strategy
d) None of these
Buy Pendrive from learnkart.in

36. Which is the issue to be considered in strategy implementation?


a) Project Implementation
b) Procedure Implementation
c) Resource Allocation
d) All the above

37. is a process of breaking down old attitudes & behaviors, customs &
traditions throughout organization.
a) Unfreezing
b) Change in new situation
c) Refreezing
d) None of these

38. intend to enable the organization to continuously learn from its experience
and to improve its capability to cope with demands of organizational growth &
development.
a) Strategic Control
b) Strategic Change
c) Strategic Intent
d) None of these

39. Which of these involves general monitoring of various sources of information


to uncover unanticipated information having a bearing on the organizational
strategy.
a) Strategic Surveillance
b) Special Alert Control
c) Premise Control
d) Implementation Control

40. Which activity is included in strategic audit process?


a) Examining underlying bases of firm’s strategy
b) Comparing expected results with actual results
c) Ensuring that performance conforms to plan
d) All the above

41. are the desired end results of the redesign process which the management
and organization attempts to realize.
a) Objectives
b) Mission
c) Goals
Buy Pendrive from learnkart.in

d) Vision

42. helps in improving performance by learning from the best practices &
the processes by which they are achieved.
a) BPR
b) Strategic Change
c) Benchmarking
d) Strategic Leadership

43. A company will be successful only when the strategy is sound & is
excellent.
a) Implementation, Formulation
b) Formulation, Implementation
c) Both (a) & (b)
d) None of these

44. Which of these deal with the impact of strategy implementation on


strategy formulation?
a) Forward Linkages
b) Backward Linkages
c) Both (a) & (b)
d) None of these

45. are required recourses for implementation of strategy.


a) Procedure
b) Project
c) Policies
d) Control

46. Which process simply makes the individuals or organizations aware of the
necessity for change & prepares them for such a change?
a) Unfreezing
b) Change in new situation
c) Refreezing
d) None of these

47. Which is a function intended to ensure & make possible the performance of
planned activities & to achieve the pre-determined goals & result?
a) Strategic Control
Buy Pendrive from learnkart.in

b) Strategic Change
c) Strategic Intent
d) None of these

48. is a tool for systematic & continuous monitoring of the environment to


verify the validity & accuracy of the assumption on which strategy is built.
a) Strategic Surveillance
b) Special Alert Control
c) Premise Control
d) Implementation Control

49. Which of these involves complete reassessment of strategy & also assesses
thee need to continue or refocus the direction of an organization?
a) Milestone Reviews
b) Premise Control
c) Special Alert Control
d) Monitoring Strategic Thrusts

50. are simply a set of activities that transforms a set of inputs into a outputs
for another person.
a) Business Process
b) Strategic Controls
c) Analytical Programs
d) Market Policies

51. is a point of reference against which things maybe compared and by


which something can be measured and judged.
a) BPR
b) Strategic Change
c) Strategic Leadership
d) Benchmarking

52. requires introduction of change in the organization to make


organizational member adapt to the new environment.
a) Strategic Intent
b) Strategic Planning
c) Strategy Crafting
d) Strategy Implementation
Buy Pendrive from learnkart.in

53. Which of these deal with the impact of strategy formulation on


strategy implementation?
a) Forward Linkages
b) Backward Linkages
c) Both (a) & (b)
d) None of these

54. is a highly specific programme for which the time schedule and cost are
predetermined.
a) Project
b) Procedure
c) Policies
d) Control

55. It takes so much time for the to Change & prevail.


a) Process
b) Controls
c) Culture
d) Programs

56. when new behavior becomes a normal way of life.


a) Unfreezing
b) Change in new situation
c) Refreezing
d) None of these

57. When strategy is formed on the basis of certain assumption about the complex
& turbulent organizational environment.
a) Premise Control
b) Strategic Surveillance
c) Special Alert Control
d) Implementation Control

58. Which helps the managers to determine whether the overall strategy is
progressing as desired or whether there is need for readjustment?
a) Milestone Reviews
b) Premise Control
c) Special Alert Control
d) Monitoring Strategic Thrusts
Buy Pendrive from learnkart.in

59. Competitive advantage normally is the result of superiority in area.


a) Resources
b) Skills
c) Position
d) All the above

60. Re-Engineering begins with a fundamental .


a) Redesigning
b) Control
c) Process
d) Re-thinking

61. strategy affects an organization from top to bottom; it affects all


functional & dicisional areas of a business.
a) Planning
b) Implementation
c) Forming
d) Crafting

62. Which of these varies substantially among different types & sizes of
the organization?
a) Strategy Formulation
b) Strategy Implementation
c) Strategy Planning
d) None of these

63. Which is broader term?


a) Programme
b) Policies
c) Procedures
d) Goals & Rules

64. Which of these is an action stage that requires implementation of changed strategy.
a) Institutionalize the change
b) Create a shared vision to manage change
c) Recognize need for change
d) None of these
Buy Pendrive from learnkart.in

65. Which of these involves internal changing of the individual’s thought processes
in order to adjust to a new environment?
a) Compliance
b) Identification
c) Internalization
d) None of these

66. Which of these is not the type of strategic control?


a) Premise Control
b) Strategic Surveillance
c) Special Alert Control
d) Processing Control

67. Strategic Implementation control unlike continuously monitors the basic


direction of the strategy.
a) Special Alert Control
b) Operational Control
c) Premise Control
d) Management Control

68. Question that is ‚Can strategy be attempted within the physical, human &
financial resources of enterprise" is related with which criteria of strategic audit?
a) Consistency
b) Consonance
c) Feasibility
d) None of these

69. means starting all over, starting from scratch.


a) Benchmarking
b) Strategic Change
c) Business Process Re-Engineering
d) Strategic Leadership

70. Determine the correct sequence of steps in BPR. i) Study the existing ii) Formulate
a redesign process iii, Implement the redesign iv) Identify customers & Determine their
need, v) Determining Objectives and Framework
a) (ii),(iv),(v),(i),(iii)
b) (iii),(iv),(ii),(v),(i)
c) (v),(iv),(i),(ii),(iii)
Buy Pendrive from learnkart.in

d) (iv),(i),(v),(iii),(ii)

1 a 11 d 21 d 31 b 41 a 51 d 61 b
2 b 12 b 22 c 32 d 42 c 52 d 62 b
3 a 13 d 23 a 33 a 43 b 53 a 63 a
4 c 14 b 24 d 34 b 44 b 54 a 64 a
5 c 15 c 25 a 35 a 45 b 55 c 65 c
6 b 16 a 26 a 36 d 46 a 56 c 66 d
7 d 17 c 27 c 37 a 47 a 57 a 67 b
8 b 18 a 28 d 38 a 48 c 58 d 68 c
9 b 19 a 29 c 39 a 49 a 59 d 69 c
10 c 20 a 30 b 40 d 50 a 60 d 70 c

CA Rachendra Mundada Best Faculty for EIS-SM all over India Page 90
lOMoARcPSD|6004607

Multiple Choice Questions

Strategic Management (Brunel University London)

StuDocu is not sponsored or endorsed by any college or university


Downloaded by Imad Quadri (imad3889@yahoo.co.in)
Multiple Choice Questions

Chapter 1 - Introduction Strategy

Which of the following are typically seen as being associated with strategic decisions?
A.
The organisation's long-term direction
B.
The detailed planning of a department's work over the next month
C.
The values and expectations of powerful actors in the organisation
D.
The scope of the organisation's activities

Which strategy is about how to compete successfully in particular markets?


A.
Business-level strategy
B.
Corporate-level strategy
C.
Alliance-based strategy
D.
Operational-level strategy

A group of managers is considering pricing strategy and differentiation. At which level of strategy are
the managers most likely to be working?
A.
Corporate level
B.
Operational level
C.
Business level
D.
Mission and vision

An organisation's general expression of its overall purpose is known as its:


A.
objective
B.
vision
C.
goal
D.
mission

Which of the following terms correctly complete the definition: Operational strategies are about
how the component parts of an organisation deliver strategies in terms of and .
A.
people
B.
alliances
C.
mission
D.
resources
E.
Processes

Strategy involves:
A.
senior managers and board members
B.
managers at all levels
C.
senior and middle managers
D.
senior management

What are the three main branches of strategy research that make up the study of strategy?
A.
Strategy content
B.
Strategic context
C.
Strategy lenses
D.
Strategy processes

In the Exploring Strategy Model, what heading is used to cover environment, capability, goals
and culture?
A.
Strategic applicability
B.
Strategic choices
C.
Strategy in action
D.
Strategic position

A design view of strategy refers to:


A.
the pulling together of ideas that develop from different parts of the organisation.
B.
the pulling together of the different decisions made throughout an organisation, so as to develop a
coherent overall strategy.
C.
the systematic, rational way in which strategy is always developed in an organisation.
D.
the deliberate positioning of the organisation through a rational, analytic, structured and directive
process.

The experience lens suggests that strategies develop:


A.
through the shared assumptions in the organisation, often thought of as the organisational culture.
B.
through the shared assumptions across similar types of organisations within an industry (or
organisational field).
C.
through the individual experience of a few top managers or strategic planners.
D.
All of the above.

The variety lens suggests that new strategies take shape in organisations:
A.
because new ideas are tried out in the market and either succeed or do not.
B.
because the new ideas that develop from within it are selected by formal evaluation through strategic
planning systems.
C.
because there are sufficient people who find them attractive.
D.
All of the above

Chapter 2 – The Environment


What is the key outcome from PESTEL analysis?
A.
Five Forces
B.
Identification of the drivers for change
C.
Critical success factors
D.
Possible scenarios

When using PESTEL it is easy to get overwhelmed by a multitude of details. Instead, it is important to
step back and identify the:
A.
key drivers for change
B.
relevant Five Forces that exist
C.
complex links between each of the factors
D.
market segments

Which three of the following are the most useful ways in which a group of managers could use
scenario planning?
A.
To consider plausible alternative futures
B.
To ensure that the managers always select the only scenario that will work in practice
C.
To develop contingency plans for each scenario
D.
To increase the managers' understanding and perception of forces in the business environment

It is always useful to ensure that the three scenarios are 'optimistic', 'middling' and 'pessimistic'.
A.
False
B.
True
The five forces that affect the level of competition in an industry are:
A.
the threat of entry; the power of buyers; the power of suppliers; the threat of substitutes; and
government action.
B.
the threat of buyers; the power of entry; the power of substitutes; the threat of suppliers; and the
threat of recession.
C.
the threat of recession; the power of buyers; the power of suppliers; the threat of management failure;
and competitive rivalry.
D.
the threat of entrants; the power of buyers; the power of suppliers; the threat of substitutes; and
competitive rivalry.

In which of the following situations is buyer power likely to be high?


A.
Where switching costs are low
B.
Where ultimate consumer power is weak
C.
Where the buyer can threaten to compete
D.
Where a few large customers account for the majority of sales

Forward vertical integration occurs when suppliers are able to cut out buyers who act as
intermediaries.
A.
True
B.
False

When identifying strategic groups, which two headings can the relevant characteristics most usefully
be grouped under?
A.
Resource commitment
B.
Competitiveness
C.
Scope of activities
D.
PESTEL factors

A strategy canvas is a simple but useful way of comparing competitors' positions in a market and
potential in different segments.
What are the two key benefits of a strategy canvas?
A.
It emphasises the importance of technical quality.
B.
It demonstrates that focusing on market segments means losing the overall picture.
C.
It emphasises the importance of seeing value through the customers' eyes.
D.
It emphasises the need to be clear about relative strengths.
Chapter 3 – Strategic Capabilities

Which of the following accurately categorises the machines an organisation uses?


A.
Tangible, financial resources
B.
Intangible, financial resources
C.
Tangible, intellectual capital
D.
Tangible, physical resources

What term is used for an organisation's abilities to renew and recreate its strategic capabilities to meet
the needs of a changing environment?
A.
Competent substitution
B.
Core competence
C.
Renewability
D.
Dynamic capabilities

Core competences are the skills and abilities by which resources are deployed through
an organisation's activities and processes such as to:
A.
survive using approaches and techniques that others cannot imitate or obtain.
B.
survive.
C.
achieve competitive advantage in ways that others cannot imitate or obtain.
D.
achieve competitive advantage.

A competitor finds it difficult to identify the basis for an organisation's competitive advantage. What
term is used for this situation?
A.
Interdependent causality
B.
Causal dependency
C.
Causal ambiguity
D.
Ambiguous intercausality

Which of the following statements correctly relate to explicit and tacit knowledge?
A.
A systems manual is an example of explicit knowledge.
B.
Tacit knowledge is easier to imitate.
C.
Explicit knowledge is easier to communicate.
D.
Tacit knowledge is personal, context-specific and therefore hard to communicate.
The sharing of knowledge and experience in organisations is an essentially social and cultural
process.
A.
False
B.
True

Which of the following statements correctly relate to value chains?


A.
Technology development is a primary activity.
B.
Marketing and sales is a support activity.
C.
Procurement is a support activity that occurs in many parts of the organisation.
D.
Operations are primary activities that transform inputs into the final product or service.

The purpose of a SWOT analysis is to analyse:


A.
the strategic capability of an organisation.
B.
external and internal environments.
C.
the business environment and the strategic capability of an organisation relative to its competitors.
D.
the business environment in which an organisation operates.

Best-in-class benchmarking seeks to assess organisational performance against:


A.
the competitor who is 'best in class' wherever that may be.
B.
the nearest geographical competitor.
C.
the nearest principal competitor.
D.
the competitor who is the best in the industry.

Chapter 4 - Strategic Purpose

Which of the following answers the question: 'Where does the organisation aspire to be in the future?'
A.
Mission statement
B.
Core values
C.
Vision statement
D.
Objectives

Which two statements accurately describe relationships that exist in a large company?
A.
The board is an agent for investment managers.
B.
Investment managers are ultimately agents for the ultimate beneficiaries.
C.
Investment managers are agents for the board.
D.
The board is an agent for managers.

Which are the two most common types of governance structure?


A.
The non-executive board model
B.
The
manager–agent
model C.
The shareholder model
D.
The stakeholder model

Which of the following statements accurately relates to the stakeholder model of governance?
A.
Shareholders have a legitimate primacy in relation to the wealth generated by organisations.
B.
Boards attempt to consider the wishes of all stakeholders.
C.
All board members are insiders (typically managers of the company).
D.
Firms generally have a single-tier structure.

Which of the following are claimed to be advantages of the shareholder model of governance?
A.
Investors get higher returns than they would under the stakeholder model.
B.
There is a reduced risk for shareholders.
C.
Major investors are more likely to view their investments as being long term.
D.
There is a greater chance of entrepreneurship.

Which of the following is not a corporate social responsibility stance?


A.
Enlightened self-interest
B.
Free enterprise
C.
Laissez-faire
D.
Shaper of society
E.
Forum for stakeholder interaction

Which stance on corporate social responsibility would focus on social and market change, and give
individuals in the organisation responsibility for social responsibility issues?
A.
Enlightened self-interest
B.
Forum for stakeholder interaction
C.
Laissez-faire
D.
Shaper of society

What name is given to the process of divulging information to outside bodies when a person believes
that their organisation is failing in its corporate socialresponsibility?
A.
Showing enlightened self-interest
B.
Social auditing
C.
Legitimising
D.
Whistle-blowing

An indicator of the power held by external stakeholders is:


A.
the organisational perception of the status of an external party.
B.
the personal relationship with a key decision maker.
C.
mutual resource dependency.
D.
their negotiating skills.

Power is:
A.
the ability of individuals or groups to persuade, induce or coerce others into following certain courses
of action.
B.
the ability of individuals or groups to persuade others into following certain courses of action.
C.
the ability of groups to persuade, induce or coerce others into following certain courses of action.
D.
the ability of individuals to persuade, induce or coerce others into following certain courses of action.

Which of the following is not one of the four broad categories of external stakeholders?
A.
Environmental
B.
Technological
C.
Economic
D.
Socio-political
Chapter 5 – Culture and Strategy

In many businesses there are periods of relative continuity during which strategy changes
incrementally. What are the three main reasons for this?
A.
By maintaining continuity during a period of environmental change, managers can ensure growth.
B.
Managers are experimenting around a theme.
C.
Managers are unwilling to change a strategy that has been successful.
D.
The environment is changing gradually.

Identify two reasons for the development of core rigidities that make it difficult for organisations to
change.
A.
Relationships are crucial to the change process.
B.
Organisational practices are developed over time. These practices may become embedded in
organisational routines that are difficult to change.
C.
The methods that have delivered past success can become taken for granted.
D.
Performance effects lag the development of core techniques.

Which of the following accurately relate to transformational change?


A.
It is usually the result of a downturn in performance.
B.
It often occurs too early to result in an improvement in market position or jobs.
C.
It does not take place frequently.
D.
It generally occurs in response to changes suggested by existing middle management.

Which of the following are reasons why the history of the organisation is important for strategists?
A.
It helps detect and avoid strategic drift.
B.
It avoids misattributing the reasons for success.
C.
It focuses the manager purely on the relevant organisation.
D.
It avoids the recency bias.

What term is used for a situation where early events and decisions establish policy paths that have
lasting effects on subsequent events and decisions?
A.
Historicisation
B.
Strategic drift
C.
Cyclical strategy
D.
Path dependency
Which of the following is not a way of carrying out an historical strategic analysis?
A.
Rigidity analysis
B.
Historical narratives
C.
Cyclical influences
D.
Chronological analysis
E.
Key event and decision analysis ('anchor points')

Which approach to historical analysis would be most suitable for a manager wanting to assess
an organisation's attitudes to markets, customers and change?
A.
Cyclical influences
B.
Anchor points
C.
Key events and decisions ('anchor points')
D.
Historical narratives

Which of the following is not one of the four layers of organisational culture?
A.
Values
B.
Beliefs
C.
Paradigm
D.
History

Which of the following best describes the cultural web?


A.
A representation of the politics in an organisation
B.
A representation of the power in an organisation
C.
A representation of the taken-for-granted assumptions, or paradigm, of an organisation and its
competitors
D.
A representation of the taken-for-granted assumptions, or paradigm, of an organisation and the
physical manifestations of the organisation culture

Which three of the following questions are most likely to be useful when analysing the cultural web of
an organisation?
A.
Who are the heroes and villains?
B.
How is power distributed?
C.
How has the organisation been affected by strategic drift?
D.
Is emphasis on reward or punishment?
Which two of the following explain why the 'taken-for-granted' nature is centrally important in relation
to strategy and the management of strategy?
A.
Because this inevitably means that the culture gives an inaccurate picture of the organisation
B.
Because the culture does not apply directly to managers
C.
Because organisations can be 'captured' by their culture and find it difficult to change their strategy
outside that culture
D.
Because it is difficult to observe, identify and control, culture is difficult to manage

Organisational culture has largely been created by previous management actions and decisions, so
can relatively easily be changed in the future.
A.
False
B.
True

'That approach was tried last year and failed.' Which two of the following apply to this statement?
A.
It is an example of a routine or ritual.
B.
It is an example of a rigid control system.
C.
It is an example of something that is taken for granted.
D.
This statement could only be made in a flat organisational structure.

When describing organisational culture some writers use phrases such as 'the way things are
around here'.
In relation to this comment, which of the following is the most useful task of strategists?
A.
To build a future based solely on the existing culture
B.
To investigate the taken-for-granted aspects of the culture
C.
To make changes in control systems so that the culture changes
D.
To make structural change so that the culture changes

Chapter 6 – Business Strategy

What is the key benefit of identifying the organisation's SBUs?


A.
It helps the development of business-level strategies.
B.
It makes financial control easier.
C.
It prevents a focus solely on market-based criteria.
D.
It decreases the complexity of the organisation's structures.
Which of the following are generally used when identifying SBUs?
A.
Market-based criteria
B.
Structurally-based criteria
C.
Capabilities-based criteria
D.
Finance-based criteria

What is meant by focused differentiation?


A.
Providing a high perceived value service or product to a selected market segment that justifies a
substantial price premium
B.
Simultaneously seeking to achieve differentiation and a price lower than that of
competitors C.
Concentrating on a particular feature of a product or service to achieve differentiation
D.
Concentrating on differentiation as the primary means of achieving competitive advantage

A differentiation strategy is defined as:


A.
the provision of products or services that offer benefits different from those of competitors and that are
widely valued by buyers.
B.
the innovation of products or services greater than the competition.
C.
higher quality products or services than those of competitors.
D.
the provision of different products or services that draw upon competences or resources which
competitors do not have.

How might an organisation sustain and win a price war?


A.
By cross-subsidising one business from another
B.
By having 'deeper pockets' to fund short- to medium-term losses
C.
By having a lower cost structure
D.
All of the above

Which of the following are key principles for successful competition in hypercompetitive conditions?
A.
Misleading the competition
B.
Maintaining constant strategies
C.
Developing new bases of success
D.
Making a series of small moves (rather than big moves)
This is the correct answer.
E.
Being unpredictable
Which one of the following best explains the aim of collaboration?
A.
To achieve advantage
B.
To avoid competition
C.
Neither to achieve advantage nor to avoid competition
D.
To achieve advantage or avoid competition

Which of the following could be major benefits for a seller that collaborates with a major customer in a
technological industry such as aerospace or carmanufacturing?
A.
It may enable joint research and development
B.
Increased seller power
C.
It increases the buyer's power
D.
It enables the customer to increase barriers to entry

Many governments have promoted, or required, collaboration between buyers of pharmaceuticals and
centralised government drug-specifying agencies. What is the outcome of these moves?
A.
Greater sensitivity to end-user requirements
B.
Increased selling power
C.
Increased buyer power
D.
Greater barriers to entry

What are the two key assumptions in understanding competitive dynamics in terms of game theory?
A.
Competitors may opt to follow game rules of their own choosing.
B.
Competitors are in an interdependent relationship.
C.
Competitors approach business as though it was a game, so do not always behave rationally.
D.
Competitors will behave rationally in trying to win their own benefit.

Based on the two basic assumptions of game theory, which two principles guide the development of
successful competitive strategies?
A.
'Think forwards and reason backwards'
B.
'Analyse forwards and think backwards'
C.
'Get in the mind of the competitors'
D.
'Assume that your competitor cannot get in your mind'
Consider the example of a company that is always battling on the basis of price, but realises that with
its cost structure it cannot hope to compete effectively. What, according to game theory, should it do?
A.
Change the rules
B.
Avoid all collaboration
C.
Continue with current strategies, knowing that the competition will act irrationally
D.
Continue with its current strategies

Chapter 7 – Corporate Strategy and Diversification

What are the two constraints most likely to face organisations seeking greater market penetration?
A.
The risk of downsizing
B.
The need to consolidate market
share C.
Legal constraints
D.
Retaliation from competitors

In what three ways may market development take place?


A.
By focusing on new users
This is the correct answer.
B.
By focusing on efficiency of production
C.
By focusing on new geographies
D.
By focusing on economies of scale

What term is used for corporate development beyond current products and markets, but within the
capabilities or the value network of the organisation?
A.
Backward integration
B.
Related diversification
C.
Vertical integration
D.
Divergent diversification

Which two of the following are most likely to be sources of conglomerate value creation?
A.
Exploiting dominant logics rather than concrete operational
relationships B.
Divestment
C.
Entering markets of high risk
D.
Entering countries with underdeveloped markets
What is meant by diversifying through vertical or horizontal integration?
A.
Vertical integration describes either backward or forward integration into adjacent activities in the
value network. Horizontal integration is development into activities that are complementary to present
activities.
B.
Vertical integration is where a firm diversifies activities that are inputs into the company's current
business. Horizontal integration refers to diversification into activities that are concerned with
the company's outputs.
C.
Vertical integration is where a firm diversifies into activities that are competitive with or complementary
to its current activities. Horizontal integration is either backward or forward integration into adjacent
activities in the value network.
D.
Vertical integration is concerned with ensuring that all the activities of the organisation are
well coordinated. Horizontal integration is concerned with coordination of activities with buyers
and suppliers.

A film company and a music recording company may choose to combine, believing that the result will
be more effective than the sum of the two component parts. What term is used for the benefits?
A.
Synergy
B.
Diversification
C.
Integration
D.
Consolidation

Diversification may create efficiency gains by applying the organisation's existing resources or
capabilities to new markets, products or services. These gains are known as economies of scale.
A.
False
B.
True

What does conventional finance theory say about the spreading of risk for shareholders when a
company diversifies?
A.
There is significant benefit, provided the diversification leads to synergy.
B.
There is little benefit to shareholders as they have already spread their risk by holding a range of
shares.
C.
There is significant benefit, provided the diversification leads to an increase in corporate parenting
capabilities.
D.
There are always significant benefits from the reductions in risk.
Which of the following definitions explains what is meant by the corporate parent?
A.
The central head office of the organisation
B.
The founder of the business
C.
The owner or major shareholder of the corporation
D.
The levels of management above that of business units

Which three of the following are the key criteria that should be considered in relation to a multi-
business portfolio?
A.
Potential problems
B.
Attractiveness
C.
Balance
D.
Fit
E.
Synergy

A particular business unit operates in a low-growth, mature market, in which it has a large market
share. What term is used in the BCG matrix for this business?
A.
Ballast
B.
Cash cow
C.
Star
D.
Harvest/divest

Chapter 8 - International Strategy

Which three of the following are categories of cost drivers of internationalisation?


A.
Scale economies
B.
Similar customer needs
C.
Country-specific differences
D.
Favourable logistics

Interdependence between country operations increases the pressure for global coordination.
A.
False
B.
True
Some brands, such as Coca-Cola, are able to successfully market in very similar ways around the
world. What is this called?
A.
Global customisation
B.
Scope economies
C.
Transferable marketing
D.
Favourable logistics

Porter's Diamond has been used by governments aiming to increase the competitive advantage of
their local industries. Which of the following are examples of this?
A.
Governments have encouraged competition rather than protect home-based industries.
B.
Governments have encouraged the growth of clusters of related industries.
C.
Governments have raised tariff barriers to protect local industries.
D.
Governments have encouraged the growth of industries previously unrepresented in the local
economy.

Internationalisation is increasingly not about exploiting existing capabilities in new national


markets. A.
False
B.
True

What name is given to the purchase of components and services from the most appropriate suppliers
around the world regardless of location?
A.
International compartmentalisation
B.
Multidomestic marketing
C.
Global sourcing
D.
The
global–
local
dilemma

The global–local
dilemma in international strategy means:
A.
the issues related to globalisation and the alleged disadvantaging of developing countries.
B.
how many local people to employ in foreign subsidiaries.
C.
whether to centralise strategic decisions in head office or to devolve decision making to subsidiaries.
D.
the extent to which products and services may be standardised across national boundaries or need to
be adapted to meet the requirements of specific national markets.
Which international strategy has a dispersed configuration and low levels of coordination of
international activities?
A.
Transnational
B.
Global
C.
Multidomestic
D.
Export

What do the four elements of Gehemawat's CAGE represent?


A.
Cultural distance, administrative and political distance, geographical distance and environmental
distance
B.
Cultural distance, administrative and political distance, geographical distance and economic
C.
Country distance, administrative and political distance, geographical distance and environmental
distance
D.
Country distance, administrative and political distance, gross distance and economic distance

Which three of the following are the key factors to consider when assessing international retaliation?
A.
The reactiveness of the defender
B.
Five Forces analysis
C.
The clout that a defender can muster
D.
The attractiveness of the market to the new entrant
E.
PESTEL factors

What term is used for an approach where firms initially use entry modes that allow them to maximise
knowledge acquisition while minimising the exposure of their assets?
A.
Multidomestic expansion
B.
International competitor elimination
C.
Foreign direct investment
D.
Staged international expansion

Chapter 9 – Innovation and Entrepreneurship

The view that innovation involves technologists creating new knowledge, which then forms the basis
for new products for the rest of the organisation to produce andmarket, is known as market push.
A.
True
B.
False
Which of the following statements correctly describe product or process innovation?
A.
Small new entrants typically have the greatest opportunity in the mature stage of an industry.
B.
Industries typically favour product innovation.
C.
Product innovation typically precedes process innovation.
This is the correct answer.
D.
New developed industries typically favour product innovation.

What term is used for the process by which innovations spread among users, varying in pace
and extent?
A.
The tipping point
B.
Incremental innovation
C.
Radical innovation
D.
Diffusion

Wherever possible, companies should aim to be first-movers.


A.
False
B.
True

Which of the following are likely to be late-mover advantages?


A.
Pre-emption of scarce resources
B.
Gaining experience curve benefits
C.
Learning
D.
Free-riding

Which of the following are the contextual factors that managers should consider when deciding
whether to move first or not?
A.
The speed of change in the market
B.
The shape of the experience curve
C.
The organisation's capacity for profit capture
D.
The availability of complementary assets

Which of the following is most likely to be a key issue during the start-up stage?
A.
Releasing capital as a reward
B.
Changing to intrapreneurship
C.
Sources of capital
D.
Changing from the role of entrepreneur to manager
What name is given to people who set up a succession of enterprises, investing the capital raised on
exit from an earlier venture into new growing ventures?
A.
Serial entrepreneurs
B.
Serial venturists
C.
Venture capitalists
D.
Intrapreneurs

Which of the following are included in the IBM ?ecosystem'?


A.
Franchisees
B.
Competitors
C.
Makers of complementary products
D.
Suppliers

Which three of the following are key issues that a social entrepreneur must consider at start-up?
A.
The development of ecosystems
B.
Business model
C.
Social mission
D.
Organisational form

Many social enterprises take on cooperative forms. What are the two main disadvantages of
this form?
A.
Cooperatives can be slow to take hard decisions.
This is the correct answer.
B.
It is difficult to keep financial records.
C.
It increases the number of levels in the hierarchy.
D.
It may be difficult to decide which stakeholders to include or exclude.

Kanter considers that there are benefits to business of involvement with social enterprise. Which of
the following are among the specific benefits that Kanter identifies?
A.
Gaining first-entrant advantages in foreign markets
B.
Developing new technologies and services
C.
Attractive publicity
D.
A feel-good factor
E.
Accessing new pools of potential employees
Chapter 10 – Mergers, Acquisitions and Alliances

Common problems in making acquisitions work relate to:


A.
lack of cultural fit.
B.
failure to add value, inability to integrate the new company, lack of organisational learning and poor
cultural fit.
C.
the two companies having different core competences.
D.
failure to add value and inability to integrate the new company.

Which three of the following are most likely to be motives for acquisitions and mergers?
A.
To increase capabilities
B.
To create consolidation opportunities
C.
To use existing capabilities more successfully
D.
To increase speed of entry into a rapidly changing market

What term is used for M&A integration in which it is implied that both the acquired firm and the
acquiring firm learn the best qualities from the other?
A.
Preservation
B.
Symbiosis
C.
Holding
D.
Absorption

The Royal Bank of Scotland's consortium competed with Barclays Bank to acquire the Dutch bank
ABN AMRO: the Royal Bank of Scotland won, but the excessive price of'70bn (~$98bn) soon drove
the victor into financial collapse and government ownership. What term is used for this situation?
A.
Winner's curse
B.
Hubris
C.
Acquisition
D.
Excessibility

In the context of strategic alliances, what is meant by the term 'collaborative advantage'?
A.
The benefits of being part of a network of alliances of which an organisation is a member
B.
The aim of two or more organisations in sharing resources and activities to pursue a strategy
C.
The benefit of creating a new entity that is owned separately by the partners involved
D.
The result of managing alliances better than competitors
Which of the following is not a stage that occurs when two organisations form and eventually dissolve
an alliance?
A.
Courtship
B.
Negotiation
C.
Storming
D.
Maintenance
E.
Start-up
F.
Termination

Networks differ from joint ventures in that networks:


A.
are arrangements whereby two or more organisations work in collaboration without creating a new
formal entity, but where there is mutual advantage in doing so.
B.
are based on personal relationships.
C.
are arrangements whereby two or more organisations work in collaboration with the creation of a new
formal entity.
D.
are limited to e-commerce businesses.

Chapter 11 – Evaluating Strategies

Analysis suggests that a company could find a strategy that gains market share for advantage, and
that exploits its superior resources and competences. The organisational culture suggests that it
should stick to what it knows best. What strategy would you suggest?
A.
Diversification
B.
Retrenchment
C.
Market penetration
D.
Market development

Analysis suggests that a company's existing markets are saturated. The company wants to exploit its
strategic capabilities in new arenas and satisfy its stakeholders by making rapid growth. What strategy
would you suggest?
A.
Retrenchment
B.
Market development
C.
Diversification
D.
Market penetration

If managers use their judgement when applying the techniques, the criteria of suitability, acceptability
and feasibility will identify the best strategy.
A.
True
B.
False
What is most often the limitation when assessing return using
cost–benefit
analysis?
A.
Difficulty in quantification
B.
Clear identification of the key stakeholders
C.
Difficulties in establishing the timescales to be applied
D.
Identifying objectives of the strategy

Profitability analyses for assessing the acceptability of a strategy include:


A.
return on capital employed (ROCE); ratio analysis; and funds flow analysis.
B.
return on capital employed (ROCE); payback period; and discounted cashflow (DCF).
C.
payback period; discounted cashflow; and decision trees.
D.
ranking; decision trees; and scenarios.

'Acceptability' assessment concerns:


A.
the strategic fit of the strategy to the future trends and changes in the environment.
B.
the resources and competences required to implement the strategy.
C.
the expected performance outcomes, such as risk, return and stakeholder reactions, if a strategy is
implemented.
D.
the stakeholder reaction to a strategy.

Chapter 12 – Strategy Development


Which of the following best describes a 'deliberate' strategy?
A.
A strategic direction that emerges from a stream of decisions
B.
A strategy that is the product of competitive pressure
C.
An expression of desired strategic direction, intentionally formulated or planned by managers
D.
An expression of desired strategic direction deliberately formulated or planned by managers that is
realised in the fullness of time

The purposes of strategic planning systems are:


A.
to assist in the communication, coordination and control of strategy.
B.
to provide a structured means of analysing and thinking about complex strategic problems and
expanding the planning horizons of strategists.
C.
to develop ownership of the strategy.
D.
All of the above.
Which of the following is not one of the four main dangers with formal planning that
Mintzberg identifies?
A.
Dampening of innovation
B.
Detachment from reality
C.
Not providing a sense of security and logic
D.
Confusing strategy with a plan
E.
Over-complex planning process

Which four of the following are the central tenets of organisational learning?
A.
Experimentation is the norm.
B.
Employees should be appointed on the basis of their previous educational success.
C.
Organisations are pluralistic.
D.
Managers facilitate rather than direct.
E.
Information flows and relationships between people are lateral as well as vertical.

The resource allocation process (RAP) explanation of strategy development says:


A.
that strategy develops as the outcome of the taken-for-granted assumptions and behaviours in
organisations.
B.
that strategy develops as the outcome of resource allocation routines in
organisations. C.
that strategy develops as the outcome of processes of bargaining a negotiation among powerful
internal or external interest groups (or stakeholders).
D.
that strategy develops as the outcome of logical incrementalism
dash–
the deliberate development of strategy by experimentation and learning from partial commitments.

What is the cultural explanation of strategy development?


A.
Strategy development must not be based on the taken-for-granted assumptions and behaviours in
organisations.
B.
Strategy development occurs as the outcome of the taken-for-granted assumptions and behaviours in
organisations.
C.
According to Hofstede's theories, strategies that work in one culture cannot be transferred to another
culture.
D.
A major task of strategy development is to alter the taken-for-granted assumptions and behaviours in
organisations.
What term is used for an organisation capable of continual regeneration from a variety of knowledge,
experience and skills of individuals within a culture that encourages mutual questioning and challenge
around a shared purpose or vision?
A.
Strategic organisation
B.
Learning organisation
C.
Pluralistic organisation
D.
Emergent organisation

A 'learning organisation' requires:


A.
a stable hierarchy.
B.
a shared vision and culture that is challenging and questioning.
C.
a questioning culture.
D.
knowledge management systems

Strategies most often develop in organisations:


A.
as the outcome of cultural and political processes in the organisation.
B.
through formal strategic planning processes.
C.
through multiple processes that vary according to the type of organisation and the context of that
organisation.
D.
through a process of learning by doing, often known as 'logical incremental'.

Middle managers are likely to see strategy development in terms of intended, rational, analytical
planned processes, whereas SEOs see strategy development more as a result of cultural and political
processes.
A.
False
B.
True

Organisations in complex situations are most likely to succeed if they use:


A.
strategies that have been successful in the past.
B.
different processes for different purposes.
C.
top-down strategic planning.
D.
different processes for different purposes and different strategy development roles at different
organisational levels.
Chapter 13 – Organising for Success

What term is used for the structures, processes and relationships through which an
organisation operates?
A.
The organisational design
B.
The organisation's culture
C.
The organisation's configuration
D.
The organisational paradigm

The speed of change and increased levels of uncertainty in the business environment have increased.
What is the most important result from this for organisations?
A.
They must become learning organisations.
B.
They must develop into international organisations.
C.
Their structures must become more rigid and stronger.
D.
They must have flexible designs and be skilled at reorganising.

Transnational corporations are required to simultaneously achieve local responsiveness and global
coordination. For this to work, global managers need to:
A.
cross national and functional boundaries while also being sensitive to local needs.
B.
ensure global innovation and learning.
C.
spot talent and foster innovation.
D.
All of the above.

Functional structures are based on:


A.
business units.
B.
geographic divisions.
C.
the primary activities of an organisation such as production, finance and marketing.
D.
product divisions

A 'holding company' structure can be defined as:


A.
one that is holding control over product or market divisions.
B.
one that is an investment company consisting of shareholdings in separate businesses.
C.
one that is managed by a group of managers who are shareholders.
D.
one that is holding control based on portfolio management principles.
Balanced scorecards are a means of control through:
A.
qualitative measures.
B.
performance targets.
C.
portfolio management.
D.
quantitative measures.

Market systems as control processes typically involve:


A.
a system for the allocation of resources.
B.
outsourcing of activities.
C.
using real market forces in the allocation of resources.
D.
a formalised system of contracting for resources.

Cultural systems of control are aiming at:


A.
standardisation of outputs.
B.
standardisation of norms/behaviours.
C.
standardisation of skills and behaviours.
D.
standardisation of processes.

Adopting a 'financial control' approach from the corporate centre involves:


A.
retaining financial control and strategic planning principles.
B.
complete devolution of both financial and strategic issues.
C.
complete devolution of strategic issues but retention of major financial controls.
D.
providing financial devolution but retaining strategic planning principles.

The 'strategic control' approach is characterised by:


A.
agreement between the centre and divisions within central guidelines.
B.
complete devolution of strategic and financial controls.
C.
the retention of strategic and financial controls in a top-down approach.
D.
complete devolution of strategic controls but retention of financial controls.
Which three of the following are typical dilemmas in organising for success?
A.
Holding the ring (tight–loose) versus holistic
solutions B.
Vertical accountability versus horizontal integration
C.
Empowering versus best practice
D.
Centralising and decentralising
E.
Hierarchies versus networks

A company is dominated by principles of hierarchy and vertical accountability, and is highly


antagonistic to radical innovation. Which approach would be most suitable if management wanted to
develop a new venture?
A.
Subdivide the organisation, setting up a new division
B.
Reorganise the whole organisation around the new venture
C.
Change the company culture

A dilemma for managers in the interconnectedness of configurations is which element drives the
others. The aim is to ensure that strategic elements drive structural elements.
A.
False
B.
True

Chapter 14 – Leading Strategic Change

There are different styles of managing strategic change. Which of the following are the potential
benefits of 'direction' as a change style?
A.
Clarity and speed
B.
Increasing ownership of a decision or process
C.
Maintaining control over the change process while also involving people in it
D.
Overcoming lack of information or misinformation

The style of leading change associated with ?persuasion' (or 'education') is best described as:
A.
Personal authority is used to set direction and the means of change
B.
A strategic leader retains coordination and authority but delegates elements of the change process
C.
Gaining support for change by generating understanding and commitment
D.
Involvement in setting the strategy agenda and resolving strategic issues by groups
Which of the following is not one of Balogun and Hope Hailey's important contextual features that
need to be taken into account when designing change programmes?
A.
Capability
B.
Readiness
C.
Capacity
D.
Technology
E.
Power

What type of change leadership involves 'incremental realignment'?


A.
Reconstruction
B.
Adaptation
C.
Revolution
D.
Evolution

Which of the following descriptions correctly apply to revolutionary change?


A.
It involves major strategic change.
B.
It occurs incrementally.
C.
It is rapid.
D.
It is likely to involve cultural change.

What type of change involves incremental transformation?


A.
Adaptation
B.
Evolution
C.
Revolution
D.
Reconstruction

Political processes in managing change may involve:


A.
senior managers.
B.
employees.
C.
all stakeholders.
D.
the Board of Directors.
Strategic change levers may be symbolic in nature. Which of the following do you think is the most
powerful symbol?
A.
Behaviour of managers
B.
Changes to systems such as reward, information and control systems
C.
Use of stories such as newsletters
D.
Physical changes in the work environment, such as relocations or changes to office space

Which of the following is least likely to be used as a mechanism for managing change from a
political perspective?
A.
Using symbols, rituals and language to legitimise change
B.
Building alliances and networks
C.
Associating with powerful stakeholder groups
D.
Looking for windows of opportunity
E.
Acquiring resources

Which of the following is least likely to be an important element in a turnaround strategy?


A.
Crisis stabilisation
B.
Focusing on long-term cost cutting
C.
Gaining stakeholder support
D.
Financial restructuring
E.
Clarifying the target market(s)

Chapter 15 – The Practice of Strategy

Porter's view is that the CEO has a key role as a strategic leader, setting a disciplined approach
to what fits and does not fit the overall strategy. Which of the following are dangers of this
approach? A.
Non-executive directors gain a greater say in strategic planning.
B.
CEOs can become over-confident and launch strategic initiatives of ever-increasing ambition.
C.
The CEO 'owns' the strategy and is responsible for its success.
D.
Failures are blamed on the CEO rather than being investigated.
Which of the following is claimed to be the main benefit of using the top management team to carry
out strategic planning activities?
A.
Top management teams are often made up of diverse individuals.
B.
They are often appointed by the CEO, so act independently.
C.
They bring additional experience and insight to the CEO.
D.
They can develop a cohesive and consistent approach to decision making.

Which of the following are good reasons why a strategic planner is often recruited from within
the organisation?
A.
Participating in strategy provides promising managers with an overview of the organisation as a
whole.
B.
They bring intuitive understanding, networks and credibility to the planning process.
C.
Participating in strategy provides promising managers with exposure to senior management.
D.
It introduces new ideas and perspectives on the organisation.

Which three of the following may increase the influence of middle managers on strategy making?
A.
When they have access to an organisation's 'strategic conversation'
B.
When they have access to organisational networks
C.
When they are deeply involved in operations
D.
When they have key organisational positions

Which of the following is not one of the four roles that consultants may play in strategy development
in organisations?
A.
Providing support for top management's views on the organisation's strategic issues
B.
Transferring knowledge
C.
Promoting strategic decisions
D.
Analysing, prioritising and generating options
E.
Implementing strategic change

What is the key finding of the McKinsey & Co. research on who should be included in
strategy making?
A.
Only the most senior managers should be involved.
B.
The people involved should be kept the same to ensure consistency.
C.
The people involved should vary according to the nature of the issue.
D.
Managers at many levels should always be involved.
What term is used for the process of winning the attention and support of top management and other
important stakeholders for strategic issues?
A.
Behavioural economics
B.
Issue packaging
C.
Strategic workshops
D.
Strategic issue selling

Senior managers should plan a strategy workshop with an open mind about what they hope to
achieve.
A.
False
B.
True

A company decides to assess whether operating from a single large site is essential to profitability. It
studies the possible outcomes of operating from a varying number of sites of different sizes. What
activity is it engaging in?
A.
Hypothesis testing
B.
Hypothetical prescription
C.
Hypothetical strategisation
D.
Strategisation

Which of the following criteria should a project team meet when making a business case for a new
product in a large organisation?
A.
Is it focused on strategic needs?
B.
Does it contain detailed procedures and policies?
C.
Does it plan actions for the whole organisation?
D.
Is it supported by key data?
E.
Does it demonstrate solutions and actions?
F.
Does it provide clear progress measures?
CHAPTER 1
The Nature of Strategic Management
True/False
Introduction
1. The underpinnings of strategic management hinge on managers gaining an understanding of competitors, markets,
prices, suppliers, distributors, governments, creditors, shareholders and customers worldwide.
Ans: T Page: 4

2. Although the Internet has increased in popularity, it has actually led to increases in company expenses.

Ans: F Page 4

3. Consumer e-commerce is five times greater than business-to-business e-commerce.

Ans: F Page 4

What Is Strategic Management?


4. Optimizing for tomorrow the trends of today is the purpose of strategic management.

Ans: F Page: 5
5. Even though useful, strategic planning has been cast aside by corporate America since the early 1990s.

Ans: F Page: 5

6. Resource allocation is included in strategy-formulation activities.


Ans: T Page: 5
7. The terms strategic management and strategy implementation are synonymous.
Ans: F Page: 5
8. A vision statement is, in essence, a company’s game plan.
Ans: F Page: 5
9. Strategy implementation is often considered to be the most difficult stage in the strategic-management
process because it requires personal discipline, commitment and sacrifice.
Ans: T Page: 6
10. The final stage in strategic management is strategy implementation.

Ans: F Page 6

11. Strategy formulation, implementation and evaluation activities occur at three hierarchical levels in a large diversified
organization: corporate, divisional and functional.
Ans: T Page: 6
12. One of the fundamental strategy evaluation activities is reviewing external and internal factors that are the bases
for current strategies.
Ans: T Page: 6
13. An objective, logical, systematic approach for making major decisions in an organization is a way to describe
the strategic-management process.
Ans: T Page: 7
14. Strategic management is an attempt to organize qualitative and quantitative information in a way that allows
effective decisions to be made under conditions of uncertainty.
Ans: T Page: 7
15. Analytical and intuitive thinking should complement each other.
Ans: T Page: 7

16. According to Albert Einstein, “Knowledge is far more important than intuition.”

Ans: F Page 7

17. Management by intuition can be defined as operating from the “I’ve-already-made-up-my-mind-don’t-bother- me-with-
the-facts mode.”

Ans: F Page 7

18. By monitoring external events, companies should be able to identify when change is
required. Ans: F Page: 8

Key Terms in Strategic Management


19. Anything the firm does especially well compared to rival firms could be considered a competitive advantage.
Ans: T Page 8

20. Once a firm acquires a competitive advantage, they are usually able to sustain the competitive advantage for an
extended period of time.

Ans: F Page 9

21. Newspaper companies in the United States provide a good example of how a company can sustain a
competitive advantage over the long term.

Ans: F Page 9

22. In order for a firm to achieve sustained competitive advantage, a firm must continually adapt to changes in
external trends and events and effectively formulate, implement, and evaluate strategies that capitalize
upon those factors.
Ans: T Page: 9
23. Strategists are usually found in higher levels of management and have considerable authority for decision-
making in the firm.
Ans: T Page: 10
24. The middle manager is the most visible and critical strategic manager.
Ans: F Page: 10
25. All strategists have similar attitudes, values, ethics and concerns for social responsibility.
Ans: F Page: 10
26. A vision statement answers the question, “What is our business?,” whereas a mission statement answers, “What
do we want to become?”
Ans: F Page: 10-11
27. In the last five years, the position of chief strategy officer (CSO) has diminished in comparison to other top
management ranks of many organizations.
Ans: F Page: 10
28. A clear mission statement describes the values and priorities of an
organization. Ans: T Page: 10
29. As of 2004, Wal-Mart was the largest corporation in the world.

Ans: T Page 11

30. Strengths and weaknesses are determined relative to competitors.


Ans: T Page: 12
31. In a multidivisional firm, objectives should be established for the overall company and not for each
division. Ans: F Page: 13
32. Objectives should be measurable, quantitative, challenging, realistic, consistent and prioritized.
Ans: T Page: 13
33. Annual objectives are long-term milestones that organizations must achieve to reach short-term objectives.
Ans: F Page: 13
34. Annual objectives are especially important in strategy formulation.
Ans: F Page: 13
35. According to research, a healthier workforce can more effectively and efficiently implement strategies.
Ans: T Page: 13
The Strategic-Management Model
36. Identifying an organization’s existing vision, mission, objectives and strategies is the final step for the strategic
management process.
Ans: F Page: 15
37. Once an effective strategy is designed, modifications are rarely
required. Ans: F Page: 15
38. Application of the strategic-management process is typically more formal in larger and well-established organizations.
Ans: T Page: 15
Benefits of Strategic Management
39. Followed by commitment, understanding is the most important benefit of strategic management.
Ans: T Page: 16
40. The changes that occurred at Disney after Robert Iger took over as CEO exemplifies the fact that more and
more organizations are centralizing the strategic-management process.
Ans: F Page: 16
41. Firms with planning systems more closely resembling strategic-management theory generally exhibit superior long-
term financial performance relative to their industry.
Ans: T Page: 17
42. Low-performing firms typically underestimate their competitor’s strengths and overestimate their own firm’s strengths.
Ans: T Page: 17
43. According to Greenley, strategic management provides a cooperative, integrated and enthusiastic approach to
tackling problems and opportunities.
Ans: T Page: 18
Why Some Firms Do No Strategic Planning

44. The poor reward structure is one reason managers do not engage in strategic planning.

Ans: T Page: 18

45. Crises and fires in an organization allows managers the training and time for effective strategic planning.

Ans: F Page: 17

Pitfalls in Doing Strategic Planning

46. Top managers making many intuitive decisions that conflict with the formal plan is one pitfall managers
should avoid in strategic planning.

Ans: T Page: 19

47. Managers must be very formal in strategic planning because formality induces flexibility and creativity.

Ans: F Page: 19

Guidelines for Effective Strategic Management

48. An integral part of strategy implementation must be to evaluate the quality of the strategic-management process.
Ans: F Page: 19
49. Strategic-management must be a self-reflective learning process that familiarizes managers and employees in
the organization with key strategic issues and feasible alternatives for resolving those issues.
Ans: T Page: 20
Business Ethics and Strategic Management
50. Today, managers and employees can be found personally liable if they ignore, conceal, or disregard a
pollution problem.

Ans: T Page: 21

51. Merely having a code of ethics is not sufficient to ensure ethical business
behavior. Ans: T Page: 23
52. An integral part of the responsibility of all managers is to provide ethical leadership by constant example
and demonstration.

Ans: T Page: 23

Comparing Business and Military Strategy


53. In most situations, business strategy is very different than military strategy.
Ans: F Page: 25
The Nature of Global Competition
54. International operations can be as simple as exporting a product to a single foreign
country. Ans: T Page: 28
55. One risk in international operations is that nationalistic factions could seize the operations.
Ans: T Page: 29
Conclusion
56. All organizations have a strategy from their inception, even if the strategy is informal, unstructured, and
sporadic. Ans: T Page: 30
57. Nonprofit organizations have less need for strategic management because they are not interested in making a profit.
Ans: F Page: 30
58. Firms can be more proactive with strategic management.
Ans: T Page: 30

Multiple Choice
Introduction

59. The term “environment” includes all of the following except:


a. air.
b. water.
c. firms.
d. natural resources.
e. fauna.
Ans: c Page: 4
60. The one factor that has most significantly impacted the nature and core of buying and selling in nearly all
industries has been
a. the Internet.
b. political borders.
c. corporate greed.
d. customer and employee focus.
e. the government.

Ans: a Page: 4

What Is Strategic Management?

61. What can be defined as the art and science of formulating, implementing and evaluating cross-functional
decisions that enable an organization to achieve its objectives?
a. Strategy formulation
b. Strategy evaluation
c. Strategy implementation
d. Strategic management
e. Strategic leading
Ans: d Page: 5
62. is used to refer to strategic formulation, implementation and evaluation, with
referring only to strategic formulation.
a. Strategic planning; strategic management
b. Strategic planning; strategic processing
c. Strategic management; strategic planning
d. Strategic management; strategic processing
e. Strategic implementation; strategic focus

Ans: c Page: 5
63. During what stage of strategic management are a firm’s specific internal strengths and weaknesses determined?
a. Formulation
b. Implementation
c. Evaluation
d. Feedback
e. Goal-setting

Ans: a Page: 5
64. An important activity in is taking corrective action.
a. strategy evaluation
b. strategy implementation
c. strategy formulation
d. strategy leadership
e. all of the above

Ans: a Page: 6

65. What step in the strategic development process involves mobilizing employees and managers to put strategies
into action?
a. Formulating strategy
b. Strategy evaluation
c. Implementing strategy
d. Strategic advantage
e. Competitive advantage

Ans: c Page: 6

66. What types of skills are especially critical for successful strategy implementation?
a. Interpersonal
b. Marketing
c. Technical
d. Conceptual
e. Thinking
Ans: a Page:
6

67. Which phase of strategic management is called the action phase?


a. Strategy formulation
b. Strategy implementation
c. Strategy evaluation
d. Allocating resources
e. Measuring performance
Ans: b Page: 6
68. is not a strategy-implementation activity.
a. Taking corrective actions
b. Establishing annual objectives
c. Devising policies
d. Allocating resources
e. Motivating employees
Ans: a Page: 6
69. Strategy evaluation is necessary because
a. internal and external factors are constantly changing.
b. the SEC requires strategy evaluation.
c. competitors change their strategies.
d. the IRS requires strategy evaluation.
e. firms have limited resources.
Ans: a Page: 6
70. Which statement best describes intuition?
a. It represents the marginal factor in decision-making.
b. It represents a minor factor in decision-making integrated with analysis.
c. It should be coupled with analysis in decision-making.
d. It is better than analysis in decision-making.
e. It is management by
ignorance. Ans: c Page: 7
71. and are external forces transforming business and society today.
a. E-commerce; strategy
b. E-commerce; globalization
c. Strategy; globalization
d. Corporate culture; stakeholders
e. Stakeholders; strategy

Ans: b Page: 8

72. Anything that a firm does especially well compared to rival firms is referred to as:
a. competitive advantage.
b. comparative advantage.
c. opportunity cost.
d. sustainable advantage.
e. an external opportunity.
Ans: a Page: 8

Key Terms in Strategic Management

73. The trends in newspaper circulation in the United States provide support for which statement?
a. Sustainable competitive advantage is easy to maintain.
b. Several firms can have similar competitive advantages.
c. Some products are relatively immune to changes in the external environment
d. Most competitive advantages are hard to sustain
e. Competition is generally good for companies and consumers
Ans: d Page 9
74. Which individuals are most responsible for the success and failure of an organization?
a. Strategists
b. Financial planners
c. Personnel directors
d. Stakeholders
e. Human resource managers
Ans: a Page: 10
75. The first step in strategic planning is generally:
a. Developing a vision statement
b. Establishing goals and objectives
c. Making a profit
d. Developing a mission statement
e. Determining opportunities and threats
Ans: a Page: 10
76. What are enduring statements of purpose that distinguish one business from other similar firms?
a. policies
b. mission statements
c. objectives
d. rules
e. employee conduct guidelines
Ans: b Page: 10
77. The largest company in the world is:
a. Honda Motor
b. ING Group
c. Wal-Mart
d. Ford Motor Company
e. Royal Dutch/Shell Group
Ans: d Page: 10
78. Usually, external opportunities and threats are:
a. uncontrollable by a single organization.
b. controlled by governments.
c. not as important as internal strengths and weaknesses.
d. key functions in strategy implementation.
e. key functions in strategy exploitation.

Ans: a Page: 12
79. Specific results an organization seeks to achieve in pursuing its basic mission are:
a. strategies
b. rules
c. objectives
d. policies
e. mission
Ans: c Page: 13
80. Internal are activities in an organization that are performed especially well.
a. opportunities
b. competencies
c. strengths
d. management
e. factors
Ans: c Page: 13

81. What are the means by which long-term objectives will be achieved?
a. strategies.
b. strengths.
c. weaknesses.
d. policies.
e. opportunities.

Ans: a Page: 13

82. Long-term objectives should be all of the following except:


a. measurable.
b. continually changing.
c. reasonable.
d. challenging.
e. consistent.
Ans: b Page:
13
83. can best be described as short-term in nature.
a. Mission statements
b. Tenure
c. Annual objectives
d. Strategies
e. Management

Ans: c Page: 13
84. In which phase of strategic management are annual objectives especially important?
a. formulation
b. control
c. evaluation
d. implementation
e. management
Ans: d Page: 13
85. What are guides to decision making?
a. laws
b. rules
c. policies
d. procedures
e. goals
Ans: c Page:
The Strategic-Management Model

86. The strategic-management process


a. occurs once a year.
b. is a sequential process.
c. is a continuous process.
d. applies mostly to companies with sales greater than $100 million.
e. applies mostly to small businesses
Ans: c Page: 13
87. Which of the following is not included in the strategic management model?
a. Measure and evaluate performance.
b. Perform internal research to identify customers.
c. Establish long-term objectives.
d. Implement strategies.
e. Develop mission and vision statements.
Ans: b Page: 14
Benefits of Strategic Management

88. Strategic management enables an organization to , instead of companies just responding to threats
in their business environment.
a. be proactive
b. determine when the threat will subside
c. avoid the threats
d. defeat their competitors
e. foresee into the future

Ans: a Page: 16
89. The act of strengthening employees’ sense of effectiveness by encouraging and rewarding them to participate in
decision-making and exercise initiative and imagination is referred to as:
a. Authoritarianism
b. Proaction
c. Empowerment
d. Transformation
e. Delegation
Ans: c Page: 16
90. How do line managers become “owners” of the strategy?
a. by attending top manager meetings
b. by gathering information about competitors
c. by involvement in the strategic-management process
d. by becoming a shareholder of the firm
e. by buying off top
managers Ans: c Page: 16
91. The changes that occurred when Robert Iger took over the reigns at Disney, demonstrate which current trend in
organizations?
a. increased formalization of the strategic management process
b. increased structuring of strategic management
c. increased decentralizing of strategic management
d. increased emphasis on strategic planning
e. increased central planning of the strategic management
process Ans: c Page 16
92. According to research, organizations using strategic management are than those that do not.
a. more profitable
b. more complex
c. less profitable
d. less static
e. less complex
Ans: a Page: 17
93. According to Greenley, strategic management offers all of these benefits except that
a. it provides an objective view of management problems.
b. it creates a framework for internal communication among personnel.
c. it encourages a favorable attitude toward change.
d. it maximizes the effects of adverse conditions and changes.
e. it gives a degree of discipline and formality to the management of a business.

Ans: d Page: 17- 18

Why Some Firms Do No Strategic Planning


94. What is not a reason given for poor or no strategic planning in organizations?
a. Wasting of time
b. Being content with success
c. Fire-fighting
d. Poor reward structure
e. Trust of management
Ans: e Page: 18-19

Pitfalls in Doing Strategic Planning

95. All of these are pitfalls an organization should avoid in strategic planning except:
a. using plans as a standard for measuring performance.
b. using strategic planning to gain control over decisions and resources.
c. failing to involve key employees in all phases of planning.
d. too hastily moving from mission development to strategy formulation.
e. being so formal in planning that flexibility and creativity are stifled.

Ans: a Page: 19
96. What is not a pitfall an organization should avoid in strategic planning?
a. Failing to communicate the plan to employees
b. Involving all managers rather than delegating planning to a “planner”
c. Top managers not actively supporting the strategic planning process
d. Doing strategic planning only to satisfy accreditation or regulatory requirements

Ans: b Page: 19
97. Which of the following statements is false?
a. Open-mindedness is an important guideline for effective strategic management.
b. Strategic management must become a self-perpetuating socialist mechanism.
c. No organization has unlimited resources.
d. Strategic decisions require trade-offs.
e. Strategic management must be a self-reflective learning process.

Ans: b Page: 20

Business Ethics and Strategic Management

98. Principles of conduct that guide decision-making are known as


a. human rights.
b. the Constitution.
c. business ethics.
d. nonprofit organization policies.
e. social responsibility requirements.
Ans: c Page: 20
99. A (n) can provide a basis on which policies can be devised to guide daily decisions and behavior
at the work site.
a. list of guidelines
b. policy for safety
c. vision statement
d. code of business ethics
e. annual objective

Ans: d Page: 23

100. Because they must take the of the firm, strategists’ salaries are high compared to those of other
individuals in the organization.
a. moral risks
b. social risks
c. environmental risks
d. societal criticism
e. employee criticism

Ans: a Page: 23

101. What can be created by ethics training and an ethics culture?


a. Competitive responsibility
b. Competitive advantage
c. Strategic advantage
d. Employee cooperation
e. Comparative advantage

Ans: c Page: 25
102. Which of these business actions is (are) always considered to be unethical?
a. poor product or service safety
b. using nonunion labor in a union shop
c. dumping flawed products in a foreign market
d. insider trading
e. all of the
above Ans: e
Page: 25
103. Ethical standards come out of in a final analysis.
a. government
b. competitors
c. history and heritage
d. stakeholder analysis
e. community involvement

Ans: c Page: 25
Comparing Business and Military Strategy
104. A strong heritage underlies the study of strategic management.
a. military
b. government
c. political
d. social
e. cultural
Ans: a Page: 25
105. Military strategy is based on an assumption of , whereas business strategy is based on an
assumption of .
a. conflict; cooperation
b. conflict; competition
c. cooperation; conflict
d. competition; conflict
e. cooperation; competition
Ans: b Page: 26
The Nature of Global Competition
106. are organizations that conduct business operations across national borders.
a. Domestic firms
b. Multinational corporations
c. Parent companies
d. Government-backed companies
e. Franchises

Ans: b Page: 28
107. A(n) refers to a firm investing in international operations, while the is the
country where that business is conducted.
a. parent company; host country
b. home country; parent company
c. parent country; host company
d. host company; home country
e. exporting company; importing company

Ans: a Page: 28
108. The greatest advantage of international operations is:
a. Reduced tariffs and taxes
b. Spreading economic risks over a wider number of markets
c. Access to global technology, culture and business practices
d. Gaining new customers
e. Less-intense competition
Ans: d Page: 28

109. All of these are potential disadvantages of an international operation except:


a. overestimated weaknesses and underestimated strengths of competitors.
b. differing languages, cultures and value systems.
c. reduced tariffs and taxes.
d. complexity due to a multiple monetary system.
e. all of these are potential disadvantages.

Ans: c Page: 29
The Nature of Strategic Management External opportunities; external threats
help an organization gather, analyze, and Internal weaknesses; external opportunities
The term is used to refer to strategy formulation, organize information. Internal strengths; internal weaknesses
implementation, and evaluation, with referring only to Ethics officers
strategy formulation. Operatives Strategic management can be defined as the art and science of
strategic planning; strategic management Lobbyists formulating, implementing, and evaluating cross-functional
assessment; planning Strategists decisions that enable an organization to achieve its objectives.
strategic management; strategic planning True
management cycle; brainstorming A disadvantage of international operations is:
f
o
r
e
i
g
n
o
p
e
r
a
ti
o
n
s
c
a
n
a
ll
o
w

fi
r
m
s
t
o
e
s
t
a
b
li
s
h
l
o
w
-
c
o
s
t
p
ro Three strategy activities are Long-range ter
S
du fundamental evaluation reviewing
plans Short- F m
cti
Strategy manipulation
on range plans S ob
differ among countries, external and internal factors,
Th je
measuring performance, and taking
e
acS cti
ca c
t
tio ve
n usi o
st s
ng r
ag wil
e co r
of l
m e
str be
at mu c
ac
eg nic t
ic hi
m ati i
ev
an on v
ag ed
bar e
e .
m rie
en 9
rs .
t is a
cal an A
c ll
le d
d t o
str pr f
i t
at obl
eg o h
y em e
n
s
Which of these s
facilities in f s e
requires a firm to ma a
locations close to o .
na r
establish annual e
raw materials r
objectives, gin p
and/or cheap m T it
devise policies, g
labor. economies u f
pe r
and allocate a
of scale can be l u ll
resources? opl
achieved from a e s
Strategy e. a
operation in t
formulation The rationale for periodically conducting strategic-management n
global rather i are especially critical for successful strategy o
Strategy
r
implementation than solely o meetings away r
imple g
domestic n from the work a
e
site is to ment n
markets. . encourage ation. i
competitors in more z
foreign markets Inter a
and
may not exist. F pers ti
among o
a onal
participants. n
l skills s
feedback; h
s Tech
rigidity o
e nical u
c l
plans
d
T an answer the question
av
r
oi organi "what do we want to
d zation' become?"
inO
str s False
at statem An organization can
eg pursue any and/or all
ic ents
strategies that
Re that
as
on Both military and business organizations do all of the following
s and
forpotentially could benefit the firm.
po except:
or cultural, False
or use of the demographic, No organization
no element of
str surprise. environmental, can pursue all the
at aim "to gain political, legal, strategies that
eg competitive
ic advantage." governmental, potentially could
pl technological, benefit the firm.
an use the
ni assumption of and competitive
ng trends and Communication is
on conflict to
pa develop events that could an act of
strengthening
rt significantly employees' sense of
of strategies. use
benefit or harm
fir their own
m an organization
s strengths to
in the future.
incexploit
lu Internal
de competitors' strengths;
weaknesses. external threats
Anything that a firm does especially well compared to rival firms
is planning except:
and laziness.
r ic planning to
u
e gain control
s
over
i
decisions and
n
resources.
g
failing to involve key
employees in all
phases of planning.
s
hastily moving from
t
mission
r development to
a strategy formulation.

t using plans as a
standard for
e measuring
g performance.
effectiveness by encouraging them to participate in decision 5. It is predicted that, by 2025, over 18% of the population in b. Implementation
making and to exercise initiative and imagination, and rewarding the United States will be over 65 years old. c. Evaluation
them for doing so.http://www.vuzs.net/ Ans: T Page 88 d. Feedback
False e. Goal-setting
Empowerment is the act of strengthening employees' sense of 6. The first step for using Porter’s Five-Forces Model is to effectiveness by encouraging and rewarding them to participate in
evaluate the relative strength of each competitive force.
Anything that a firm does especially well compared to rival
decision
A f
making and
exercise i
initiative and r
imagination.
m
s

i
s

r
e
f
e
r
r
e
d

t
o

a
s

a
.

c
o
m
p
e Strategic management must be a self-reflective learning
firm’s value chain formulating,
process. 7. Metaphors are handed-down narratives of some provide support for
t
wonderful activities are implementing and which statement?
i
True competitive compared evaluating cross- a. Sustainable
event that is c. opportunity cost
t competitive
to rivals. Ans: T Page functional decisions
based on history d. sustainable advantage is easy
i
One pitfall to avoid advantage 157 that to maintain.
but has been
v
in strategic e. an external b. Several firms can
embellished with
e opportunity have similar
planning is top 9. Since a combination
fictional detail. competitive
management strategy is not risky, advantages.
Ans: F Page 127
a making many many organizations c. Some products
d intuitive decisions pursue a combination are relatively
v that conflict with of two or more immune to changes
a formal planning. strategies in the external
True simultaneously. Ans: F
n environment
whether a 14. The trends in newspaper circulation in the
t United States Page: 171 d. Most competitive
a answers, “What advantages are
hard to sustain
g C do we want to 10. Most
S
become?” e. Competition is
e a h companies favor generally good for
a Ans: F Page: 10-11 related companies and
m
consumers
p p diversification
t 2. The changes that strategies in order
l
occurred at Disney to exploit common 15. In which phase
e e
after Robert Iger use of a well-known of strategic
r
took over as CEO brand name. Ans: T management are
s
C annual objectives
exemplifies the fact Page 180
o especially
that more and more
m 1 important?
organizations are
p a. Formulation
centralizing the Multiple Choice
r b. Control
– strategic- 11. What can be
e c. Evaluation
management defined as the art
h d. Implementation
process. and science of
5
e Ans: F Page: 16 e. Management
True/False 3. In multidivisional organizations, each division should develop a
n
enable an organization to achieve its objectives?
s
1. A vision mission McGinnis, to be organization aspired
i
statement statement effective, all a to be.
v
answers the independent of mission Ans: F Page: 63
e
question, “What the parent statement need
is our company. Ans: to do is define
E
business?,” F Page: 63 what the
x
whereas a organization is
a
mission 4. According to Vern and what the
m
statement
a. Strategy
16. Principles of
formulation
m conduct that
b. Strategy guide decision-
evaluation a making are
n known as
c.
a .
S
g a. human rights
t
e b. the Constitution
r
a m c. business
ethics
t e
d. nonprofit
e n organization
g t policies

y e. Strategic e. social
leading responsibility
requirements

i 12. During what


m stage of
p strategic
l management
e are a firm’s
m specific internal
e strengths and
n weaknesses
t determined? a.
a Formulation
t
i
o
n

d
.

S
t
r
a
t
e
g
i
c
17. Which of these basic questions should a vision statement e. Japan
answer? 22. Which of these examples of a mission statement’s focus
a. What is our business? area is not effective? 27. The term that refers to the EU’s effort to end competitive
b. Who are our employees? a. AT&T focuses on communication rather than telephones. tax breaks among member countries is .
c. Why do we exist? b. Exxon/Mobil focuses on oil and gas rather than energy. a. double taxation
d. What do we want to become? c. Union Pacific focuses on transportation rather than railroads. b. taxation bias
e. Who are our competitors?

d. Universal Studios focuses on entertainment rather than movies.


c. equity taxation
objectives?
e. Starbucks focuses
a. Developing vision on the café
18. What is
and mission experience rather
the first step statements than coffee.
in the b. Performing
comprehens external audits
23. Which type of
ive c. Performing
trend is exemplified
internal audits
strategic- by the increasing
d. Generating,
managemen evaluating, and numbers of two-
t model? selecting income households
strategies
a. Developing in America?
vision and e. Measuring and
evaluating a. Social
mission
statements performance b. Economic
b. Performing c. Cultural
external audits
d. Technological
c. Performing
e. Historical
internal audits
d. Measuring and
evaluating 24. In general,
performance what happens to
e. Establishing long- American goods
term objectives
in overseas
markets when
19. According to the
there is a strong
comprehensive
dollar?
strategic-
a. Less expensive
management
b. More attractive
model, which step
c. Cheaper
needs to be
d. More expensive
completed
e. Desirable
immediately
following the
establishment of
long-term
d i s e. Potential entry of of practical r h i
new competitors
. o u results are s e a
n m called l
29. A a. folktales
e
t d U
standardized,
28. According r b. rites
a . n e
detailed set of
to Porter, what s c.
x i .
techniques and
is usually the m
t
behaviors that r
most powerful e
d e
r manage i v
of the five t
. d
e anxieties, but t a
competitive a
c seldom produce u l
forces? p
o R intended, a S u
a. Potential h
n development of i technical l t e
o
substitute
c v consequences s a s
products
i a 20. What is the 2 t
b. Bargaining
best time to
l power of l 5 e
develop a o
suppliers
i r mission . s
r
c. statement?
a y
B a. Before a
t
business is I w
a e
i opened
a t i
r t
o
m l
g h
n
o l
a i n
n
i s i
e g
n h c
.
i a
p
n c v
r m
t g o e
e a
a m
d j
x p
p i h o
e
o c o r
t
h w t w i
i
a e e t
n
r r d i
g m
m e
a
o s
o t n
n f
f h y
i i
a b
z r
t y
c r
a m
o a
t s
n t c t
h specification
e 2 , job

0 analysis and

7 unity of
y
5 command?
e
a ?
r
b. When the a. None 30. What
firm is
b. One are
successful
c. Two historical
c. When the firm
is in financial d. Three narrative
trouble e. Five s
d. When the firm describin
is in legal
trouble 26. Who g the
is the unique
e. When the firm largest
encounters exporter accompli
competition to the shments
United
States? of a
21. A proactive
environment a. Mexico group
al policy is b. China and its
likely to lead
to . c. Canada leaders,

a. higher d. United usually


cleanup costs Kingdo in heroic
m
b. conservation terms?
of energy
a. Rites
c. reduced
b. Sagas
customer
loyalty c. Stories
d. numerous d. Myths
liability suits e. Folktales
e. higher medical
costs
31. Which
function
of
managem
ent
includes
areas
such as
job
design,
job
a. Planning e. Unrelated diversification b. Best-value
b. Organizing c. Low-cost focus
c. Motivating 36. Advanced Medical Optics using acquisitions to obtain all d. Best-value focus
d. Staffing medical aspects of eye care, from laser surgery to contacts to e. Differentiation
e. Controlling implants for all ages is an example of which type of strategy?
a. Forward integration 40. Which strategy would be most appropriate when the distinctive
32. Which of the following is the process of influencing people b. Backward integration competencies of two or more firms complement each other
to accomplish specific objectives? c. Horizontal integration especially well?
a. Staffing d. Market development a. Conglomerate diversification
b. Motivating e. Product development b. Divestiture
c. Planning c. Joint venture
d. Controlling
37. In which situation would horizontal integration be an
especially d. Retrenchment
e. Organizing
effective strategy? e. Integration
a. When an
33. Which decision
organization can
concerns
gain monopolistic 59.The term
determining the best
characteristics in “environment”
capital structure for
a particular area includes all of
the firm and includes
or region without the following
examining various
being challenged except: a.air.
methods by which
by the federal b.water.
the firm can raise
government for
capital?
“tending
a. Investment
substantially” to
reduce
competition.
b. When an
organization
competes in a
slowing industry.
b. Dividends
c. When decreased economies of scale provide major
competitive c.firms.
c. Financing maintain its
advantages.
d. Capital economic position in
d. When an
budgeting
the growth of the
organization has
e. Implementation economy and
neither the capital nor
human talent needed
34. What category of
to successfully
ratios measures how
manage an
effectively a firm can
expanded . s t
d e
organization. . o e
. n
b
e. When m .
n e
competitors are e t
a . c
succeeding due to r h
t f .
managerial e
u a c
expertise or having
r u o a
particular
a n r n g
resources an
l a p d o
organization
. o v
possesses.
Ans: c Page: 4 r e
r e
a r
e m
60. The one factor t n
s p
that has most e m
o l
significantly e
u o
impacted the nature n
r g y
and core of buying t
c r e
and selling in nearly .
e e e
all industries has Ans: a Page: 4
s e
been
d
industry? f
. What Is Strategic
eliminating 22,000 a.the Internet. o Management?
d
a. Profitability c 61. What can be
jobs in an b
. defined as the
b. Liquidity u
attempt to .
c art and
c. Leverage s
emerge from p science of
u
d. Activity . formulating,
bankruptcy o
s
e. Growth would be an l a. Forward integration
example of i strategy offers products or services implementing and
evaluating cross-functional decisions that
35. Budget Rent- a. divestiture t
b. B able on the
a-Car opening b. backward i to a
a
integration c. H market?
car rental shops c small
d. R a. Low-cost
in Wal-Mart c. liquidation rang
a
stores is an d. retrenchm l e of
ent
example of custo
e. forward
which type of mers
integration b
strategy? at
o
the
r
lowe
d
st
e
price
r
avail
c.Strategy implementation e.Competitive advantage a.It represents the marginal factor in decision-making.
d.Strategic management Ans: c Page: 6 b.It represents a minor factor in decision-making integrated
e. Strategic leading with analysis.
Ans: d Page: 5 66. What types of skills are especially critical for c.It should be coupled with analysis in decision-making.
successful strategy implementation? d.It is better than analysis in decision-making.
62. is used to refer to strategic formulation, a.Interpersonal e.It is management by ignorance.
implementation and evaluation, with referring b.Marketing Ans: c Page: 7
only to strategic formulation. c.Technical
a.Strategic planning; strategic management d.Conceptual 71. and are external forces transforming
b.Strategic planning; strategic processing e.Thinking business and society today.
c.Strategic management; strategic planning Ans: a Page: 6 a.E-commerce; strategy
d.Strategic management; strategic processing b.E-commerce; globalization
e.Strategic implementation; strategic focus 67. Which phase of strategic management is called the c.Strategy; globalization
Ans: c Page: 5 action phase? d.Corporate culture; stakeholders
a.Strategy formulation e.Stakeholders; strategy
63. During what stage of strategic management are a b.Strategy implementation Ans: b Page: 8
firm’s specific internal strengths and weaknesses c.Strategy evaluation
determined? a.Formulation d.Allocating resources 72. Anything that a firm does especially well compared to rival
b.Implementation e.Measuring performance firms is referred to as:
c.Evaluation Ans: b Page: 6 a.competitive advantage.
d.Feedback b.comparative advantage.
e.Goal-setting 68. is not a strategy-implementation c.opportunity cost.
Ans: a Page: 5 activity. a.Taking corrective actions d.sustainable advantage.
b.Establishing annual objectives e.an external opportunity.
64. An important activity in is taking corrective action.
c.Devising policies Ans: a Page: 8
a u g
. a y
s t
t i
i
r o
m
a n
p
t
l
e
b e
g
. m
y
s e
t n
e r t
v a a
a t t
l e i
o d 6 in sy to
d o strategies.
n e ab maintain
. y
r 65. What step in Key Terms in le .
A e
the strategic Strategic
s co b.Sever
c development l e Management
h process m al firms
. l s 73. The
i involves pe can
s o trends in
p titi have
t c newspaper
A
e ve similar
r a circulation in
n
. ad competit
a t the United
s
a va ive
t i States
:
l nt advanta
e n provide
l ag ges.
g g support for
a e c.Some products
y which are relatively
is
o statement? immune to
r
ea changes in the
f P a.Susta
f e external
s a
o mobilizing employees and environment
o g managers to put
r t
strategies into action?
u e
m h d.the IRS requires
r : strategy evaluation.
u e
l c a d.Most
e
a e . competitiv
6
a 7
t s e
b 0
i 69. advantage
o
o Strategy s are hard
v e
n evaluation to sustain
e .
is e.Competit
M
necessary ion is
d o
A generally
t because
.
n good for
i a.internal
s
s companies
v and
t
: and
a external
r
factors are consumers
a t
a constantly Ans: d
t i
changing. Page 9
e n
g b.the SEC
g P
requires 74. Which
y a individuals
strategy are most
g e
evaluation. responsible
l e m for the
c.competito success
e : p rs change and
l their
a
failure of an organization? e.key functions in strategy exploitation. 83. can best be described as short-term in nature.
a.Strategists Ans: a Page: 12 a.Mission statements
b.Financial planners b.Tenure
c.Personnel directors 79. Specific results an organization seeks to achieve in c.Annual objectives
d.Stakeholders pursuing its basic mission are: d.Strategies
e.Human resource managers a.strategies e.Management
Ans: a Page: 10 b.rules Ans: c Page: 13
c.objectives
75. The first step in strategic planning is generally: d.policies 84. In which phase of strategic management are annual objectives
a.Developing a vision statement e.mission especially important?
b.Establishing goals and objectives Ans: c Page: 13 a.formulation
c.Making a profit b.control
d.Developing a mission statement 80. Internal are activities in an organization that are c.evaluation
e.Determining opportunities and threats performed especially well. d.implementation
Ans: a Page: 10 a.opportunities e.management
b.competencies Ans: d Page: 13
76. What are enduring statements of purpose that distinguish one
c.strengths
b m s
u i b
s l .
i a m
n r i
e s
s s
f
s i
i
o
r
n
f m
r s
o ? s
m t
a
a
t
o .
e
t p
m
h o
e
e l
n
r i
t
c
s
i
s
e
i
c e . y : s . except:
d
. s I . a.measurable.
.
o N b
e 1 m e
b G .
A . 0 a b .
j c
n R n . o
e o
s G o 78. Usu a s p
c n
: r y ally, g t p
t t
o a external e r o
i i
u l opportu m e r
v b n
p nities e n t
e u
and n g u
s D a
P threats t t n
d.rules c u l
a are: h i
e . t l
g a.uncon s t
W c e
. e y
trollable . i
a h .
e : by a e
l / f
m
single s c
- S a c
p
1 organiz . h
M h c .
l
0 ation. a
a e t w
o
b.contro n
r l o e A
y
77. The lled by g
t l r a n
e largest govern i
compa d s k s
e
ny in ments. n
. Ans: c Page: 13 n :
the G c.not as g
world F e
c r important as .
is: o 81. What are the s
o o internal a
a r means by which s
n u strengths and
. d long-term e c
d p weaknesses.
H objectives will be s P .
u o d.key functions a
achieved? . r
c M in strategy
n A a g e
t o implementation
d n . e a
t . d
a s s : s
o .
g : t o
r p
u r 1 n
M o
i d a 3 a
o l
d C t b
t i
e o e l
o c 82. Long-term
m P
l g objectives e
r a i
i p i .
e should be all of
a g
n e the following
b e s
n
d l a s million. n
85.
. a l s e s
W
c w s . :
h
h s Ans: c Page: a
a a
13
a b p
t .
l . p c
o
l r l
c
a
e u The Strategic- i
c P
r Management
n l e
Model u a
e
g e s
86. r g
i s
T s e
n g
h m :
g u
c e o
. o
i
. s
n 1
d
p t
s c 3
e
e o l
t e
s
. l y
r 87. W
c i
a a hich
o t c
t t of
n o i
e o the
s e y
g follo
i s e
d i wing
s s
a
e c is
t m
d r
c - not
e a
. .
i m inclu
n l
p b.is a
s a ded
t sequential l
i r n process. in
.
o o a c.is a the
Ans: b continuous b
Page: 13 n c g strat
process. u
e e egic
d.appli s
d m man
m es
u i
e age
a mostly
r n
n men
k to
e e
t t
i compa
s s
mod
n nies s
el?
g p with e
? e r sales s
. o greater
g c than
a
o A
e $100
.
a.Measure and evaluate performance. organizations? Pitfalls in Doing Strategic Planning
b.Perform internal research to identify customers. a.increased formalization of the strategic management process 95. All of these are pitfalls an organization should avoid in
c.Establish long-term objectives. b.increased structuring of strategic management strategic planning except:
d.Implement strategies. c.increased decentralizing of strategic management a.using plans as a standard for measuring performance.
e.Develop mission and vision statements. d.increased emphasis on strategic planning b.using strategic planning to gain control over decisions and
Ans: b Page: 14

e.increased central planning of the strategic management process


resources.
h t
e h
Benefits of Strategic
Management e
88. Strategic
t
management
enables an h t
organization to r h
, e r
instead of a e
companies just t a
responding to t
threats in their s
w
business d
i
environment. .
l
a.be proactive d
l
b e
. f
d s e
e u a
t b t
e s
r i
t
m d
h
i e
e
n
i
e
c r
.
w a
c
h v
o
e o
m
n i
p
d
e
t t
i P s referred to as: a b.it creates a
Ans: c Page c.failing to
t a 16 p a.Authoritarian t framework for
involve key
o g r ism i internal
c employees in
r e 92. Acco o b o communication
o all phases of
s : rding to f . n among
m planning. d.too
research i P personnel.
p hastily moving
, t r c.it encourages a
e 1 l from mission e favorable attitude
organiza a o
. 6 e development to . toward change.
tions b a
f x strategy D d.it maximizes the
using l c effects of adverse
o formulation. e
strategic t conditions and
r e e.being so l changes.
manage A i
e formal in e e.it gives a degree of
ment n o
s planning that g discipline and
d s
are n
e flexibility and a formality to the
. :
that do
e creativity are t management of a
l
not. stifled. c i business.
e
a.more a Ans: a Page: 19
s . o Ans: d Page: 17- 18
i profitabl
s E n
n e
P 96. What is not a m
t b
a pitfall an p
o . s A
g organization should o
m t n
e avoid in strategic w
o a s
t planning? e
: :
h r t a.Failing to r
e e i communicate the
m
1 plan to employees
c c
e
7 b.Involving
f c n
all
u o e t P
93. According to managers
t m . Greenley, a
rather than
u p l strategic g
management delegating d
r l e offers all of e
planning to .
e e s :
a “planner” T
x 89. The act of strengthening c.Top r
employees’ sense of effectiveness
A managers not a 1
by these benefits except that
n c actively n 6
encouraging and rewarding them to
s . participate in decision-making a.it supporting s
provides an objective view of the strategic
: l f
management problems.
e planning o
s process r
a
s and exercise initiative and imagination is m
d.Doing regulator statements is c. b.by 94. What is not a reason given for
false? N gatheri poor or no strategic planning in
strategi y
o ng
a.Open- d.Strategic decisions require trade-
c requirem o informa
mindedness r tion offs.
plannin ents
is an g about
g only Ans: b a competi
Page: 19 important ni tors
to
guideline for z
satisfy c.by g orga
at
97. Which effective io involvemen e nizati
accredit
of the n 9
strategic t in the ons?
ation or following h
management. strategic- 1
a a.Wa
s manageme sting
become a self- u
90. How do nt process of
perpetuating nl
line i d.by time
managers s
m
become o becoming a b.Being
it
“owners” of c
e shareholder content
the i
d
strategy? a of the firm with
r
l
e e success
i
s
s . c.Fire-
o
t b
u fighting
rc y
m e d.Po
e s. or
c
h b rew
a u ard
n
y stru
i
s i ctur
m
n e
.
g e.Tr
a r Why
. Some ust
b Firms of
y Do No o
a Strategi man
tt f
c age
e Plannin f
n men
g
d t
i
n t Ans:
g o e
t
p Pag
o
p e:
m
18-
a m
n 19
a
a
g n
e a
e.St
rate 98.
gic Pri
nci
ma ple
s
nag of
co
em nd
uct
ent tha
t
mu gui
de
st de
cisi
be on-
ma
a kin
g
self are
kn
- ow
n
refl as

ecti
ve
lear
nin
g
pro
ces
s.
Ans: b Page:
20

Business
Ethics and
Strategic
Management
a.human rights. b.using nonunion labor in a union shop operations across national borders.
b.the Constitution. c.dumping flawed products in a foreign market a.Domestic firms
c.business ethics. d.insider trading b.Multinational corporations
d.nonprofit organization policies. e.all of the above c.Parent companies
e.social responsibility requirements. Ans: e Page: 25 d.Government-backed companies
Ans: c Page: 20 e.Franchises
1
0
3
.
E
t
h
i
c
a
l
s
t
a
n
d
a
r
d
s
c
o
m
e
o
u
t
o
f

i
n
a
fi
n
a
l
a
n
a
l
y
s
i
s
.
A
n
s
:
b
Page: 28 a m mpany;
r c
99. A (n) can provide e A
. pa home
a basis on which policies can f n C ny; country
a.government e o
r o ho
b a
1 s
e st
0 t .
7. o co
A( a un
n) fi
r try
m b.
ho
i
n m
v e
e
s co
ti un
n
try
g
i ;
n
pa
i
n re
t nt
e
r co
n m
a
ti pa
o ny
n
a c.p
l ar
en
t
co
un
try
;
ho
st
co
m
pa
ny
d.
ho
st
co
104.A strong m l an assumption Ans: a Page: 28 c.Access to m
a i of global p
heritage underlies the study of
strategic e.exporting n t 108.T technology, e
company; importing a i culture and t
he
100.B i e g c great business i
ecau s e a est practices t
se k m l advan d.Gaining i
c
they s e tage new o
r
must n of customers n
i d
take t intern e
b t .
the . ationa .
. i s A
s c l L
o n
of the
o i opera e
a c s
firm,
c s tions s
. i :
strate
i m is: s
m a
gists’
a a.Red -
i l
salari d
l uced i
A l
es
tariffs n
n i
are e and t P
r s t
high . taxes e a
i : a
comp c b.Spreadin n g
s r
ared u g economic s e
k y
to a l risks over a e :
those s t wider
of c.envir b u
onmen P number of
c 2
other tal . r
a markets
risks o 8
indivi g a
d. g 101.What can be created by ethics training and an ethics culture?
duals o l
societa e 109.All of these are potential disadvantages of an international
in the l v Ans: a
: Page: 25 a
organ criticis e a operation
m . except:
izatio r
e 105.Militar a.ov
n. 2 n
. y strategy eres
a 3 m
is based tima
. e
on an ted
m e n
assumptio wea
o m t
n of , kne
r p
whereas sse
a l
c s
l o business
. and
y strategy is
p und
e based on
r o eres
t d
.
i
potential disadvantages.
102.Which
Ans: c Page: 29
of these
The Nature of Global
business Competition
actions is 106. are
(are) organizations that
conduct business
always
considere
d to be
unethical?
a.poor
product or
service
safety
SAMPLE MCQs
SUBJECT: STRATEGIC MANAGEMENT

Sr. No. Question Content Module No.


1 Which one is not one of the elements of strategic management? 1
A) Formulation of Strategy
B) Co-Ordination of Strategy
C) Implementation of Strategy
D) Evaluation of Strategy
2 VUCA stands for: 2
A) Volatility, Uncertainty, Combination, and Ambiguity
B) Volatility, Unknown, Complexity and Ambiguity
C). Volatility, Uncertainty, Complexity and
Ambiguity
D). Variety, Uncertainty, Complexity and Ambiguity
3 Companies often make acquisitions abroad or enter into cross-border 2
alliances to
A) gain access to capabilities that complement their own or to learn
from their partners
B) to achieve lower costs through economies of scale
C) influence their strategy choices in international markets
D) expand into foreign markets
4 Which one Is not A VUCA component? 2
A) Ambiguity
B) Vulnerability
C) Complexity
D) Uncertainty
5 In SWOT Analysis, SW refers to the factors that are: 3
A) Internal
B) External
C) Global
D) Essential
6 Which of the following is not an example of a political risk? 3
A) Government Regulations
B) Cost of Production
C) War
D) Civil Unrest
7 What BCG Matrix Analyses? 4
A) Portfolio of companies
B) Sale of the products
C) Growth of companies
D) Revenue generated by companies
8 Market penetration strategy can be executed 5
A) Forming Strategic Partnership
B) Decreasing Prices to New Customer
C) Understanding Potential Synergies For Competition
D) When There Is Globally Standardized And Integrated Approach
9 Low-cost provider strategy becomes increasingly appealing and 6
competitively powerful when
A) the products of rival sellers are similar
B) most buyers use the product in the different ways
C) price competition among rival sellers is vigorous
D) there is less competition in the market
10 Core Competencies Fulfil Criteria as Below: 7
A) Provides Platform for Company’s Entire Product Range
B) Make A Significant Contribution In Sales Volumes
C)It Is Difficult to Imitate by Competitors
D) It Satisfies Consumer Needs
11 According to the , it can be concluded that 8
firms have a competitive advantage in a market if they offer products of a
higher perceived value to the customers and lower relative costs than
competing firms.
A) red ocean strategy
B) blue ocean strategy
C) analysis of national competitiveness
D) competitive triangle
12 The Costs of Manufacturing are significantly lower in some 9
locations
A) Geographic
B) Company
C) Local
D) International
13 Horizontal Integration Means 10
A) acquire a profitable business
b) acquire a loss-making business
c) acquire business in your own industry
d) acquire a business from foreign country
14 Strategy Evaluation Means 11
A) comparing costs
B) comparing time horizon
C) comparing human resource
D) company plans with actuals.
15 Regional Integration Is 12
A) multi-lateral trading
B) removing the barriers for group countries
C) managing exchange rate
D) anti-dumping tariff
lOMoARcPSD|6004607

Sm mcqs 1 - List of MCQ for Strategic


Management

MBA (Savitribai Phule Pune University)

Downloaded by Imad Quadri (imad3889@yahoo.co.in)


StuDocu is not sponsored or endorsed by any college or university

Downloaded by Imad Quadri (imad3889@yahoo.co.in)


lOMoARcPSD|6004607

MBA@IICMR

MBA-II, Semester- III

Course Code: 301

Course Name: Strategic Management

Multiple Choice Questions

1) Above-average returns are:

a. higher profits than the firm earned last year.


b. higher profits than the industry average over the last 10 years.
c. profits in excess of what an investor expects to earn from a historical
pattern of performance of the firm.
d. profits in excess of what an investor expects to earn from other
investments with a similar level of risk.

2) The strategic management process is

a. a set of activities that will assure a temporary advantage and average


returns for the firm.
b. a decision-making activity concerned with a firm's internal
resources, capabilities, and competencies, independent of the
conditions in
its externalenvironment.1
c. a process directed by top-management with input from other stakeholders
that seeks to achieve above-average returns for investors through effective use

Downloaded by Imad Quadri (imad3889@yahoo.co.in)


of the organization's resources.
d. the full set of commitments, decisions, and actions required for the firm
to achieve above-average returns and strategic competitiveness.

3) All of the following are assumptions of the industrial organization


(I/O) model EXCEPT

a. Organizational decision makers are rational and committed to acting in


the firm’s best interests.
b. Resources to implement strategies are firm-specific and attached to firms
over the long-term.
c. The external environment is assumed to impose pressures and constraints
that determine the strategies that result in above-average returns.
d. Firms in given industries, or given industry segments, are assumed to
control similar strategically relevant resources.

All of the following are assumptions of the resource-based model EXCEPT

a. Each firm is a unique collection of resources and capabilities.


b. The industry's structural characteristics have little impact on a
firm's performance over time.
c. Capabilities are highly mobile across firms.
d. Differences in resources and capabilities are the basis of competitive
advantage.

4) In the resource-based model, which of the following factors would be


considered a key to organizational success?

a. unique market niche


b. weak competition
c. economies of scale
d. skilled employees
5) All of the following are resources of an organization EXCEPT

a. an hourly production employee's ability to catch subtle quality defects


in products.
b. oil drilling rights in a promising region.
c. weak competitors in the industry.
d. a charity's endowment of $400 million..

6) The resource-based model of the firm argues that

a. all resources have the potential to be the basis of sustained


competitive advantage.
b. all capabilities can be a source of sustainable competitive advantage.
c. the key to competitive success is the structure of the industry in which
the firm competes.
d. resources and capabilities that are valuable, rare, costly to imitate,
and non-substitutable form the basis of a firm's core competencies.

7) The goal of the organization's is to capture the hearts and minds of


employees, challenge them, and evoke their emotions and dreams.

a. vision
b. mission
c. culture
d. strategy

8) A firm's mission
a. is a statement of a firm's business in which it intends to compete
and the customers which it intends to serve.
b. is an internally-focused affirmation of the organization's financial, social,
and ethical goals.
c. is mainly intended to emotionally inspire employees and other stakeholders.
d. is developed by a firm before the firm develops its vision.

9) An analysis of the economic segment of the external environment would


include all of the following EXCEPT

a. interest rates.
b. international trade.
c. the strength of the U.S. dollar.
d. the move toward a contingent workforce.

10) Product differentiation refers to the:

a. ability of the buyers of a product to negotiate a lower price.


b. response of incumbent firms to new entrants.
c. belief by customers that a product is unique.
d. fact that as more of a product is produced the cheaper it becomes per unit.

11) Which of the following is NOT an entry barrier to an industry?


a. expected competitor retaliation
b. economies of scale
c. customer product loyalty
d. bargaining power of suppliers
12) Switching costs refer to the:

a. cost to a producer to exchange equipment in a facility when


new technologies emerge.
b. cost of changing the firm's strategic group.
c. one-time costs suppliers incur when selling to a different customer.
d. one-time costs customers incur when buying from a different supplier.

13) New entrants to an industry are more likely when (i.e., entry barriers are low
when...)

a. it is difficult to gain access to distribution channels.


b. economies of scale in the industry are high.
c. product differentiation in the industry is low.
d. capital requirements in the industry are high.

14) Suppliers are powerful when:

a. satisfactory substitutes are available.


b. they sell a commodity product.
c. they offer a credible threat of forward integration.
d. they are in a highly fragmented industry.

15) The highest amount a firm can charge for its products is most
directly affected by

a. expected retaliation from competitors.


b. the cost of substitute products.
c. variable costs of production.
d. customers' high switching costs.
Answer: b. the cost of substitute products.
16) All of the following are forces that create high rivalry within an
industry EXCEPT
a. numerous or equally balanced competitors.
b. high fixed costs.
c. fast industry growth.
d. high storage costs.

17) According to the five factors model, an attractive industry would have all
of the following characteristics EXCEPT:

a. low barriers to entry.


b. suppliers with low bargaining power.
c. a moderate degree of rivalry among competitors.
d. few good product substitutes.

18) Internal analysis enables a firm to determine what the firm

a. can do.
b. should do.
c. will do.
d. might do.

19) An external analysis enables a firm to determine what the firm

a. can do.
b. should do.
c. will do.
d. might do.

20) is/are the source of a firm's , which is/are the source of the
firm's .
a. Resources, capabilities, core competencies
b. Capabilities, resources, core competencies
c. Capabilities, resources, above average returns
d. Core competencies, resources, competitive advantage

21) Compared to tangible resources, intangible resources are

a. of less strategic value to the firm.


b. not the focus of strategic analysis.
c. a more potent source of competitive advantage.
d. more likely to be reflected on the firm's balance sheet.

22) Which of the following is a true statement about capabilities?

a. Capabilities emerge over time through complex interactions of tangible


and intangible resources.
b. Valuable capabilities are based almost entirely on tangible resources.
c. Capabilities based on human capital are more vulnerable to
obsolescence than other intangible capabilities because of the tendency for
employee knowledge to become outdated.
d. The link between firm financial performance and capabilities is dependent
on whether the capabilities are based on tangible or intangible resources.

23) To be a core competency, a capability must satisfy all of the


following criteria EXCEPT:

a. be technologically innovative.
b. be hard for competing firms to duplicate.
c. be without good substitutes.
d. be valuable to customers.
24) Capabilities that other firms cannot develop easily are classified as

a. costly to imitate.
b. rare.
c. valuable.
d. nonsubstitutable.

25) Costly-to-imitate capabilities can emerge for all of the following reasons
EXCEPT

a. lack of scientific transference.


b. social complexity.
c. unique historical conditions.
d. causal ambiguity.

26) Gamma, Inc., has struggled for industry dominance with Ardent, Inc., its
main competitor, for years. Gamma has gathered and analyzed large
amounts of competitive intelligence about Ardent. It has observed as much
of the firm's internal functioning and technology as it can legally, yet
Gamma cannot understand why ABC has a competitive advantage over it.
The source of ABC's success is

a. impregnable.
b. causally ambiguous.
c. rationally obscure.
d. elusive.
27) Firms that achieve competitive parity can expect to:

a. earn below-average returns.


b. earn average returns.
c. earn above-average returns.
d. initially earn above-average returns, declining to average returns.

28) Business-level strategies detail commitments and actions taken to provide


value to customers and gain competitive advantage by exploiting core
competencies in

a. the selection of industries in which the firm will compete.


b. specific product markets.
c. primary value chain activities.
d. particular geographic locations.

29) The three dimensions of a firm's relationships with customers include all
the following EXCEPT

a. exclusiveness.
b. affiliation.
c. richness.
d. reach.

30) The effectiveness of any of the generic business-level strategies is


contingent upon

a. customer needs and competitors' strategies.


b. the match between the opportunities and threats in its external market
and the strengths and weaknesses of its internal environment.
c. the trends in the general consumer base and the robustness of the global
and industry economy.
d. the firm's competitive scope and its competitive advantage.
31) A cost leadership strategy targets the industry's customers.

a. most typical
b. poorest
c. least educated
d. most frugal

32) When the costs of supplies increase in an industry, the low-cost leader

a. may continue competing with rivals on the basis of product features.


b. will lose customers as a result of price increases.
c. will be unable to absorb higher costs because cost-leaders operate on very
narrow profit margins.
d. may be the only firm able to pay the higher prices and continue to
earn average or above- average returns.

33) The typical risks of a differentiation strategy do NOT include which of the
following?

a. Customers may find the price differential between the low-cost product and
the differentiated product too large.
b. Customers' experience with other products may narrow customers'
perception of the value of a product's differentiated features.
c. Counterfeit goods are widely available and acceptable to customers.
d. Suppliers of raw materials erode the firm's profit margin with price
increases.

34) Competitive rivalry has the most effect on the firm's


the firm's other strategies. strategies than
a. business-level
b. corporate-level
c. acquisition
d. international

35) Multimarket competition occurs when firms

a. sell different products to the same customer.


b. have a high level of awareness of their competitors' strategic intent.
c. simultaneously enter into an attack strategy.
d. compete against each other in several geographic or product markets.

36) Competitive dynamics refers to the

a. circumstances where competitors are aware of the degree of their mutual


interdependence resulting from market commonality and resource
similarity.
b. set of competitive actions and competitive responses the firm takes to
build or defend its competitive advantages and to improve its market
position.
c. total set of actions and responses taken by all firms competing within a
market.
d. ongoing set of competitive actions and competitive responses
between competitors as they maneuver for advantageous market
position.

37) Firms with few competitive resources are more likely

a. to not respond to competitive actions.


b. respond quickly to competitive actions.
c. delay responding to competitive actions.
d. respond to strategic actions, but not to tactical actions.

38) Which of the following would be an example of a strategic action?

a. a "two movies for the price of one" campaign by Blockbuster Video


b. use of product coupons by a local grocer
c. entry into the European market by Home Depot
d. fare increases by Southwest Airlines

39) The chief disadvantage of being a first mover is the

a. high degree of risk.


b. high level of competition in the new marketplace.
c. inability to earn above-average returns unless the production process is very
efficient.
d. difficulty of obtaining new customers.

.
40) Competitors are more likely to respond to competitive actions that are
taken by

a. differentiators.
b. larger companies.
c. first movers.
d. market leaders.
Answer: d. market leaders.

41) Ninety percent of Wm. Wrigley Company's total revenue comes from
chewing gum. This is an example of

a. market commonality.
b. standard-cycle markets.
c. economies of scale.
d. market dependence.

42) All competitive advantages do not accrue to large sized firms. A


major advantage of smaller firms is that they

a. are more likely to have organizational slack.


b. can launch competitive actions more quickly.
c. have more loyal and diverse workforces.
d. can wait for larger firms to make mistakes in introducing
innovative products.

43) A sustained or sustainable competitive advantage requires that:

a. the value creating strategy be in a formulation stage.


b. competitors be simultaneously implementing the strategy.
c. other companies not be able to duplicate the strategy.
d. average returns be earned by the company.

44) An integrated and coordinated set of commitments and actions designed to


exploit core competencies and gain a competitive advantage in a specific
product market is a definition of:

a. business strategy.
b. core competencies.
c. sustained competitive advantage.
d. strategic mission.

45) In evaluating its customers, which of the following is NOT a relevant


question?

a. How will core competencies meet the customer's needs?


b. Who is the customer?
c. What are the customers' needs?
d. How will our top management team interact with the customer?

46) Customer needs are related to the:

a. characteristics that can be used to subdivide a large market into segments.


b. set of values exhibited by a group of customers.
c. use of core competencies to implement a strategy.
d. benefits and features of a good or service that customers want
to purchase.

47) A company using a narrow scope in its business strategy is:

a. following a cost leadership business strategy.


b. focusing on a broad array of geographic markets.
c. limiting the group of product segments served.
d. likely to earn only average returns.

48) A cost leadership strategy provides goods or services with features


that are:

a. acceptable to customers.
b. unique to the customer.
c. highly valued by the customer.
d. able to meet unique needs of the customer

49) When the costs of supplies increase in an industry, the low-cost leader
may:

a. continue competing with rivals on the basis of product features.


b. lose customers as a result of price increases.
c. make it difficult for new entrants to the industry to achieve above-average
returns.
d. be the only firm able to pay the higher prices and continue to earn
average or above- average returns.

50) The risks of a cost leadership strategy include:

a. becoming "stuck in the middle."


b. production and distribution processes becoming obsolete
c. the ability of competing firms to provide similar features in a product.
d. customers deciding the product isn't worth what the firm must charge for it.

51) A firm successfully implementing a differentiation strategy would expect:

a. customers to be sensitive to price increases.


b. to charge premium prices.
c. customers to perceive the product as standard.
d. to automatically have high levels of power over suppliers.
Answer: b. to charge premium prices.

52) A differentiation strategy provides products that customers perceive


as having:

a. acceptable features.
b. features of little value relative to the value provided by the low-cost leader's
product.
c. features for which the customer will pay a low price.
d. features that are non-standardized for which they are willing to pay a
premium.

53) When implementing a focus strategy, the firm seeks:


a. to be the lowest cost producer in an industry.
b. to offer products with unique features for which customers will pay
a premium.
c. to avoid being stuck in the middle.
d. to serve the specialized needs of a market segment.
Answer: d. to serve the specialized needs of a market segment.
54) T or F? Firms operating in the same market, offering similar products
and targeting similar customers are competitors.

Answer: T

55) T or F? Intensified rivalry within an industry results in decreased average


profitability for the firms within it

Answer: T

56) T or F? Competitive dynamics indicates that firms and their strategic


actions are independent

Answer: F

57) T or F? Extensive market commonality guarantees intense competition


in an industry

Answer: F

58) T or F? Two firms that have similar resources, but do not share markets
would not be direct and mutually acknowledged competitors.

Answer: T

59) T or F? Wal-Mart has recently moved to Alsatia, Missouri. Several local


small retailers have decided that choosing not to respond to Wal-Mart's
competitive actions is a viable long-term option, because although the
companies have high market commonality they have little resource
similarity. These small retailers are correct in their decision

Answer: F

60) T or F? A competitive action is a strategic or tactical action taken by a


firm to gain or defend a competitive advantage.
Answer: T

61) T or F? First movers can gain a sustained competitive advantage


when they reduce their costs through reverse engineering

Answer: F

62) T or F? Product quality is a universal theme and is a necessary, but not a


sufficient, condition for competitive success.

Answer: T

63) T or F? The probability of a competitive response to a competitive action


is based partly on the reputation of the competitor

Answer: T

64) Which of the following is not one of the appeals of an


unrelated diversification strategy?

A. The ability to spread business risk over truly diverse businesses (as
compared to related diversification which is limited to spreading risk only
among businesses with strategic fit)
B. An ability to employ the company's financial resources to maximum
advantage by investing in whatever industries/businesses offer the best
profit prospects
C. Superior top management ability to cope with the wide variety of
problems encountered in managing a broadly diversified group of
businesses
65) Which of the following is not among the disadvantages and managerial
problems encountered by companies pursuing unrelated
diversification strategies?

A. Being without the added source of competitive advantage that cross-business


strategic fit provides
B. Spreading corporate resources too thinly over too many different lines
of business
C. The strain it places on corporate-level management in trying to stay on top of
fresh industry developments and the strategic progress and plans of each
business subsidiary
D. Ending up with too many cash hog businesses (as compared to
related diversification strategies where cash hog businesses are rare)

66) The two biggest drawbacks or disadvantages


of unrelated diversification are

A. The difficulties of passing the cost-of-entry test and the ease with which top
managers can make the mistake of diversifying into businesses where
competition is too intense
B. The difficulties of capturing financial fit and having insufficient financial
resources to spread business risk across many different lines of business
C. Demanding managerial requirements and being without the added
source of competitive advantage that cross-business strategic fit
provides

67) The two biggest drawbacks or disadvantages of


unrelated diversification are

A. Underemphasizing the importance of resource fit and the strong likelihood


of diversifying into businesses that top management does not know all that
much about
B. Insufficient cash flows to finance so many different lines of business and a
lack of uniformity among the strategies of the businesses it has diversified
into
C. Volatile sales and profits and making the mistake of diversifying into
too many cash cow businesses
D. The difficulties of competently managing many different businesses and
being without the added source of competitive advantage that cross-
business strategic fit provides

68) A fundamental weakness of unrelated diversification is

A. The tendency of corporate managers to place too much emphasis on


investing in cash cows rather than promising cash hogs
B. Reducing a company's access to economies of scope
C. Greater potential for there to be too much diversity among the
competitive strategies of the various business subsidiaries
D. The greater risk of getting trapped in tough struggles with strong
competitors
E. That the greater the number of businesses a company is in and the more
diverse they are, the harder it is for corporate managers to stay abreast of what's
happening in each industry and each subsidiary, know much about the
problems and issues each business

69) A fundamental weakness of unrelated diversification is

A. The tendency of corporate managers to place too much emphasis


on investing in cash cows rather than promising cash hogs
B. Reducing a company's access to economies of scope
C. Greater potential for there to be too much diversity among the
competitive strategies of the various business subsidiaries
D. The greater risk of getting trapped in tough struggles with strong competitors
E. That the greater the number of businesses a company is in and the more
diverse they are, the harder it is for corporate managers to stay abreast of
what's happening in each industry and each subsidiary, know much about
the problems and issues each business confronts and know what to do if a
business unit stumbles and its results suddenly head downhill

70) To identify a diversified company's strategy, one should consider


such factors as
A. The extent to which the firm is broadly or narrowly diversified, whether
it is pursuing related or unrelated diversification (or a mixture of both) and
the recent moves it has made to divest businesses acquire new
businesses and strengthen the positions of existing businesses
B. Whether the company is focusing on "milking its cash cows" or "feeding its
cash hogs."
C. The technological proficiencies, labor skill requirements and functional
area strategies characterizing each of the firm's businesses

71) When identifying a diversified company's present corporate strategy,


which of the following would not be something to look for?

A. Recent moves to build positions in new industries


B. The company's approach to allocating investment capital and
resources across its present businesses
C. Recent management actions to strengthen the company's positions in
existing businesses
D. Recent moves to divest weak or unattractive business units
E. Actions over the past few years to substitute global strategies for multi-
country strategies in one or more business units

72) Which of the following is not a major consideration in evaluating the


pluses and minuses of a diversified company's strategy?

A. Checking whether the company's resources fit the requirements of its


present business lineup
B. Scrutinizing each industry/business to determine where driving
forces are strongest/weakest and how many profitable strategic groups the
company has diversified into
C. Ranking the performance prospects of the various businesses from best
to worst and determining what the corporate parent's priorities should be in
allocating resources to its different businesses

73) Evaluating a diversified company's corporate strategy and critiquing


the pluses and minuses of its business lineup involves
A. A SWOT analysis of each industry in which the firm has a business interest
B. Applying the cost-of-entry test, the better-off test, the profitability test and
the shareholder value test to each business and industry represented in the
company's business portfolio
C. Evaluating the strategic fits and resource fits among the various sister
businesses and deciding what priority to give each of the
company's business units in allocating resources

74) Which one of the following is not an important aspect of evaluating


the merits of a diversified company's strategy?

A. Assessing the competitive strength of each business the company


has diversified into
B. Determining which business units are cash cows and which ones are cash
hogs and then evaluating how soon the company's cash hogs can be
transformed into cash cows
C. Evaluating the strategic fits and resource fits among the various
sister businesses

75) Which of the following is not generally something that ought to be


considered in evaluating the attractiveness of a diversified company's
business makeup?

A. Market size and projected growth rate, industry profitability and the
intensity of competition
B. Industry uncertainty and business risk
C. The frequency with which strategic alliances and collaborative
partnerships are used in each industry, the extent to which firms in the
industry utilize outsourcing and whether the industries a company has
diversified into have common key success factors

76) Assessments of the long-term attractiveness of each industry represented


in a diversified company's lineup of businesses should be based on

A. A complete value-chain analysis of each industry


B. Whether the industries have the same kinds of driving forces
C. How many companies in each industry are making money and how many
are losing money
D. Quantitative industry attractiveness scores derived from rating each
industry on several relevant attractiveness measures (weighted according
to their relative importance in determining overall attractiveness)

77) The chief purpose of calculating quantitative


industry attractiveness scores for each industry a company has diversified
into is to

A. Determine which industry is the biggest and fastest growing


B. Get in position to rank the industries from most competitive to
least competitive
C. Provide a basis for drawing analysis-based conclusions about the
attractiveness of the industries a company has diversified into, both
individually and as a group and further to provide an indication of which
industries offer the best and worst long-term prospects

78) A weighted industry attractiveness assessment is generally analytically


superior to an unweighted assessment because

A. A weighted ranking identifies which industries offer the best/worst long-term


profit prospects
B. An unweighted ranking doesn't discriminate between strong and weak
industry driving forces and industry competitive forces
C. It does a more accurate job of singling out which industry key success
factors are the most important
D. An unweighted ranking doesn't help identify which industries have
the easiest and hardest value chains to execute
E. The various measures of attractiveness are not likely to be
equally important in determining overall attractiveness

79) When industry attractiveness ratings are calculated for each of the
industries a multi-business company has diversified into, the results help
indicate

A. Which industries appear to be the best and worst ones to be in and the
attractiveness of all the industries as a group from the standpoint of the
company's long-term performance
B. Which industries have attractive key success factors and which
industries have unattractive key success factors
C. Which industries have the biggest economies of scale and which industries
have the greatest economies of scope and the overall potential for cost
reduction in the industries as a group

80) Calculating quantitative attractiveness ratings for the industries


a diversified company has invested in

A. Allows a company to rank the competitive advantage opportunities in


each industry from best to worst
B. Helps identify which industries have the best/worst prospects for
revenue growth
C. Identifies which industry has the best/worst value chain from the
standpoint of cost reduction potential
D. Provides a basis for deciding whether a diversified company has good
prospects for growth and profitability, given the attractiveness ratings
of the industries in which it has business interests
81) Assessments of how a diversified company's subsidiaries compare
in competitive strength should be based on such factors as

A. Vulnerability to seasonal and cyclical downturns, vulnerability to


driving forces and vulnerability to fluctuating interest rates and exchange
rates
B. Relative market share, ability to match or beat rivals on key product
attributes, brand image and reputation, costs relative to competitors and
ability to benefit from strategic fits with sister businesses
C. The appeal of its strategy, relative number of competitive capabilities, the
number of products in each businesses product line, which businesses have the
highest/lowest market shares and which businesses earn the highest/lowest
profits before taxes

82) The basic purpose of calculating competitive strength scores for each of a
diversified company's business units is to

A. Rank the business unit from best to worst in terms of potential for
cost reduction and profit margin improvement
B. Determine how strongly positioned each business unit is in its
industry and the extent to which it already is or can become a strong
market contender
C. Determine which business unit has the greatest number of resource strengths,
competencies and competitive capabilities and which one has the least

83) Using relative market share to assess a business's competitive strength


is analytically superior to straight percentage measures of market share
because relative market share
A. Is a better measure of a business's potential for increased sales
and profitability
B. Is a better indicator of competitive strength than is a simple percentage
measure of market share—for instance, a company with a 20% share is in
a much stronger competitive position if its largest rival has a share of 10%
(which means its relative market share is 2.0) than it is if its largest rival
has a 30% market share (in which case the company's relative market
share is only 0.67)

84) A weighted competitive strength analysis of a diversified company's


business units is conceptually stronger than an unweighted analysis
because

A. It provides a more accurate assessment of the strength of cross-business


strategic fits
B. It provides better indication of which business units have the best
strategy (vis-à-vis the rival in their respective industry)
C. The different measures of competitive strength are unlikely to be
equally important

85) The value of determining the relative competitive strength of each business
a company has diversified into is

A. To have a quantitative basis for identifying which businesses have


large/small competitive advantages or competitive disadvantages vis-à-vis the
rivals in their respective industries
B. To have a quantitative basis for rating them from strongest to weakest
in terms of contributing to the corporate parent's revenue growth
C. To compare resource strengths and weaknesses, business by business
D. To have a quantitative basis for rating them from strongest to weakest
in contending for market leadership in their respective industries

86) The nine-cell industry attractiveness-competitive strength matrix

A. Is useful for helping decide which businesses should have high, average
and low priorities in allocating corporate resources
B. Indicates which businesses are cash hogs and which are cash cows
C. Pinpoints what strategies are most appropriate for businesses positioned
in the three top cells of the matrix but is less clear about the best strategies
for
businesses positioned in the bottom six cells

87) The most important strategy-making guidance that comes from drawing a
9-cell industry attractiveness-competitive strength matrix is

A. Which businesses in the portfolio have the most potential for strategic fit and
resource fit
B. Why cash cow businesses are more valuable than cash hog businesses
C. That corporate resources should be concentrated on those businesses
enjoying both a higher degree of industry attractiveness and competitive
strength and that businesses having low competitive strength in
relatively unattractive industries should be looked at for possible
divestiture

88) One of the most significant contributions to strategy-making in diversified


companies that the 9-cell industry attractiveness/competitive strength
matrix provides is

A. Identifying which businesses have strategies that should be continued,


which business have strategies that need fine-tuning and which businesses have
strategies that need major overhaul
B. That businesses having the greatest competitive strength and positioned
in the most attractive industries should have the highest priority for
corporate resource allocation and that competitively weak businesses in
relatively unattractive industries should have the lowest priority and
perhaps even be considered for divestiture
C. Pinpointing what strategies are most appropriate for businesses positioned
in the four corners of the matrix (although the matrix reveals little about the
best strategies for businesses positioned in the remainder of the matrix)

89) In a diversified company, a business subsidiary has more competitive


advantage potential when
A. It is a cash cow
B. It has value chain relationships with other business subsidiaries that
present competitively valuable opportunities to transfer skills or
technology or intellectual capital from one business to another, combine the
performance of related activities and reduce costs, share use of a well-
respected brand name or collaborate to create new competitive capabilities
C. It is the company's biggest profit producer or is capable of becoming
the biggest

90) Checking a diversified company's business portfolio for the competitive


advantage potential of cross-business strategic fits does not involve
ascertaining
A. The extent to which sister business units have value chain match-ups that
offer opportunities to combine the performance of related value chain
activities and reduce costs
B. The extent to which sister business units have value chain match-ups that
offer opportunities to transfer skills or technology or intellectual capital
from one business to another
C. The extent to which sister business units have opportunities to share use of
a well-respected brand name
D. The extent to which sister business units have value chain match-ups that
offer opportunities to create new competitive capabilities or to leverage
existing resources
E. Which business units are cash cows and which ones are cash hogs

91) Checking a diversified firm's business portfolio for the competitive


advantage potential of cross-business strategic fits entails consideration of
A. Whether the parent's company's competitive advantages are being
deployed to maximum advantage in each of its business units
B. Whether the competitive strategies employed in each business act to
reinforce the competitive power of the strategies employed in the
company's other businesses
C. Whether the competitive strategies in each business possess good strategic
fit with the parent company's corporate strategy
D. The extent to which there are competitively valuable relationships
between the value chains of sister business units and what opportunities
they present to reduce costs, share use of a potent brand name, create
competitively valuable new capabilities via cross-business collaboration or
transfer skills or technology or intellectual capital from one business to
another

92) Which of the following is not a part of checking a diversified company's


business units for cross-business competitive advantage potential?

A. Ascertaining the extent to which sister business units have value chain
match-ups that offer opportunities to combine the performance of related
value chain activities and reduce costs
B. Ascertaining the extent to which sister business units have value chain
match-ups that offer opportunities to transfer skills or technology or
intellectual capital from one business to another
C. Ascertaining the extent to which sister business units are
making maximum use of the parent company's competitive
advantages

93) A diversified company's business units exhibit good resource fit when

A. Each business is a cash cow


B. A company has the resources to adequately support the requirements
of its businesses as a group without spreading itself too thin and when
individual businesses add to a company's overall strengths
C. Each business is sufficiently profitable to generate an attractive return
on invested capital

94) The businesses in a diversified company's lineup exhibit good resource


fit when

A. The resource requirements of each business exactly match the resources the
company has available
B. Individual businesses add to a company's resource strengths and when a
company has the resources to adequately support the requirements of its
businesses as a group without spreading itself too thin
C. Each business is generates just enough cash flow annually to fund its own
capital requirements and thus does not require cash infusions from the
corporate parent

95) A "cash cow" type of business

A. Generates unusually high profits and returns on equity investment


B. Is so profitable that it has no long-term debt
C. Generates positive cash flows over and above its internal requirements, thus
providing a corporate parent with cash flows that can be used for financing
new acquisitions, investing in cash hog businesses and/or paying dividends

96) A "cash hog" type of business

A. Is one that is losing money and requires cash infusions from its
corporate parent to continue operations
B. Is one that generates cash flows that are too small to fully fund its
operations and growth
C. Generates negative cash flows from internal operations and thus
requires cash infusions from its corporate parent to report a profit

97) The difference between a "cash-cow" business and a "cash hog" business
is that
A. A cash cow business is making money whereas a cash hog business is
losing money
B. A cash cow business generates enough profits to pay off long-term debt
whereas a cash hog business does not
C. A cash cow business generates positive retained earnings whereas a cash hog
business produces negative retained earnings
D. A cash cow business produces large internal cash flows over and above
what is needed to build and maintain the business whereas the internal
cash flows of a cash hog business are too small to fully fund its operating
needs and capital requirements

98) The tests of whether a diversified company's businesses exhibit


resource fit do not include

A. Whether the excess cash flows generated by cash cow businesses


are sufficient to cover the negative cash flows of its cash hog
businesses
B. Whether a business adequately contributes to achieving the corporate
parent's performance targets
C. Whether the company has adequate financial strength to fund its
different businesses and maintain a healthy credit rating
D. Whether the corporate parent has sufficient cash to fund the needs of its
individual businesses and pay dividends to shareholders without having to
borrow money

99) Which one of the following is not part of the task of checking a diversified
company's business line-up for adequate resource fit?

A. Determining whether the excess cash flows generated by cash cow


businesses are sufficient to cover the negative cash flows of its cash
hog businesses
B. Determining whether recently acquired businesses are acting to strengthen a
company's resource base and competitive capabilities or whether they are
causing its competitive and managerial resources to be stretched too thinly
across its businesses (sometimes newly-acquired businesses soak up a
disproportionate share of management's time and put a strain on other
company resources)
C. Determining whether some business units have value chain match-ups
that offer opportunities to transfer skills or technology or intellectual
capital from one business to another
100) Which one of the following is not a particularly relevant consideration in
deciding what the priorities should be for allocating resources to the
various businesses of a diversified company?

A. Whether and how corporate resources and capabilities can be used to


enhance the competitiveness of particular business units
B. What competitive strategy the business is presently using
C. Whether a business exhibits good strategic fit and resource fit with sister
businesses

101) Which one of the following is the best guideline for deciding what the
priorities should be for allocating resources to the various businesses of
a diversified company?

A. Businesses with high industry attractiveness ratings should be given top


priority and those with low industry attractiveness ratings should be given low
priority
B. Business subsidiaries with the brightest profit and growth prospects and
solid strategic and resource fits generally should head the list for corporate
resource support
C. The positions of each business in the nine-cell attractiveness-strength
matrix should govern resource allocation

102) Which one of the following is not a reasonable option for deploying
a diversified company's financial resources

A. Making acquisitions to establish positions in new businesses or to


complement existing businesses
B. Concentrating most of a company's financial resources in cash cow
businesses and allocating little or no additional resources to cash hog
businesses until they show enough strength to generate positive cash flows
C. Funding long-range R&D ventures aimed at opening market opportunities in
new or existing businesses

103) The strategic options to improve a diversified company's overall


performance do not include which of the following categories of actions?
A. Broadening the company's business scope by making new acquisitions
in new industries
B. Increasing dividend payments to shareholders and/or repurchasing
shares of the company's stock
C. Restructuring the company's business lineup and putting a whole new face
on the company's business makeup

104) Once a company has diversified into a collection of related or unrelated


businesses and concludes that some strategy adjustments are needed, which
one of the following is not one of the main strategy options that a company
can pursue?

A. Pursue multinational diversification


B. Restructure the company's business lineup
C. Craft new initiatives to build/enhance the reputation of the company's
brand name

105) Retrenching to a narrower diversification base

A. Is usually the most attractive long-run strategy for a broadly diversified


company confronted with recession, high interest rates, mounting
competitive pressures in several of its businesses and sluggish growth
B. Has the advantage of focusing a diversified firm's energies on building
strong positions in a few core businesses rather the stretching its
resources and managerial attention too thinly across many businesses
C. Is an attractive strategy option for revamping a diverse business lineup that
lacks strong cross-business financial fit

106) In which of the following instances is retrenching to a narrower


diversification base not likely to be an attractive or advisable strategy for a
diversified company?

A. When a diversified company has struggled to make certain


businesses attractively profitable
B. When a diversified company has too many cash cows
C. When one or more businesses are cash hogs with questionable long-term
potential

107) Strategies to restructure a diversified company's business lineup involves

A. Revamping the value chains of each of a diversified company's businesses


B. Focusing on restoring the profitability of its money-losing businesses and
thereby improving the company's overall profitability
C. Revamping the strategies of its different businesses, especially those that are
performing poorly
D. Divesting some businesses and acquiring new ones so as to put a new
face on a diversified company's business makeup

108) Corporate restructuring strategies

A. Involve making radical changes in diversified company's business


lineup, divesting some businesses and acquiring new ones so as to put a new
face on the company's business lineup
B. Entails reducing the scope of diversification to a smaller number
of businesses
C. Entail selling off marginal businesses to free up resources for redeployment
to the remaining businesses
109) What sets a multinational diversification strategy apart from
other diversification strategies is

A. The presence of extra degrees of strategic fit and more economies of scope
B. The potential to have a higher degree of technological expertise
C. A diversity of businesses and a diversity of national markets

110) The sources of a competitive advantage for a diversified


multinational corporation do not include

A. Transferring competitively valuable resources from one business to another


and one country to another
B. The ability to exploit opportunities for both cross-business and cross-country
collaboration and strategic coordination
C. Leveraging use of a well-known and competitively powerful brand name
D. Pursuing cross-business economy of scope opportunities and striving to fully
capture scale economies
E. Trying to maximize the number of cash cow businesses and minimize the
number of cash hog businesses

111) Which one of the following is not a way for a company to build
competitive advantage by pursuing a multinational diversification
strategy?

A. Fully capturing economies of scale and experience curve effects as well


as cross-business economies of scope
B. Using cross-business or cross-country market subsidization to outcompete
rivals
C. Fully capturing both cross-business financial fits and cross-country financial
fits
112) One of the biggest Internet-related strategic issues facing many
businesses is

A. Whether to have a company Web site


B. Whether and how to incorporate use of Internet technology applications in
performing various internal value chain activities
C. How best to try to offset the company's competitive disadvantage vis-à-vis
rivals that already sell direct to buyers at their Web site
D. Whether to form a strategic alliance with a pure dot-com enterprise
E. What role the company's Web site should play in the company's
competitive strategy

113) Which of the following is not one of the options that companies have for
using the Internet as a distribution channel to access buyers?

A. Establishing a company Web site so as to have an Internet presence


B. Operating a Web site that provides existing and potential customers with
extensive product information but that relies on click-throughs to distribution
channel partners to handle orders and sales transactions
C. Using online sales at the company's Web site as a relatively
minor distribution channel for achieving incremental sales

114) One very important advantage of a product-information-only Web site


strategy is

A. Lower advertising costs and lower customer service costs


B. Avoiding the extra costs associated with operating Web site e-stores
C. Added ability to interest potential buyers in purchasing the
company's products
D. Avoiding channel conflict
115) The advantages of a brick-and-click strategy include

A. Being able to attract bargain-hunting shoppers by selling the


company's merchandise online at lower prices than in traditional retail
stores
B. Being able to offer a much wider product line than is stocked at brick-and-
mortar stores
C. Low incremental investments to establish a Web site, the ability to
access a wider customer base and the ability to use existing distribution
centers and/or company store locations for picking orders from on-hand
inventories and making deliveries

116) Two big appeals of a brick-and-click strategy are

A. Lower advertising costs and enhanced ability to charge lower prices than
rivals
B. Economically expanding a company's geographic reach and giving
existing and potential customers another choice of how to communicate
with the company, shop for company products, make purchases or resolve
customer service problems
C. Low incremental investments to establish a Web site and the ability of
customers to use existing company store locations to view and inspect
items prior to purchase

117) A company that elects to use the Internet as its exclusive channel
for accessing buyers must address such strategic issues as

A. Whether it will have a broad or narrow product offering


B. How it will deliver unique value to buyers
C. How it will draw traffic to its Web site and then convert page views
into revenues
D. Whether it will perform order fulfillment activities internally or
outsource them
E. All of the above
118) Assuming a company elects to use the Internet as its exclusive channel for
accessing buyers, then which of the following is not one of the strategic
issues that it will need to address?

A. Whether to pursue a competitive advantage based on low-costs,


differentiation or more value for the money
B. How to deliver unique value to buyers
C. How to draw traffic to its Web site and then convert page views into
revenues
D. Whether to employ a forward integration strategy

119) In which of the following instances is being a first-mover not particularly


advantageous?
A. When a pioneer is using a low-cost provider strategy
B. When buyers are not loyal to pioneering firms in making repeat
purchases
C. When a pioneer is pursuing product innovation

120) Because when to make a strategic move can be just as important as what
move to make, a company's best option with respect to timing is

A. To be the first mover


B. To be a fast follower
C. To be a late mover (because it is cheaper and easier to imitate the
successful moves of the leaders and moving late allows a company to avoid
the mistakes and costs associated with trying to be a pioneer—first-mover
disadvantages usually overwhelm first-mover advantages)
D. To be the last-mover—playing catch-up is usually fairly easily and nearly
always much cheaper than any other option
E. To carefully weigh the first-mover advantages against the first-mover
disadvantages and act accordingly
121) When the race among rivals for industry leadership is a marathon rather
than a sprint,

A. It is best to be a fast follower rather than a first mover or a slow mover


B. Fast followers find it easy to leapfrog the pioneer with even better next-
generation products of their own
C. A slow mover may not be unduly penalized and first-mover advantages
can be fleeting

122) First-mover disadvantages arise when

A. The costs of pioneering are much higher than being a follower and only
negligible buyer loyalty or cost savings accrue to the pioneer
B. Technological change is rapid and following rivals find it easy to
leapfrog the pioneer with next-generation products of their own
C. The pioneer's skills, know-how and products are easily copied or even bested
by late movers
D. All of these

123) In which of the following cases are first-mover disadvantages not likely
to arise?

A. When the costs of pioneering are much higher than being a follower and
only negligible buyer loyalty or cost savings accrue to the pioneer
B. When new infrastructure is needed before market demand can surge
C. When the pioneer's skills, know-how and products are easily copied or even
bested by late movers
124) The task of crafting corporate strategy for a diversified
company encompasses

A. Picking the new industries to enter and deciding on the means of entry
B. Initiating actions to boost the combined performance of the businesses the
firm has entered
C. Pursuing opportunities to leverage cross-business value chain relationships
and strategic fits into competitive advantage
D. Establishing investment priorities and steering corporate resources into
the most attractive business units
E. All of these

125) Which one of the following is not one of the elements of crafting corporate
strategy for a diversified company?

A. Picking new industries to enter and deciding on the means of entry


B. Choosing the appropriate value chain for each business the company has
entered
C. Pursuing opportunities to leverage cross-business value chain relationships
and strategic fits into competitive advantage

126) Diversification merits strong consideration whenever a single-business


company

A. Has integrated backward and forward as far as it can


B. Is faced with diminishing market opportunities and stagnating sales
in its principal business
C. Has achieved industry leadership in its main line of business

127) Diversification ought to be considered when

A. A company's profits are being squeezed and it needs to increase its net
profit margins and return on investment
B. A company lacks sustainable competitive advantage in its present business
C. A company begins to encounter diminishing growth prospects in its
mainstay business

128) Diversification becomes a relevant strategic option in all but which one of
the following situations?

A. When a company spots opportunities to expand into industries


whose technologies and products complement its present business
B. When a company is only earning a low profit margin in its
principal business
C. When a company has a powerful and well-known brand name that can be
transferred to the products of other businesses and thereby used as a lever
for driving up the sales and profits of such businesses

129) Diversifying into new businesses is justifiable only if it

A. Results in increased profit margins and bigger total profits


B. Builds shareholder value
C. Helps acompany escape the rigors of competition in its present business

130) To create value for shareholders via diversification, a company must

A. Get into new businesses that are profitable


B. Diversify into industries that are growing rapidly
C. Spread its business risk across various industries by only acquiring firms that
are strong competitors in their respective industries
D. Diversify into businesses that can perform better under a single
corporate umbrella than they could perform operating as independent,
stand-alone businesses

131) The three tests for judging whether a particular diversification move
can create value for shareholders are

A. The attractiveness test, the profitability test and the shareholder value test
B. The strategic fit test, the competitive advantage test and the return
on investment test
C. The resource fit test, the profitability test and the shareholder value test
D. The attractiveness test, the cost-of-entry test and the better-off test

132) To test whether a particular diversification move has good prospects


for creating added shareholder value, corporate strategists should use

A. The profit test, the competitive strength test, the industry attractiveness
test and the capital gains test
B. The better-off test, the competitive advantage test, the profit expectations
test and the shareholder value test
C. The barrier to entry test, the competitive advantage test, the growth test
and the stock price effect test
D. The strategic fit test, the industry attractiveness test, the growth test, the
dividend effect test and the capital gains test
E. The attractiveness test, the cost of entry test and the better-off test

133) The attractiveness test for evaluating whether diversification into


a particular industry is likely to build shareholder value involves
determining whether
A. Conditions in the target industry are sufficiently attractive to permit
earning consistently good profits and returns on investment
B. The potential diversification move will boost the company's
competitive advantage in its existing business
C. Shareholders will viewed the contemplated diversification move as attractive

134) The cost-of-entry test for evaluating whether diversification into


a particular industry is likely to build shareholder value involves

A. Determining whether a newly entered business presents opportunities to


cost-efficiently transfer competitively valuable skills or technology from
one business to another
B. Determining whether the cost to enter the target industry will strain
the company's credit rating
C. Considering whether a company's costs to enter the target industry are
low enough to preserve attractive profitability or so high that the
potentials for good profitability and return on investment are eroded

135) The better-off test for evaluating whether a particular diversification


move is likely to generate added value for shareholders involves

A. Assessing whether the diversification move will make the company


better off because it will produce a greater number of core competencies
B. Assessing whether the diversification move will make the company better off
by improving its balance sheet strength and credit rating
C. Assessing whether the diversification move will make the company better off
by spreading shareholder risks across a greater number of businesses and
industries
D. Evaluating whether the diversification move will produce a 1 + 1 =3
outcome such that the company's different businesses perform better
together than apart and the whole ends up being greater than the sum
of the parts
136) Acquisition of an existing business is an attractive strategy option for
entering a promising new industry because it

A. Is an effective way to hurdle entry barriers, is usually quicker than


trying to launch a brand-new start-up operation and allows the acquirer to
move directly to the task of building a strong position in the target industry
B. Is less expensive than launching a new start-up operation, thus passing the
cost-of-entry test
C. Is a less risky way of passing the attractiveness test

137) Internal start-up of a new business subsidiary can be a more attractive


means of entering a desirable new business than is acquiring an existing
firm already in the targeted industry when

A. The costs associated with internal startup are less than the costs of
buying an existing company and the company has ample time and adequate
resources to launch the new internal start-up business from the ground up
B. There is a small pool of desirable acquisition candidates
C. The target industry is growing rapidly and no good joint venture partners are
available

138) The most popular strategy for entering new businesses and accomplishing
diversification is

A. Forming a joint venture with another company to enter the target industry
B. Internal startup
C. Acquisition of an existing business already in the chosen industry

139) A company can best accomplish diversification into new industries by


A. Outsourcing most of the value chain activities that have to be performed
in the target business/industry
B. Acquiring a company already operating in the target industry, creating
a new subsidiary internally to compete in the target industry or forming a
joint venture with another company to enter the target industry
C. Integrating forward or backward into the target industry

140) Which one of the following is not a factor that makes it appealing to
diversify into a new industry by forming an internal start-up subsidiary to
enter and compete in the target industry?

A. When internal entry is cheaper than entry via acquisition


B. When a company possesses the skills and resources to overcome entry
barriers and there is ample time to launch the business and compete
effectively
C. When adding new production capacity will not adversely impact the
supply demand balance in the industry by creating oversupply conditions
D. When the industry is growing rapidly and the target industry is
comprised of several relatively large and well-established firms

141) Diversifying into a new industry by forming a new internal subsidiary to


enter and compete in the target industry is attractive when

A. All of the potential acquisition candidates are losing money


B. It is impractical to outsource most of the value chain activities that have to
be performed in the target business/industry
C. There is ample time to launch the new business from the ground up and
entry barriers can be hurdled at acceptable cost

142) A joint venture is an attractive way for a company to enter a new


industry when

A. A firm is missing some essential skills or capabilities or resources and


needs a partner to supply the missing expertise and competencies or fill the
resource gaps
B. It needs access to economies of scope and good financial fits in order to
be cost-competitive
C. It is uneconomical for the firm to achieve economies of scope on its
own initiative

143) A joint venture is an attractive way for a company to enter a new industry
when

A. The pool of attractive acquisition candidates in the target industry


is relatively small
B. It needs better access to economies of scope in order to be cost-competitive
C. The industry is growing slowly and adding too much capacity too soon
could create oversupply conditions
D. The firm has no prior experience with diversification and the industry is
on the verge of explosive growth
E. The opportunity is too risky or complex for a company to pursue alone,
a company lacks some important resources or competencies and needs a
partner to supply them and/or a company needs a local partner in order to
enter a desirable business in a foreign country

144) The essential requirement for different businesses to be "related" is that

A. Their value chains possess competitively valuable cross-business


relationships
B. The products of the different businesses are bought by much the same
types of buyers
C. The products of the different businesses are sold in the same types of retail
stores

145) Businesses are said to be "related" when

A. They have several key suppliers and several key customers in common
B. Their value chains have the same number of primary activities
C. Their products are both sold through retailers
D. Their value chains possess competitively valuable cross-business
relationships that present opportunities to transfer resources from one
business to another, combine similar activities and reduce costs, share use
of a well-known brand name and/or create mutually useful resource
strengths and capabilities

146) Which of the following is the best example of related diversification?

A. An airline firm acquiring a rent-a-car company


B. A greeting card manufacturer deciding to open a chain of stores to retail
its lines of greeting cards
C. A manufacturer of ready-to-eat cereals acquiring a producer of cake
mixes and baking products

147) Which of the following is the best example of related diversification?

A. A beer brewer acquiring a maker of aluminum cans


B. A manufacturer of canoes diversifying into the production of tennis rackets
C. A PC producer deciding to diversify into producing and marketing its
own brands of MP3 players and LCD TVs

148) A big advantage of related diversification is that

A. It offers ways for a firm to realize 1 + 1 = 3 benefits because the value


chains of the different businesses present competitively valuable cross-
business relationships
B. It is less capital intensive and usually more profitable than
unrelated diversification
C. It involves diversifying into industries having the same kinds of key success
factors

149) Which of the following is not one of the appeals of related diversification?

A. It can offer opportunities for transferring expertise, technology and other


capabilities from one business to another
B. It can offer opportunities for reducing costs and for leveraging use of
a competitively powerful brand name
C. Related diversification is particularly well-suited for the use of offensive
strategies and capturing valuable financial fits

150) Which of the following is an important appeal of a related diversification


strategy?

A. Related diversification is an effective way of capturing valuable financial


fit benefits
B. Related diversification offers more competitive advantage potential than
does unrelated diversification
C. Related diversification offers significant opportunities to strongly
differentiate a company's product offerings from those of rivals

151) A company pursuing a related diversification strategy would likely


address the issue of what additional industries/businesses to diversify into
by

A. Locating businesses with well-known brand names and large market shares
B. Identifying industries with the least competitive intensity
C. Identifying an attractive industry whose value chain has good strategic
fit with one or more of the firm's present businesses
152) One strategic fit-based approach to related diversification would be to

A. Diversify into new industries that present opportunities to transfer


competitively valuable expertise, technological know-how or other
skills/capabilities from one sister business to another
B. Diversify into those industries where the same kinds of driving forces
and competitive forces prevail, thus allowing use of much the same
competitive strategy in all of the business a company is in
C. Acquire rival firms that have broader product lines so as to give the
company access to a wider range of buyer groups

153) The best place to look for cross-business strategic fits is

A. In R&D and technology activities


B. In supply chain activities
C. In sales and marketing activities
D. In production and distribution activities
E. Anywhere along the respective value chains of related businesses—no
one place is best

154) Cross-business strategic fits can be found

A. In unrelated as well as related businesses and in the markets of


foreign countries as well as in domestic markets
B. Only in businesses whose products/services satisfy the same general types
of buyer needs and preferences
C. Mainly in either technology related activities or sales and
marketing activities
D. Chiefly in the R&D portions of the value chains of unrelated businesses
E. Anywhere along the respective value chains of related businesses
155) Which of the following statements about cross-business strategic fit in a
diversified enterprise is not accurate?

A. Strategic fit between two businesses exists when the management know-how
accumulated in one business is transferable to the other
B. Strategic fit exists when two businesses present opportunities to economize
on marketing, selling and distribution costs
C. Competitively valuable cross-business strategic fits are what enable
related diversification to produce a 1 + 1 = 3 performance outcome
D. Strategic fit is primarily a byproduct of unrelated diversification and
exists when the value chain activities of unrelated businesses possess
economies of scope and good financial fit

156) Economies of scope

A. Are cost reductions that flow from operating in multiple businesses


B. Arise only from strategic fit relationships in the production portions of the
value chains of sister businesses
C. Are more associated with unrelated diversification than
related diversification

157) Economies of scope

A. Stem from the cost-saving efficiencies of operating over a wider geographic


area
B. Have to do with the cost-saving efficiencies of distributing a firm's product
through many different distribution channels simultaneously
C. Stem from cost-saving strategic fits along the value chains of related
businesses
158) Which of the following best illustrates an economy of scope?

A. Being able to eliminate or reduce costs by combining related value-chain


activities of different businesses into a single operation
B. Being able to eliminate or reduce costs by performing all of the value chain
activities of related sister businesses at the same location
C. Being able to eliminate or reduce costs by extending the firm's scope of
operations over a wider geographic area

159) What makes related diversification an attractive strategy is


A. The ability to broaden the company's product line
B. The opportunity to convert cross-business strategic fits into competitive
advantages over business rivals whose operations don't offer comparable
strategic fit benefits
C. The potential for improving the stability of the company's financial
performance

160) A diversified company that leverages the strategic fits of its


related businesses into competitive advantage

A. Has a distinctive competence in its related businesses


B. Has a clear path to achieving 1 + 1 = 3 gains in shareholder value
C. Has a clear path to global market leadership in the industries where it
has related businesses

161) A strategy of diversifying into unrelated businesses

A. Is aimed at achieving good financial fit (whereas related diversification aims


at good strategic fit)
B. Is the best way for a company to pass the attractiveness test in
choosing which types of businesses/industries to enter
C. Discounts the value and importance of strategic fit benefits and instead
focuses on building and managing a group of businesses capable of
delivering good financial performance irrespective of the industries these
businesses are in

162) Different businesses are said to be "unrelated" when

A. They are in different industries


B. The products of the different businesses are not bought by the same types
of buyers or sold in the same types of retail stores
C. The products of the different businesses satisfy different buyer needs
D. The businesses have different supply chains and different types of suppliers
E. There is an absence of competitively valuable strategic fits between
their respective value chains

163) The basic premise of unrelated diversification is that

A. The least risky way to diversify is to seek out businesses that are leaders
in their respective industry
B. The best companies to acquire are those that offer the greatest economies
of scope rather than the greatest economies of scale
C. The best way to build shareholder value is to acquire businesses with
strong cross-business financial fit
D. Any company that can be acquired on good financial terms and that has
satisfactory growth and earnings potential represents a good acquisition
and a good business opportunity

164) Which of the following is the best example of unrelated diversification?


A. A chain of radio stations acquiring TV stations
B. An electrical equipment manufacturer acquiring an athletic footwear
company
C. A producer of canned soups acquiring a maker of breakfast cereals

165) In companies pursuing a strategy of unrelated diversification,

A. The main basis for competitive advantage and improved shareholder value
is increased ability to achieve economies of scope
B. Each business is on its own in trying to build a competitive edge and the
consolidated performance of the businesses is likely to be no better than the
sum of what the individual businesses could achieve if they were
independent
C. There is a strong chance that the combined competitive advantages of the
various businesses will produce a 1 + 1 = 3 performance outcome as opposed to
just a 1 + 1 = 2 performance outcome

166) In diversified companies with unrelated businesses, the strategic attention


of top executives tends to be focused on

A. Screening acquisition candidates and evaluating the pros and cons or


keeping or divesting existing businesses
B. Identifying acquisition candidates that can pass the better-off test
C. Identifying opportunities to achieve greater economies of scope

167) Which of the following is not likely to command much strategic attention
from the top executives of companies pursuing an unrelated diversification
strategy?

A. Acquiring new businesses with attractive profit prospects


B. Whether existing businesses should be retained or divested based on
their ability to meet corporate targets for profit and returns on investment
C. Looking for new businesses that present good opportunities for
achieving economies of scope

168) Which of the following merits top priority attention by top executives of
companies pursuing an unrelated diversification strategy?

A. Acquiring new businesses that utilize much the same technology as


existing businesses
B. Whether to keep or divest businesses whose competitive strategies do
not match the overall competitive strategy of the corporation
C. Looking for new businesses having attractive distribution-related and
customer-related strategic fits with existing businesses
D. Identifying acquisition candidates that are financially distressed, can be
acquired at a bargain price and whose operations can, in management's
opinion, be turned around with the aid of the parent company's financial
resources and managerial know-how

169) With an unrelated diversification strategy, the types of companies that


make particularly attractive acquisition targets are

A. Financially distressed companies with good turnaround potential,


undervalued companies that can be acquired at a bargain price and
companies that have bright growth prospects but are short on investment
capital
B. Companies offering the biggest potential to reduce labor costs
C. Cash cow businesses with excellent financial fit

170) A key issue in companies pursuing an unrelated diversification strategy is

A. How wide a net to cast in building a portfolio of unrelated businesses


B. Whether to keep or divest businesses whose technological approaches do not
match the overall technology and R&D strategy of the corporation
C. How quickly to divest businesses whose competitive strategies do not
closely match the competitive strategies of sister businesses

171) One of the chief advantages of an unrelated diversification strategy is that


it

A. Expands a firm's competitive advantage opportunities to include a


wider array of businesses
B. Spreads the stockholders' risks across a group of truly diverse
businesses
C. Increases strategic fit opportunities and the potential for a 1 + 1 =3
outcome on the bottom line

172) What sets focused (or market niche) strategies apart from low-cost
leadership and broad differentiation strategies is

A. The extra attention paid to top-notch product performance and product


quality
B. Their concentrated attention on serving the needs of buyers in a
narrow piece of the overall market
C. Greater opportunity for competitive advantage

173) Companies pursuing a focused low-cost or focused differentiation strategy


strive to

A. Build a value-based competitive advantage keyed to product uniqueness


B. Develop the capability to simultaneously serve buyers in a variety of
distinct and different market segments
C. Do a better job of serving the needs and expectations of buyers in the
target market niche than other competitors in the industry
174) A focused low-cost strategy seeks to achieve competitive advantage by

A. Outmatching competitors in offering niche members an absolute rock-


bottom price
B. Delivering more value for the money than other competitors
C. Performing the primary value chain activities at a lower cost per unit
than can the industry's low-cost leaders
D. Dominating more market niches in the industry via a lower cost and a
lower price than any other rival
E. Serving buyers in the target market niche at a lower cost and lower
price than rivals

175) The chief difference between a low-cost leader strategy and a focused low-
cost strategy is

A. Whether the product is strongly differentiated or weakly differentiated from


rivals
B. The degree of bargaining power that buyers have
C. The size of the buyer group that a company is trying to appeal to

176) A focused differentiation strategy aims at securing competitive advantage

A. By providing niche members with a top-of-the-line product at a premium


price
B. By catering to buyers looking for an upscale product at an attractively
low price
C. With a product offering carefully designed to appeal to the unique
preferences and needs of a narrow, well-defined group of buyers
177) A focused low-cost strategy can lead to attractive competitive advantage
when

A. Buyers are looking for the best value at the best price
B. Buyers are looking for a budget-priced product
C. Buyers are price sensitive and are attracted to brands with low switching
costs
D. Demand in the target market niche is growing rapidly and a company can
achieve a big enough volume to fully capture all the available scale economies
E. A firm can lower costs significantly by limiting its customer base to a
well-defined buyer segment; its two options for achieving a low-cost
advantage are (1) out-managing rivals in controlling the factors that drive
costs and (2) reconfiguring its value chain in ways that deliver a cost edge
over rivals

178) The chief difference between a broad differentiation strategy and a


focused differentiation is

A. The size of the buyer group that a company is trying to appeal to


B. The degree of bargaining power that buyers have
C. Whether the product is strongly differentiated or weakly differentiated from
rivals

179) Which one of the following does not represent market circumstances that
make a focused low-cost or focused differentiation strategy attractive?

A. When it is costly or difficult for multi-segment competitors to put


capabilities in place to meet the specialized needs of the target market niche
and at the same time satisfy the expectations of their mainstream customers
B. When the industry has many different segments and market niches, thereby
allowing a focuser to pick an attractive niche suited to its resource strengths and
capabilities
C. When industry leaders do not see that having a presence in the niche
is crucial to their own success
D. When the target market niche is not overcrowded with a number of
other rivals attempting to focus on the same niche
E. When buyers are not strongly brand loyal and most industry
competitors are pursuing some sort of a focused strategy

180) The risks of a focused strategy based on either low-cost or differentiation


include

A. The chance that competitors outside the niche will find effective ways to
match the focuser's capabilities in serving the target niche
B. The potential for the preferences and needs of niche members to shift
over time towards many of the same product attributes and capabilities
desired by buyers in the mainstream portion of the market
C. The potential for the segment to become so attractive that it is soon
inundated with competitors, intensifying rivalry and splintering sales,
profits and growth prospects
D. The potential for segment growth to slow to such a small rate that a
focuser's prospects for future sales and profit gains become unacceptably dim
E. All of these

181) The production emphasis of a company pursuing a broad differentiation


strategy usually involves

A. A search for continuous cost reduction without sacrificing acceptable quality


and essential features
B. Strong efforts to be a leader in manufacturing process innovation
C. Efforts to build-in whatever differentiating features that buyers are
willing to pay for and striving for product superiority

182) The marketing emphasis of a company pursuing a broad


differentiation strategy usually is to
A. Underprice rival brands with comparable features
B. Tout differentiating features and charge a premium price that more
than covers the extra costs of differentiating features
C. Out-advertise rivals and make frequent use of discount coupons

183) The marketing emphasis of a company pursuing a focused low-cost


provider strategy usually is to

A. Tout the company's lower prices


B. Tout the lack of frills and extras
C. Out-advertise rivals and make frequent use of discount coupons
D. Communicate the attractive features of a budget-priced product
offering that fits niche members' expectations

184) One of the big dangers in crafting a competitive strategy is that managers,
torn between the pros and cons of the various generic strategies, will opt
for

A. A low-cost provider strategy because it is usually the safest, least risky


competitive strategy
B. A "stuck-in-the-middle" strategy
C. A broad differentiation strategy because it is frequently the most
profitable competitive strategy

185) Once a company has decided to employ a particular generic competitive


strategy, then it must make such additional strategic choices as

A. Whether to enter into strategic alliances or collaborative partnerships


B. Whether and when to employ offensive and defensive moves
C. What type of Web site strategy to employ
186) Which one of the following is not a strategic choice that a company
must make to complement and supplement its choice of one of the five
generic competitive strategies?

A. Whether to enter into strategic alliances or collaborative partnerships


B. Whether and when to employ offensive and defensive moves
C. Whether to employ a market share leadership strategy

187) Strategic alliances

A. Are the cheapest means of developing new technologies and getting new
products to market quickly
B. Are collaborative arrangements where two or more companies
join forces to achieve mutually beneficial strategic outcomes
C. Are a proven means of reducing the costs of performing value
chain activities

188) A strategic alliance

A. Is a collaborative arrangement where companies join forces to defeat


mutual competitive rivals
B. Involves two or more companies joining forces to pursue vertical integration
C. Is a formal agreement between two or more companies in which there is
strategically relevant collaboration of some sort, joint contribution of
resources, shared risk, shared control and mutual dependence

189) Entering into strategic alliances and collaborative partnerships can be


competitively valuable because

A. Working closely with outsiders is essential in developing new


technologies and new products in virtually every industry
B. Cooperative arrangements with other companies are very helpful in
racing against rivals to build a strong global presence and/or racing to seize
opportunities on the frontiers of advancing technology
C. They represent highly effective ways to achieve low-cost leadership and
capture first-mover advantages

190) The best strategic alliances

A. Are highly selective, focusing on particular value chain activities and on


obtaining a particular competitive benefit
B. Are those whose purpose is to create an industry key success factor
C. Are those which help a company move quickly from one strategic group
to another

191) Companies racing against rivals for global market leadership need
strategic alliances and collaborative partnerships with companies
in foreign countries in order to

A. Combat the bargaining power of foreign suppliers and help defend against
the competitive threat of substitute products produced by foreign rivals
B. Help raise needed financial capital from foreign banks and use the
brand names of their partners to make sales to foreign buyers
C. Get into critical country markets quickly and accelerate the process of
building a potent global presence, gain inside knowledge about unfamiliar
markets and cultures and access valuable skills and competencies that are
concentrated in particular geographic locations

192) A company racing to seize opportunities on the frontiers of advancing


technology often utilizes strategic alliances and collaborative partnerships
in order to

A. Discourage rival companies from merging with or acquiring the very


companies that it is partnering with
B. Reduce overall business risk and raise entry barriers into the newly
emerging industry
C. Help master new technologies and build new expertise and competencies
faster than would be possible through internal efforts, establish a stronger
beachhead for participating in the target industry and open up broader
opportunities in the target industry by melding their capabilities with the
resources and expertise of partners

193) Which of the following is not a typical reason that many alliances prove
unstable or break apart?

A. Diverging objectives and priorities


B. An inability to work well together
C. The emergence of more attractive technological paths that are better
pursued alone or with other partners
D. Disagreement over how to divide the profits gained from joint
collaboration

194) Experience indicates that strategic alliances

A. Are generally successful


B. Work well in cooperatively developing new technologies and new products
but seldom work well in promoting greater supply chain efficiency
C. Work best when they are aimed at achieving a mutually beneficial
competitive advantage for the allies
D. Have a high "divorce rate."

195) Which of the following is not a factor that makes an alliance "strategic"
as opposed to just a convenient business arrangement?

A. The alliance is critical to the company's achievement of an


important objective
B. The alliance helps block a competitive threat
C. The alliance helps open up important new market opportunities
D. The alliance helps build, enhance or sustain a core competence or
competitive advantage
E. The alliance helps the company obtain additional financing on better
credit terms

196) The Achilles heel (or biggest disadvantage/danger/pitfall) of


relying heavily on alliances and cooperative strategies is

A. That partners will not fully cooperate or share all they know, preferring
instead to guard their most valuable information and protect their more
valuable know-how
B. Becoming dependent on other companies for essential expertise and
capabilities
C. The added time and extra expenses associated with engaging in
collaborative efforts

197) Which of the following is not one of the factors that affects whether a
strategic alliance will be successful and realize its intended benefits?

A. Picking a good partner


B. Recognizing that the alliance must benefit both sides
C. Minimizing the amount of resources that the partners commit to the
alliance

198) Which one of the following is not a strategically beneficial reason why a
company may enter into strategic partnerships or cooperative
arrangements with key suppliers, distributors or makers of complementary
products?

A. To improve access to new markets


B. To expedite the development of promising new technologies or products
C. To enable greater vertical integration

199) The competitive attraction of entering into strategic alliances


and collaborative partnerships is

A. In allowing companies to bundle competencies and resources that are


more valuable in a joint effort than when kept separate
B. Speeding new products to market more quickly
C. Enabling greater vertical integration

200) The difference between a merger and an acquisition is that

A. A merger involves one company purchasing the assets of another


company with cash, whereas an acquisition involves a company acquiring
another company by buying all of the shares of its common stock
B. A merger is a pooling of equals whereas an acquisition involves
one company, the acquirer, purchasing and absorbing the operations
of another company, the acquired
C. In a merger the companies retain their original names whereas in an
acquisition the name of the company being acquired is changed to be the
name of the acquiring company

201) Which of the following is not a typical strategic objective or benefit that
drives mergers and acquisitions?

A. To gain quick access to new technologies or other resources and capabilities


B. To create a more cost-efficient operation out of the combined companies
C. To expand a company's geographic coverage
D. To facilitate a company's shift from a broad differentiation strategy to a
focused differentiation strategy

202) Mergers and acquisitions are often driven by such strategic objectives as
to

A. Expand a company's geographic coverage or extend its business into


new product categories
B. Reduce the number of industry key success factors
C. Reduce the number of strategic groups in the industry

203) Merger and acquisition strategies


A. Are nearly always a superior strategic alternative to forming alliances
or partnerships with these same companies
B. May offer considerable cost-saving opportunities (perhaps helping to
transform otherwise high-cost companies into a competitor with average or
below-average costs) and can also be beneficial in helping a company try to
invent a new industry and lead the convergence of industries whose
boundaries are being blurred by changing technologies and new market
opportunities
C. Are a particularly effective way of pursuing a blue ocean strategy
and outsourcing strategies

204) Mergers and acquisitions


A. Are nearly always successful in achieving their desired purpose
B. Frequently do not produce the hoped-for outcomes
C. Are generally less effective than forming alliances or partnerships with
these same companies

205) Vertical integration strategies


A. Extend a company's competitive scope within the same industry by
expanding its operations across more parts of the industry value chain
B. Are one of the best strategic options for helping companies win the race
for global market leadership
C. Offer good potential to expand a company's lineup of products and services

206) The two best reasons for investing company resources in


vertical integration (either forward or backward) are to

A. Expand into foreign markets and/or control more of the industry value chain
B. Broaden the firm's product line and/or avoid the need for outsourcing
C. Enable use of offensive strategies and/or gain a first mover advantage
over rivals in revamping the industry value chain
D. Strengthen the company's competitive position and/or boost its
profitability

207) For backward vertical integration into the business of suppliers to be


a viable and profitable strategy, a company

A. Must first be a proficient manufacturer


B. Must be able to achieve the same scale economies as outside suppliers
and match or beat suppliers' production efficiency with no drop-off in
quality
C. Must have excess production capacity, so that it has ample in-house ability to
undertake additional production activities

208) The strategic impetus for forward vertical integration is to

A. Gain better access to end users and better market visibility


B. Achieve the same scale economies as wholesale distributors and/or
retail dealers
C. Control price at the retail level

209) Which of the following is typically the strategic impetus for forward
vertical integration?

A. Being able to control the wholesale/retail portion of the industry value chain
B. Fewer disruptions in the delivery of the company's products to end-users
C. Gaining better access to end users and better market visibility

210) A good example of vertical integration is


A. A global public accounting firm acquiring a small local or regional
public accounting firm
B. A large supermarket chain getting into convenience food stores
C. A crude oil refiner purchasing a firm engaged in drilling and exploring
for oil

211) Which of the following is not a potential advantage of backward


vertical integration?

A. Reduced vulnerability to powerful suppliers (who may be inclined to


raise prices at every opportunity)
B. Reduced risks of disruptions in obtaining crucial components or support
services
C. Reduced costs
D. Reduced business risk because of controlling a bigger portion of the
overall industry value chain

212) Which of the following is not a strategic disadvantage of vertical


integration?

A. Vertical integration boosts a firm's capital investment in the industry,


thus increasing business risk if the industry becomes unattractive later
B. Vertical integration backward into parts and components manufacture can
impair a company's operating flexibility when it comes to changing out the use
of certain parts and components
C. Vertical integration reduces the opportunity for achieving greater
product differentiation

213) Outsourcing strategies

A. Are nearly always a more attractive strategic option than merger and
acquisition strategies
B. Carry the substantial risk of raising a company's costs
C. Carry the substantial risk of making a company overly dependent on
its suppliers
D. Increase a company's risk exposure to changing technology and/or changing
buyer preferences
E. Involve farming out value chain activities presently performed in-house
to outside specialists and strategic allies

214) Outsourcing the performance of value chain activities presently


performed in-house to outside vendors and suppliers makes strategic sense
when

A. An activity can be performed better or more cheaply by outside specialists


B. It allows a company to focus its entire energies on those activities that are at
the center of its expertise (its core competencies) and that are most critical to its
competitive and financial success
C. Outsourcing won't adversely hollow out the company's technical know-how,
competencies or capabilities
D. It reduces the company's risk exposure to changing technology
and/or changing buyer preferences
E. All of these

215) The two big drivers of outsourcing are

A. Increased ability to cut R&D expenses and increased ability to avoid the
problems of strategic alliances
B. A desire to take advantage of the fact that outsiders can perform certain
activities better or cheaper and allowing a company to focus its entire
energies on those activities that are at the center of its expertise (its core
competencies) and that are most critical to its competitive and financial
success
C. A desire to reduce the company's investment in fixed assets and the need
to narrow the scope of the company's in-house competencies and competitive
capabilities

216) Which of the following is not one of the benefits of outsourcing value
chain activities presently performed in-house?

A. Streamlining company operations in ways that improve organizational


flexibility and cut the time it takes to get new products into the
marketplace
B. Allowing a company to concentrate on its core business, leverage its
key resources and do even better what it already does best
C. Helping the company assemble diverse kinds of expertise speedily
and efficiently
D. Preventing a company from hollowing out its technical know-how,
competencies or capabilities

217) Relying on outsiders to perform certain value chain activities offers such
strategic advantages as

A. Obtaining higher quality and/or cheaper components or services


B. Improving the company's ability to innovate by allying with "best-in-world"
suppliers
C. Reducing the company's risk exposure to changing technology and/or
changing buyer preferences
D. Increasing the firm's ability to assemble diverse kinds of expertise speedily
and efficiently
E. All of the above
218) Outsourcing strategies can offer such advantages as

A. Increasing a company's ability to strongly differentiate its product and be


successful with either a broad differentiation strategy or a focused
differentiation strategy
B. Obtaining higher quality and/or cheaper components or services,
improving a company's ability to innovate and reducing its risk exposure
C. Speeding a company's entry into foreign markets

219) The big risk of employing an outsourcing strategy is

A. Causing the company to become partially integrated instead of being


fully integrated
B. Hollowing out a firm's own capabilities and losing touch with activities
and expertise that contribute fundamentally to the firm's competitiveness
and market success
C. Hurting a company's R&D capability

220) Which of the following is not one of the principal offensive strategy
options?

A. Leapfrogging competitors by being the first adopter of next-generation


technologies
B. Offering an equally good or better product at a lower price
C. Blocking the avenues open to challengers

221) Which one of the following is an example of an offensive strategy?

A. Blocking the avenues open to challengers


B. Signaling challengers that retaliation is likely
C. Pursuing continuous product innovation to draw sales and market share away
from less innovative rivals

222) A blue ocean type of offensive strategy

A. Is an offensive attack used by a market leader to steal customers away


from unsuspecting smaller rivals
B. Involves a preemptive strike to secure an advantageous position in a fast-
growing market segment
C. Works best when a company is the industry's low-cost leader
D. Involves abandoning efforts to beat out competitors in existing markets
and, instead, inventing a new industry or new market segment that renders
existing competitors largely irrelevant and allows a company to create and
capture altogether new demand

223) A hit-and-run or guerilla warfare type of offensive strategy involves

A. Random offensive attacks used by a market leader to steal customers


away from unsuspecting smaller rivals
B. Undertaking surprise moves to secure an advantageous position in a fast-
growing and profitable market segment; usually the guerilla signals rivals that
it will use deep price cuts to defend its newly-won position
C. Work best if the guerilla is the industry's low-cost leader
D. Pitting a small company's own competitive strengths head-on against the
strengths of much larger rivals
E. Random raids by a small competitor to grab sales and market share
from complacent or distracted rivals

224) Launching a preemptive strike type of offensive strategy entails

A. Cutting prices below a weak rival's costs


B. Moving first to secure an advantageous position that rivals
are prevented or discouraged from duplicating
C. Using hit-and-run tactics to grab sales and market share away
from complacent or distracted rivals

225) Which one of the following statements about offensive strategies is false?

A. It often takes the use of successful offensive strategies to build


to competitive advantage
B. One situation when a company needs to use offensive strategies is when it
has no choice but to try to whittle away at a strong rival's competitive
advantage
C. Offensive strategies have much to recommend when a company sees
an opening to gain profitable market share at the expense of rivals
D. One of the most potent types of offensive strategy is to introduce
new features or models to fill vacant niches in a company's overall
product offering and thereby better match the product offerings of key
rivals

226) Which one of the following is not a trait of a good strategic offensive?

A. Trying to build a more cost-efficient supply chain than rivals have


B. Being impatient with the status quo and displaying a strong bias for
swift, decisive actions to boost a company's competitive position vis-à-vis
rivals
C. Applying resources where rivals are least able to defend themselves

227) Which one of the following is not a good type of rival for an offensive-
minded company to target?

A. Market leaders that are vulnerable


B. Runner-up firms with weaknesses in areas where the challenger is strong
C. Small local and regional companies with limited capabilities
D. Other offensive-minded companies with a sizable war chest of cash and
marketable securities
228) Which one of the following statements regarding the basis for offensive
attack on rivals is false?

A. It is generally wise to use a company's resource strengths to attack rivals


in those competitive areas where they are strong
B. Ignoring the need to tie a strategic offensive to a company's strengths is
like going to war with a popgun
C. Strategic offensives should, as a general rule, be predicated on leveraging a
company's competitive assets—its core competencies, competitive
capabilities and other resource strengths
D. Offensive initiatives aimed at exploiting the competitive weaknesses of
rivals stand a better chance of success than do those that challenge a
competitor's strengths
E. Attacking a market leader is always unwise

229) The purposes of defensive strategies are to

A. Aggressively retaliate against rivals pursuing offensive strategies and


prevent against price wars
B. Lower the risk of being attacked by rivals, weaken the impact of any
attack that occurs and influence challengers to aim their offensive efforts at
other rivals
C. Guard against adverse changes in the company's macro-environment and
insulate the company from the impact of industry driving forces

230) Which one of the following is not a defensive option for protecting a
company's market share and competitive position?

A. Adding new features or models and otherwise broadening the product line to
close off vacant niches and gaps to opportunity-seeking challengers
B. Thwarting the efforts of rivals to attack with lower prices by maintaining
economy-priced options of its own
C. Running comparison ads that call attention to weaknesses in rivals'
products

231) Which of the following is a potential defensive move to ward off


challenger firms?

A. Granting volume discounts or better financing terms to


dealers/distributors and providing discount coupons to buyers to help
discourage them from experimenting with other suppliers/brands
B. Signaling challengers that retaliation is likely in the event they launch
an attack
C. Lengthening warranties, offering free or low-cost training and support
services and providing coupons and sample giveaways to buyers most prone to
experiment with using rival brands
D. Maintaining a war chest of cash and marketable securities
E. All of these

232) A company's competitive strategy deals with

A. Management's game plan for competing successfully—the specific


efforts to please customers, offensive and defensive moves to counter the
maneuvers of rivals, the reactions and responses to whatever market
conditions prevail at the moment and the initiatives undertaken to improve
the company's market position
B. What its strategy will be in such functional areas as R&D, production,
sales and marketing, distribution, finance and accounting and so on
C. Its efforts to change its position on the industry's strategic group map

233) The objective of competitive strategy is to

A. Contend successfully with the industry's 5 competitive forces


B. Knock the socks off rival companies by doing a better job of satisfying
buyer needs and preferences
C. Get the company into the best strategic group and then dominate it

234) A company achieves competitive advantage whenever

A. It is the acknowledged market share leader


B. It is the industry's acknowledged technology leader
C. It has greater financial resources than its rivals
D. It has a well-known and well-regarded brand name, prefers offensive
strategies to defensive strategies and has a strong balance sheet
E. It has some type of edge over rivals in attracting customers and
coping with competitive forces

235) A company can be said to have competitive advantage if

A. It is the acknowledged leader in product quality


B. It has a different value chain than rivals
C. It has some type of edge over rivals in attracting customers and coping
with competitive forces

236) While there are many routes to competitive advantage, they all involve

A. Building a brand name image that buyers trust


B. Delivering superior value to buyers and building competencies and
resource strengths in performing value chain activities that rivals cannot
readily match
C. Achieving lower costs than rivals and becoming the industry's sales and
market share leader

237) The biggest and most important differences among the competitive
strategies of different companies boil down to

A. How they go about building a brand name image that buyers trust
and whether they are a risk-taker or risk-avoider
B. The different ways that companies try to cope with the five
competitive forces
C. Whether a company's market target is broad or narrow and whether
the company is pursuing a competitive advantage linked to low cost or
differentiation

238) Which of the following is not one of the five generic types of competitive
strategy?

A. A low-cost provider strategy


B. A broad differentiation strategy
C. A best-cost provider strategy
D. A focused low-cost provider strategy
E. A market share dominator strategy

239) The generic types of competitive strategies include

A. Build market share, maintain market share and slowly surrender


market share
B. Offensive strategies and defensive strategies
C. Low-cost provider, broad differentiation, best-cost provider, focused
low-cost and focused differentiation

240) Which one of the following generic types of competitive strategy is


typically the best strategy for a company to employ?

A. A low-cost leadership strategy


B. A broad differentiation strategy
C. A best-cost provider strategy
D. A focused low-cost provider strategy
E. There is no such thing as a "best" competitive strategy; a company's
"best" strategy is always one that is customized to fit both industry and
competitive conditions and the company's own resources and competitive
capabilities

241) A low-cost leader's basis for competitive advantage is

A. Lower prices than rival firms


B. Using a low cost/low price approach to gain the biggest market share
C. High buyer switching costs
D. Meaningfully lower overall costs than competitors

242) How valuable a low-cost leader's cost advantage is depends on

A. Whether it is easy or inexpensive for rivals to copy the low-cost leader's


methods or otherwise match its low costs
B. How easy it is for the low-cost leader to gain the biggest market share
C. The aggressiveness with which the low-cost leader pursues converting the
cost advantage into the absolute lowest possible costs

243) A low-cost leader can translate its low-cost advantage over rivals into
superior profit performance by

A. Cutting its price to levels significantly below the prices of rivals


B. Either using its low-cost edge to underprice competitors and attract
price sensitive buyers in large enough numbers to increase total profits
or refraining from price-cutting and using the low-cost advantage to earn
a bigger profit margin on each unit sold
C. Going all out to use its cost advantage to capture a dominant share of
the market

244) The major avenues for achieving a cost advantage over rivals include

A. Revamping the firm's value chain to eliminate or bypass some cost-


producing activities and/or out-managing rivals in the efficiency with
which value chain activities are performed
B. Having a management team that is highly skilled in cutting costs
C. Being a first-mover in adopting the latest state-of-the-art technologies,
especially those relating to low-cost manufacture

245) A competitive strategy of striving to be the low-cost provider is


particularly attractive when

A. Buyers are not very brand-conscious


B. Most rivals are trying to be best-cost providers
C. There are many ways to achieve product differentiation that have value to
buyers
D. Buyers are large and have significant power to bargain down prices;
buyers use the product in much the same ways; and buyers have low
switching costs

246) Which of the following is not an action that a company can take to do
a better job than rivals of performing value chain activities more cost-
effectively?

A. Striving to capture all available economies of scale and learning/experience


curve effects
B. Trying to operate facilities at full capacity
C. Adopting labor-saving operating methods
D. Improving supply chain efficiency
E. Outsourcing all production-related activities

247) Which of the following is not one of the ways that a company can achieve a
cost advantage by revamping its value chain?

A. Cutting out distributors and dealers by selling direct to customers


B. Replacing certain value chain activities with faster
and cheaper online technology
C. Increasing production capacity and then striving hard to operate at full
capacity

248) To succeed with a low-cost provider strategy, company managers have to

A. Pursue backward or forward integration to detour suppliers or buyers with


considerable bargaining power and leverage
B. Move the performance of most all value chain activities to low-
wage countries
C. Sell direct to users of their product or service and eliminate use of
wholesale and retail intermediaries
D. Do two things: (1) do a better job than rivals of pursuing cost
savings throughout the value chain and (2) be proactive in revamping the
firm's overall value chain to eliminate low value-added activities and
bypass "nonessential" cost-producing activities

249) Achieving a cost advantage over rivals entails

A. Concentrating on the primary activities portion of the value chain


and outsourcing all support activities
B. Being a first-mover in pursuing backward and forward integration and
controlling as much of the industry value chain as possible
C. Out-managing rivals in performing value chain activities cost-effectively
and finding creative ways to cut cost-producing activities out of the value
chain

250) The best evidence that a company is the industry's low-cost provider is
that

A. It sells more of its product/service than its key competitors and is the
market share leader
B. It has lower overall per unit costs for its product/service than
other competitors in the industry
C. It has lower total operating costs on its income statement than do
its competitors

251) A company pursuing a low-cost leadership strategy must generally

A. Have products with good-to-excellent attributes so that its low prices


will provide customers with more value for the money
B. Have acceptable quality products that incorporate a good basic design
with few frills and offer a limited number of models/styles to select from
C. Have a wide selection of products that are of average or better quality
252) Being the overall low-cost provider in an industry has the attractive
advantage of

A. Building strong customer loyalty and locking customers into its


product (because customers have such high switching costs)
B. Giving the firm a very appealing brand image
C. Putting a firm in position to compete offensively on the basis of low
price, win the business of price sensitive customers, set the floor on
market price and defend against price war conditions should they arise

253) A competitive strategy to be the low-cost provider in an industry


works well when

A. Price competition among rival sellers is especially vigorous


B. There are few ways to achieve product differentiation that have value
to buyers
C. Buyers incur low costs in switching their purchases from one seller/brand
to another
D. Industry newcomers use low introductory prices to attract buyers and build a
customer base
E. All of these

254) A competitive strategy predicated on low-cost leadership tends to work


best when

A. There are widely varying needs and preferences among the various buyers
of the product or service
B. There are many market segments and market niches, such that it is feasible
for a low-cost leader to dominate the niche where buyers want a budget-priced
product
C. Price competition is especially vigorous and the offerings of rival firms
are essentially identical, standardized, commodity-like products

255) In which of the following circumstances is a strategy to be the


industry's overall low-cost provider not particularly well matched to the
market situation?

A. When the offerings of rival firms are essentially identical, standardized,


commodity-like products
B. When there are few ways to achieve differentiation that have value to buyers
C. When price competition is especially vigorous
D. When buyers have widely varying needs and special requirements and
the prices of substitute products are relatively high

256) A strategy to be the industry's overall low-cost provider tends to be more


appealing than a differentiation or best-cost or focus/market niche strategy
when

A. There are many ways to achieve product differentiation that buyers find
appealing
B. Buyers use the product in a variety of different ways and have high
switching costs in changing from one seller's product to another
C. The offerings of rival firms are essentially identical, standardized,
commodity-like products

257) In which of the following circumstances is a low-cost leadership strategy


not likely to be particularly successful?

A. When the industry's product is a standardized commodity


B. When buyers are looking for a good-to-excellent product at a bargain price
C. When the industry is composed of more than three strategic groups and
the companies in at least one of the groups are pursuing full vertical
integration strategies

258) Which of the following is not one of the pitfalls of a low-cost provider
strategy?

A. Overly aggressive price-cutting


B. Trying to set the industry's price ceiling
C. Not emphasizing avenues of cost advantage that can be kept proprietary
or that relegate rivals to playing catch up

259) The essence of a broad differentiation strategy is to


A. Appeal to the high end part of the market and concentrate on providing
a top-of-the-line product to consumers
B. Incorporate a greater number of differentiating features into
its product/service than rivals
C. Lower buyer switching costs
D. Outspend rivals on advertising and promotion in order to inform
and convince buyers of the value of its differentiating attributes
E. Be unique in ways that are valuable and appealing to a wide range
of buyers

260) A company attempting to be successful with a broad differentiation


strategy has to

A. Study buyer needs and behavior carefully to learn what buyers consider
important, what they think has value and what they are willing to pay for
B. Incorporate more differentiating features into its product/service than rivals
C. Concentrate its differentiating efforts on marketing and advertising
(where almost all differentiating features are created)

261) Successful differentiation allows a firm to


A. Be the industry's best-cost provider
B. Set the industry ceiling on price
C. Avoid being dragged into a price war with industry rivals and not be overly
concerned about whether entry barriers into the industry are high or low
D. Command a premium price for its product and/or increase unit sales
(because additional buyers are won over by the differentiating features),
and/or gain buyer loyalty to its brand (because some buyers prefer the
differentiating features and are thus brand loyal)

262) A company that succeeds in differentiating its product offering from those
of its rivals can usually

A. Avoid having to compete on the basis of simply a low price


B. Charge a price premium for its product (because buyers see its differentiating
features as worth something extra)
C. Increase unit sales (because of the attraction of its differentiating product
attributes)
D. Gain buyer loyalty to its brand (because some, maybe many, of its
customers will have a strong preference for the company's differentiating
features)
E. All of the above

263) A broad differentiation strategy improves profitability when

A. It is focused on product innovation


B. Differentiating enhances product performance
C. The differentiating features appeal to sophisticated and prestigious buyers
D. Unit sales increase and the extra price the product commands exceeds
the added costs of achieving the differentiation

264) Whether a broad differentiation strategy ends up enhancing company


profitability depends mainly on whether
A. Many buyers view the product's differentiating features as having value
B. Most buyers have similar needs and use the product in the same ways
C. Unit sales increase and the extra price the product commands exceeds
the added costs of achieving the differentiation

265) Using a broad differentiation strategy to produce an attractive


competitive advantage is least likely to be based on

A. Developing a superior performing product


B. Offering buyers a product which is superior in quality and reliability
as compared to rivals' brands
C. Giving consumers comprehensive support services
D. Providing buyers with a continuing stream of better-designed, better-
performing and more stylish products
E. Undercutting the prices being charged by rivals

266) Opportunities to differentiate a company's product offering


A. Are most reliably found in the R&D portion of the value chain
B. Are typically located in the sales and marketing portion of the value chain
C. Can exist in activities all along an industry's value chain

267) Easy-to-copy differentiating features

A. Cannot produce sustainable competitive advantage


B. Seldom are perceived by buyers as having much value
C. Tend to give buyers a high degree of power in bargaining for a lower price

268) The most appealing approaches to differentiation are

A. Those that are also being pursued by other rivals with


differentiation strategies
B. Those that are the most costly to incorporate (because expensive
attributes are perceived by buyers as more valuable and worth paying more
for)
C. Those that can be made even more attractive to buyers via clever advertising
D. Generally related to flavor and taste or sophisticated use of Internet
technology applications
E. Those that are hard or expensive for rivals to duplicate and that also
have considerable buyer appeal

269) Perceived value and signaling value are often an important part of a
successful differentiation strategy because

A. Of the diversity of buyer needs and preferences


B. Buyers seldom will pay for value they don't perceive, no matter how
real the value of the differentiating extras may be
C. Most buyers are heavily influenced by clever ads that signal value

270) A differentiation-based competitive advantage

A. Nearly always is attached to the quality and service aspects of a


company's product offering
B. Most usually is the result of highly effective marketing and advertising
C. Requires developing at least one distinctive competence that buyers
consider valuable
D. Hinges on a company's success in developing top-of-the-line product
features that will command the biggest price premium in the industry
E. Often hinges on incorporating features that (1) raise the performance of
the product or (2) lower the buyer's overall costs of using the company's
product or (3) enhance buyer satisfaction in intangible or non-economic
ways

271) Which of the following is not one of the four basic routes to achieving a
differentiation-based competitive advantage?
A. Delivering value to customers via competencies and competitive
capabilities that rivals don't have or can't afford to match
B. Incorporating features that raise product performance
C. Incorporating product attributes and user features that lower the
buyer's overall costs of using the company's product
D. Appealing to buyers who are sophisticated and shop hard for the
best, stand-out differentiating attributes

272) Achieving a differentiation-based competitive advantage can involve

A. Incorporating product attributes and user features that lower a buyer's


overall cost of using the product
B. Incorporating features that raise the performance a buyer gets from using
the product
C. Incorporating features that enhance buyer satisfaction in non-economic or
intangible ways
D. Delivering value to customers via competencies and competitive
capabilities that rivals don't have or can't afford to match
E. All of the above are viable ways of building competitive advantage via
differentiation

273) Broad differentiation strategies are well-suited for market circumstances


where

A. There are many ways to differentiate the product or service and


many buyers perceive these differences as having value
B. Most buyers have the same needs and use the product in the same ways
C. Buyers are susceptible to clever advertising

274) Broad differentiation strategies generally work best in market


circumstances where
A. Buyer needs and preferences are too diverse to be fully satisfied by a
standardized product
B. Most buyers have similar needs and use the product in the same ways
C. The products of rivals are weakly differentiated and most competitors
are resorting to clever advertising to try to set their product offerings apart

275) A broad differentiation strategy works best in situations where

A. Technological change is slow-paced and new or improved products are


infrequent
B. Buyer needs and uses of the product are very similar
C. Buyers incur low costs in switching their purchases to rival brands
D. Buyers have a low degree of bargaining power and purchase the product
frequently
E. Technological change is fast-paced and competition revolves around
rapidly evolving product features

276) A broad differentiation strategy generally produces the best results in


situations where

A. Buyer brand loyalty is low


B. Buyer needs and uses of the product are diverse
C. New and improved products are introduced only infrequently

277) In which one of the following market circumstances is a


broad differentiation strategy generally not well-suited?
A. When buyer needs and preferences are too diverse to be fully satisfied by
a standardized product
B. When few rivals are pursuing a similar differentiation approach
C. When the products of rivals are weakly differentiated and most
competitors are resorting to clever advertising to try to set their product
offerings apart

278) The pitfalls of a differentiation strategy include

A. Trying to differentiate on the basis of attributes or features that are easily


copied
B. Choosing to differentiate on the basis of attributes that buyers do
not perceive as valuable or worth paying for
C. Trying to charge too high a price premium for the differentiating features
D. Being timid and not striving to open up meaningful gaps in quality or
performance or service or other attractive differentiating attributes
E. All of these

279) Which of the following is not one of the pitfalls of pursuing a


differentiation strategy?

A. Trying to strongly differentiate the company's product from those of


rivals rather than be content with weak product differentiation
B. Over-differentiating so that the features and attributes incorporated
exceed buyer needs and requirements
C. Trying to charge too high a price premium for the differentiating features

280) Which one of the following statements about pursuing a broad


differentiation strategy is false?
A. Any differentiating feature that works well is a magnet for imitators
B. The best opportunities for achieving strong product differentiation are
in the production technology and marketing portions of the value chain
C. A low-cost provider strategy can defeat a broad differentiation strategy
when buyers are satisfied with a basic product and don't think "extra" attributes
are worth paying a higher price

281) A company achieves best-cost provider status by

A. Selling a product with the best cost at the best price


B. Having the best cost (as compared to rivals) for each activity in the
industry's value chain
C. Providing buyers with the best attributes at the best cost
D. Incorporating attractive or upscale attributes into its product offering at
a lower cost than rivals

282) A firm pursuing a best-cost provider strategy

A. Seeks to be the low-cost provider in the largest and fastest growing (or best)
market segment
B. Tries to have the best cost (as compared to rivals) for each activity in
the industry's value chain
C. Tries to outcompete a low-cost provider by attracting buyers on the basis
of charging the best price
D. Seeks to deliver superior value to buyers by satisfying their expectations
on key quality/service/features/performance attributes and beating their
expectations on price (given what rivals are charging for much the same
attributes)
283) Best-cost provider strategies

A. Aim at using the best operating practices to achieve lower costs and charge
lower prices than companies pursuing low-cost provider strategies
B. Involve charging a lower price for a product that has more upscale
attributes and features than the products offered by companies pursuing either
focused differentiation or broad differentiation strategies
C. Seek to attract buyers on the basis of charging the best price for a mid-
quality, average-performing product
D. Aim at giving customers more value for the money

284) The objective of a best-cost provider strategy is to

A. Deliver superior value to buyers by satisfying their expectations on


key quality/performance/features/service attributes and beating their
expectations on price (given what rivals are charging for much the same
attributes)
B. Offer buyers the industry's best-performing product at the best cost and
best (lowest) price in the industry
C. Attract buyers on the basis of having the industry's overall best-performing
product at a price that is slightly below the industry-average price

285) The competitive objective of a best-cost provider strategy is to

A. Outmatch the resource strengths of both low-cost providers and


differentiators
B. Position the company outside the competitive arena of low-cost producers
and differentiators
C. Meet or exceed buyer expectations on key
quality/performance/features/service attributes and beat their
expectations on price (given what rivals are charging for much the same
attributes)—
thereby achieving a value-based competitive advantage

286) For a best-cost provider strategy to be successful, a company must


have
287)
A. Excellent marketing and sales skills in convincing buyers to pay a
premium price for the attributes/features incorporated in its product
B. Resource strengths and competitive capabilities that allow it to
incorporate upscale attributes at lower costs than rivals whose
products have similar upscale attributes
C. Access to greater learning/experience curve effects and scale economies
than rivals
Answer:b

288) The competitive advantage of a best-cost provider is

A. Having the best value chain in the industry


B. Its brand name reputation
C. Its capability to incorporate upscale attributes at lower costs than rivals
whose products have similar upscale attributes

289) The target market of a best-cost provider is

A. Value-conscious buyers
B. Brand-conscious buyers
C. Price-sensitive buyers

290) Best-cost provider strategies are appealing in those market


situations where

A. Diverse buyer preferences make product differentiation the norm and


where many buyers are sensitive to both price and value
B. A company is positioned between competitors who have ultra-low prices and
competitors who have top-notch products in terms of both quality and
performance
C. Buyers are more quality-conscious than price-conscious

291) The big danger or risk of a best-cost provider strategy is

A. That buyers will be highly skeptical about paying a relatively low price for
upscale attributes/features
B. Not establishing strong alliances and partnerships with key suppliers
C. That low-cost leaders will be able to steal away some customers on the
basis of a lower price and high-end differentiators will be able to steal
away customers with the appeal of better product attributes

292) A company's biggest vulnerability in employing a best-cost provider


strategy is

A. Relying too heavily on outsourcing


B. Getting squeezed between the strategies of firms employing low-cost
provider strategies and high-end differentiation strategies
C. Getting trapped in a price war with low-cost leaders

293) Focused strategies keyed either to low-cost or differentiation are especially


appropriate for situations where

A. The market is composed of distinctly different buyer groups who have


different needs or use the product in different ways
B. Most other rival firms are using a best-cost producer strategy
C. Buyers have strong bargaining power and entry barriers are low
294) is based on the belief that the true measure of a really good
strategist is the ability to solve problems.

A.Managing by subjectives
B.Managing by
extrapolation C.Managing
by crisis D.Managing by
hope

295) Long-term objectives are needed at the of an organization.

A.corporate and divisional levels


B.corporate levels
C.corporate, divisional, and functional levels
D.divisional and functional levels

296) strategy is designed to fortify an organization's basic


distinctive competence.

A. Integrative
B. Retrenchment
C. Intensive
D.Diversification

297) Many, if not most, organizations pursue a:

A. concentric diversification.
B. combination strategy.
C. geographic strategy.
D.product development
strategy.
298) An example of strategy is establishing Web sites to sell products
directly to consumers.

A.backward integration
B.conglomerate diversification
C.horizontal integration
C. forward integration

299) Which of the following is not a guideline for pursuing liquidation?

A. When divestiture has not been successful.


B. When an organization's only alternative is bankruptcy.
C. When the stockholders of the firm can minimize their losses by selling
the firm's assets.
D. When the organization has successfully pursued retrenchment.

300) A popular strategy that occurs when two or more companies form a
temporary partnership for the purpose of capitalizing on some opportunity
is called a(n):

A. merger.
B. joint
venture.
C. acquisition.
D. takeover.

301) includes increasing the number of salespersons, increasing


advertising expenditures, offering extensive sales promotion items, or
increasing publicity efforts.

A.Market Development
B.Product development
C.Horizontal integration
D.Market penetration

302) is selling a division of an organization.

A.Franchising
B.Divestiture
C.Liquidation
D.Horizontal integration

303) When employing a strategy, a firm must be careful not to use such
aggressive price cuts that their own profits are low or nonexistent.

A.differentiation
B.focus
C.cost leadership
D.product development

304) Managing by Crisis is based on the belief that the true measure of a really
good strategist is the ability to solve problems.

True
False

305) To successfully employ a focus strategy, a firm must ensure that its total
costs across its overall value chain are lower than competitors' total
costs.

True
False
306) High-velocity change is becoming more the rule rather than the exception.
True
False

307) bankruptcy applies to farmers.


True
False

308) Joint ventures are being used increasingly because they allow companies
to minimize risk.

True
False

309) Diversification strategies are becoming more popular as organizations are


finding it easier to manage diverse business activities.

True
False

310) Pioneering new and better technologies is an action based on the


strategic posture of leading change.

True
False

311) Patrick majored in entrepreneurship and computer science in college.


After graduation, he decided to start his own business as an e-business
entrepreneur, founding an online B2B reverse auction company
called CompuSave.
a. After hiring several employees, Patrick requires that each person in this
company be involved in studying trends involving new technology,
competitors, and customers. These employees are involved in
.

i. external analysis
ii. internal environment
iii. strategic management
iv. strategic flexibility

312) Casey realizes that she has a personal characteristic that suggests she
is not comfortable interacting with strangers. She interprets this as a(n)
if she is to get a job as a salesperson.
i. A.Alternative
ii. B.opportunity
iii. C.strength
iv. D.weakness

313) Casey majored in marketing and really enjoyed studies in market


research. Through research on the Internet and in the university library, she
discovers that this industry appears to have significant positive external trends.
She interprets this as a(n) .
A. alternative
B.strength
C.weakness
D. opportunity

314) Casey has been involved in at a personal level.


A. opportunity analysis
B.risk avoidance
C.strategic planning
D. Stage decision making

315) You decide to concentrate on Taco Rocket’s primary business by only


increasing the menu to include new items such as enchiladas and rice
bowls. This is an example of what type of growth strategy?
A. lateral growth
B. horizontal integration
C. concentration
D. related diversification

316) Your oldest supplier, Zorro Distributors, is a family-owned firm.


Recently, the firm’s president, Diego De La Vega, made the decision to
retire. To his disappointment, none of his five children stepped forward to
take his place at the helm of the firm. Sr.De La Vega is concerned that if he
sells his company to a larger distributor, many of his employees will lose
their jobs. You approach your old friend with a generous offer to buy
Zorro and continue its current operations. Should your offer be accepted,
Taco Rocket would be undertaking .

A. lateral growth
B. unrelated diversification
C. forward vertical integration
D. backward vertical integration

317) You’ve decided to purchase a controlling interest in a chain of Oriental


fast-food restaurants, called Honk Kong Fooey. However, you have decided
to change the name of the chain to the Shanghai Grill. This move is most
representative of what type of growth strategy?

A. lateral growth
B. horizontal integration
C. unrelated diversification
D. related diversification

318) You decide to purchase a local five-store hardware chain because it was a good
investment. This is an example of .

A. a lateral growth strategy


B.a combination purchase
C.related diversification
D. unrelated diversification
319) Because of the good profits and a fear of growing too fast, you
decide to keep Taco Rocket in the same business and do not change the
menu. You hope to retain the same market share and return-on-investment
record. This is considered a strategy.

A. stability
B. growth
C. combination
D. diversification

320) Colleen invested a dollar in the Powerball Lottery and won $60
million. Subsequently, she decides to start her own business selling
lawnmowers.
a. Colleen is successful after the first 3 years, and she is approached by a
competitor who is nearing retirement age. The competitor is interested in
selling his business to Colleen. For Colleen, this would be a(n)
strategy.

A. unrelated diversification
B. horizontal integration
C. vertical integration
D. retrenchment
321) Colleen finds a business opportunity in a supplier who sells her wheels for
lawnmowers. This would be a(n) strategy.

A. unrelated diversification
B.horizontal integration
C.vertical integration
D. related diversification

322) A business broker hears that Colleen is interested in purchasing a


business and approaches her with an offer to sell her a company that owns
a patent on a new roofing product and who installs this new roofing in the
southwestern United States. If she buys this firm, she will be using a(n)
strategy.

i. unrelated diversification
ii. horizontal integration
iii. vertical integration
iv. related diversification
b. Initially she begins a business that has a single-line business. She realizes
that this organization will need a strategy.

A. business-level
B. organizational
C. operational-level
D. corporate-level

323) No matter which business Colleen decides to buy, she intends to operate
each business independently and allow each to determine its own strategy.
This will mean that each company will be a(n) .

A. operational unit
B.strategic business unit
C.competitive advantage
D. legal subunit

324) Colleen decides that she wants to assemble lawn mowers. She decides that
she wants a business to develop a distinctive edge in producing high-quality
lawn mowers. This emphasis on quality is to be so strong that her company
will have a that will set her company apart from her
competition.

A. core competence
B.competitive power
C.legal propriety
D. competitive advantage

325) You purchased Shanghai Grill and Zorro Distributors after being in
business for five years. What level of strategy integrates the strategies
of your various business units?

A. corporate level
B. business level
C. functional level
D. strategic level
326) You called the Boston Consulting Group (BCG), and they have provided
you with some advice based on their famous corporate portfolio matrix.
Your oldest holding, Taco Rocket, has not grown much in recent years, but
due to low debt, it generates a huge amount of cash. According to BCG,
Taco Rocket would be considered a .

A. cash cow
B. star
C. question mark
D. dog

327) Recently, you also purchased a company that manufactures a new satellite
dish, allowing you to enter into the cable television market. The business is
profitable and growing, but the technological unknowns make it risky.
BGC considers it a .

A. cash cow
B. star
C. question mark
D. dog

328) Another purchase you made was to acquire a local coffee-cart chain with
30 locations around the city. You don’t see it growing very much, but
then, it doesn’t cost much to operate. BCG would label this venture a
.

A. cash cow
B. star
C. question mark
D. dog

329) You now need to decide how to best manage and utilize the large number
of assets represented by the numerous companies you own. For each SBU,
you must create a strategy to determine how your corporation
should compete in each of its businesses.

A. corporate-level
B. business-level
C. functional-level
D. tactical

330) Your old friend, Ariel Eskenazi, is the owner and general manager
of Megabyte Center, a computer reseller and systems integrator located
in Panama City, Panama. Since leaving IBM to start a business in his home
country, Ariel’s company has steadily grown, due in large part to
the business partnerships he’s established over the years with large foreign
computer and software firms, such as Goldstar and Microsoft. These
relationships have helped his company win considerable market share
in Panama, as well as in other parts ofLatin America. However, since the
1999 turnover of the Panama Canal to the Panamanian government, there
has been a huge influx of foreign capital into Panama. For example, several
large Asian firms have made Panama a beachhead for their American
operations. Tourism is on the rise, with over a score of new hotels built in
the metropolitan area alone over the past 3 years. As a result, demand for
Megabyte’s products and services has increased markedly, but so has the
level and diversity of its competition. While Megabyte’s customer base has
remained fairly loyal, many longtime customers are beginning to demand
price concessions and enhanced service levels in return for their continued
business. Additionally, Ariel has learned recently that several of his former
suppliers and business partners are considering establishing local sales
offices of their own in Panama City. Ariel knows you are very
knowledgeable about competitive strategy and calls you asking for advice.
You begin by telling him a little about Michael Porter’s five forces theory
of competition and the three generic strategies.
a. Demand growth and increasing intensity and diversity of competition that
Ariel is facing is indicative of which one of Porter’s five forces?

A. threat of substitutes
B. threat of new entrants
C. bargaining power of customers
D. current rivalry

b. Once Ariel has assessed the five forces and determined the threats and
opportunities that exist in the current environment, you tell him that he is
then ready to select an appropriate competitive strategy. Porter outlines
three “generic” strategies: cost leadership, differentiation, and
.
A. niche
B. segmentation
C. focus
D. stuck in the middle

c. Because his customers are demanding price concessions and enhanced


service levels in return for their continued business, Ariel decides that he
wants to compete by offering unique products that are widely valued by
customers.. What strategy is Ariel following?

i. focus
ii. cost leadership
iii. differentiation
iv. stability

d. Ariel is considering forgoing the retail side of his business


entirely. Instead, he will redirect his resources toward reselling hardware
and software and providing systems integration services to the Latin
American governmental and industrial sectors. Such a move would be most
representative of which one of Porter’s generic strategies?

A. niche
B.segmentation
C.focus
D. stuck in the middle

331) The environmental segments that comprise the general


environment typically will NOT include

a. demographic factors.
b. socio-cultural factors.
c. substitute products or services.
d. technological factors.
332) In the airline industry, frequent-flyer programs, ticket kiosks, and e-
ticketing are all examples of capabilities that are
a. rare.
b. causally ambiguous.
c. socially complex.
d. valuable.

333) Business-level strategies are concerned specifically with:

a. creating differences between the firm's position and its rivals.


b. selecting the industries in which the firm will compete.
c. how functional areas will be organized within the firm.
d. how a business with multiple physical locations will operate one of
those locations.

334) When a product's unique attributes provide value to customers, the firm
is implementing

a. a differentiation strategy.
b. a cost leadership strategy.
c. an integrated cost leadership/differentiation strategy.
d. a single-product strategy.

335) A company pursuing a differentiation or focused differentiation strategy


would

a. have highly efficient systems linking suppliers' products with the


firm's production processes.
b. use economies of scale.
c. have strong capabilities in basic research.
d. make investments in easy-to-use manufacturing technologies.
336) T or F? A firm using a differentiation strategy can charge a premium
price.
Answer: T

337) On the whole there are more competitive responses to

a. strategic actions than to tactical actions.


b. tactical actions than to strategic actions.
c. buyer pressures than to supplier pressures.
d. the demands of the top management team than to industry structural pressures

338) Investors in a company judge the adequacy of the returns on their


investment in relation to:

a. the returns on other investments of similar risk..


b. the stock market's overall performance.
c. the initial size of the investment.
d. the prime interest rate.
Answer: a. the returns on other investments of similar risk..
lOMoARcPSD|6004607

Strategic Management- MCQ

Strategic Management (Indian Institute of Management Kozhikode)

Downloaded by Imad Quadri (imad3889@yahoo.co.in)


lOMoARcPSD|6004607

StuDocu is not sponsored or endorsed by any college or university

Downloaded by Imad Quadri (imad3889@yahoo.co.in)


1. Horizontal integration is concerned with
(A) Production

(B) Quality

(C) Product planning

(D) All of the above

2. It refers to formal and informal rules, regulations and procedures that complement
the company structure
(A) Strategy

(B) Systems

(C) Environment

(D) All of the above

3. Micro environment is the.............environment of a company.


(A) Working

(B) Human

(C) External

(D) Internal

4. Techniques used in environmental appraisal are


(A) single-variable extrapolation/multivariable interaction analysis

(B) Structured/ unstructured expert/inexpert opinion

(C) Dynamic modes and mapping

(D) All of the above

5. Match The Following

Question Your Answer

a. Ballast business 1. Give the highest priority

b. Gap analysis 2. Difference between desired and projected performance

c. Corporate parenting 3. Managing SBU’s by a corporate

d. Heartland business 4. Fit well but low opportunities

The correct answer is

(A) a-1, b-3, c-2, d-4

(B) a-2, b-1, c-4, d-3

(C) a-4, b-2, c-3, d-1

(D) a-2, b-3, c-4, d-1

6. It enables the strategists to take corrective action at the right time


(A) Implementation control

(B) Special alert control

(C) Strategic Surveillance control

(D) Premise control


7. Like roots of a tree, of organization is hidden from direct view.
(A) Performance

(B) Strategy

(C) Core competence

(D) All of the above

8. Changes in company.............also necessitates changes in the systems in various


degrees
(A) structure

(B) system

(C) strategy

(D) turnover

9. The actual performance deviates positively over the budgeted performance. This is
an indication of.........performance.
(A) superior

(B) inferior

(C) constant

(D) any of the above

10. Criteria for making an evaluation is (are)


(A) Consistency with goals

(B) Consistency with environment

(C) Money

(D) All of the above

11. 11. The.............of any organization is “the aggregate of all conditions, events and
influences that surround and affect it.”
(A) system

(B) environment

(C) structure

(D) strategy

12. Strategic management is mainly the responsibility of


(A) Lower management

(B) Middle management

(C) Top management

(D) All of the above

13. The major issue(s) of appraisal system is (are)


(A) Factors of appraisal

(B) Relevance of appraisal

(C) Procedure of appraisal

(D) All of the above


14. 14. They have time based utility
(A) Goals

(B) Resources

(C) both ‘A’ and ‘B’

(D) None of the above

15. Formal systems are adopted to bring & amalgamation of decentralized units
into product groups.
(A) Manpower

(B) Co-ordination

(C) Production

(D) All of the above

Answers:
1-(A), 2-(B), 3-(D), 4-(D), 5-(C), 6-(D), 7-(C), 8-(B), 9-(A), 10-(D), 11-(B), 12-(C), 13-(D), 14-(C), 15-
(B)
16. Change in company’s ………. gives rise to problems necessitating a new ……… to
be made
(A) structure, strategy

(B) strategy, structure

(C) structure, structure

(D) strategy, strategy

17. Systems are formal and informal rules and regulations that complement the company
………..
(A) strategy

(B) structure

(C) system

(D) environment

18. The reasons for acquisition are


(A) Increased market power

(B) Increased diversification

(C) Increased speed to market

(D) All of the above

19. Market research is conducted by


(A) By employees

(B) By research agencies

(C) By consultants

(D) all of the above

20. Vertical integration is concerned with


(A) supply chain

(B) production
(C) Quality

(D) planning

21. cost accounting measures the cost of producing and ignores the cost of
non- producing
(A) Lean

(B) Traditional

(C) Environmental

(D) Throughput

22. Match the following

Question Correct Answer

a. Retrenchment Strategies 1. Retrenchments – either internally or externally

2. Contraction of activities through elimination of the scope of one or m


b. Divestment Strategies business

c. Turnaround Strategies 3. Involves the sale or liquidation of a portion of a business


(A) a-1, b-2, c-3

(B) a-3, b-2, c-1

(C) a-2, b-3, c-1

(D) a-3, b-1, c-2

23. ETOP stands for .


(A) environmental threat & opportunity project

(B) environmental threat & opportunity profile

(C) environmental treaty & opportunity profile

(D) environmental threat & optimum profile

24. The control process requires the following types of information


(A) Planned performance

(B) Variances

(C) Reasons

(D) All of the above

25. Financial environment is concerned with


(A) demand & supply of money

(B) capital markets

(C) both ‘A’ and ‘B’

(D) None of the above

26. The process of forecasting an organization’s future demands for and supply of
right type of people in right number is
(A) Product planning

(B) Process planning

(C) Man power planning


(D) All of the above

27. It is designed to monitor a broad range of events inside and outside the company
that are likely to threaten a firm’s strategy
(A) Strategic surveillance

(B) Strategic planning

(C) both ‘A’ and ‘B’

(D) None of the above

28. Harvest strategy is used for


(A) Dogs

(B) Question marks

(C) both ‘A’ and ‘B’

(D) none of the above

29. These are critical situations that occur unexpectedly and threaten the course of
a firm’s strategy
(A) Crisis

(B) Emergency

(C) Shutdown

(D) All of the above

30. Attack strategies are


(A) frontal attack

(B) flank attack

(C) encirclement attack

(D) all of the above

Answers:
16-(B), 17-(B), 18-(D), 19-(D), 20-(A), 21-(B), 22-(C), 23-(B), 24-(D), 25-(C), 26-(C), 27-(A), 28-(C),
29-(A), 30-(D)
31. An approach that strives to follow ethical principles and percepts is
(A) Moral management

(B) Immoral management

(C) Amoral management

(D) None of the above

32. Niche marketing means


(A) End user specialist

(B) Specific customer specialist

(C) Geographic specialist

(D) all of the above

33. Type(s) important managerial skill(s) required for the effective strategic management
(A) Conceptual skill

(B) Human skill


(C) Intellectual skill

(D) all of the above

34. The model(s) of social responsibility is (are)


(A) Austere Model

(B) Vendors Model

(C) Civic Model

(D) all of the above

35. Delay in measurement defeats the purpose of evaluation.


(A) Time

(B) Quality

(C) Production

(D) All of the above

36. Module of Flotilla strikes an optimum balance of and flexibility


(A) Optimisation

(B) Standardisation

(C) Organization

(D) All of the above

37. The are distinct little business set up as units in a larger company.
(A) Small business Units

(B) Strategic business Units

(C) Internal business Units

(D) All of the above

38. Knowledge of the relationships between choices, environment & outcomes is


(A) Familiarity

(B) Conversancy

(C) Informality

(D) Normality

39. should have the ability to develop a vision to see patterns into the future.
(A) Leaders

(B) Managers

(C) Management

(D) Workers

40. Factors encouraging joint ventures are


(A) Uneconomical separate existence

(B) Risk of business gets shared

(C) Sharing competence of each other

(D) All of the above


41. is the drive to achieve beyond one’s expectations.
(A) Motivation

(B) Training

(C) Development programme

(D) All of the above

42. The decisions which are applied to structured or routine problems are
(A) Semi-programmed decisions

(B) Un-programmed

(C) Programmed

(D) Any of the above

43. Strategic management deals with


(A) Production and quality

(B) Profit and loss

(C) Business process

(D) All of the above

44. Benchmarking is
(A) Historical analysis

(B) Competitive analysis

(C) Re-engineering

(D) All of the above

45. GDP stands for


(A) Gross domestic product

(B) Gross domestic production

(C) Gross daily production

(D) Gross domestic process

Answers:
31-(), 32-(), 33-(D), 34-(D), 35-(A), 36-(B), 37-(B), 38-(A), 39-(A), 40-(D), 41-(A), 42-(C), 43-(C), 44-
(B), 45-(A)
46. Macro environment
(A) Political- legal

(B) socio-cultural

(C) economic-demographic

(D) All of the above

47. Factor(s) determining HRP (Human Resource Planning)


(A) Type & Strategy of organisation

(B) Environmental uncertainties

(C) Type & quality of information

(D) All of the above


48. A major part of strategy implementation is
(A) Planning

(B) Communication

(C) Resource allocation

(D) Monitoring

49. These people are charged with the responsibility of continuous screening
of performance?
(A) Managers

(B) Supervisors

(C) Top management

(D) Audit committee

50. and greater accountability replace formal control.


(A) Self-discipline

(B) Self appraisal

(C) both ‘A’ and ‘B’

(D) None of the above

51................................function applies to all company levels irrespective of levels of hierarchy.


(A) Planning

(B) Organizing

(C) Staffing

(D) Directing

52. Three C’s affecting today’s companies are


(A) Customer, Competition, Change

(B) Cost, Competition, Change

(C) Customer, Competition, Cost

(D) Customer, Cost, Change

53. Strategic management is concerned with


(A) Short range planning

(B) Long range planning

(C) Both ‘A’ and ‘B’

(D) None of the above

54. It provides a way to bring in the people dimension in macro company analysis
without using psychological models of human behaviour.
(A) Environment

(B) Society

(C) Culture

(D) All of the above

55. It is used extensively in organisation and is concerned with action or performance.


(A) Operational control

(B) Production control

(C) Quality control

(D) All of the above

56. Companies employ trained to develop “competency models” to identify


potential leaders.
(A) Consultants

(B) Psychologists

(C) Trainers

(D) All of the above

57. Style of a company are the patterns of actions taken by members of ….


management over a period of time
(A) Top

(B) Middle

(C) Lower

(D) Any of the above

58. It concentrates on organisation design and work flow


(A) Systems approach

(B) Flotilla

(C) Method study

(D) Work study

59. The benefits of a change in process are defined in terms of cost savings
(A) Labour

(B) Infrastructure

(C) Production

(D) Raw material

60. Core ideology


(A) Core values

(B) Core purpose

(C) both ‘A’ and ‘B’

(D) None of the above

Answers:
46-(D), 47-(D), 48-(C), 49-(D), 50-(A), 51-(C), 52-(A), 53-(B), 54-(C), 55-(A), 56-(B), 57-(A), 58-(B),
59-(A), 60-(C)

1. The fundamental purpose for the existence of any organization is described by its

a. policies
ADVERTISEMENTS:

b. mission

c. procedures

d. strategy

Ans. b

ADVERTISEMENTS:

2. The fundamental purpose of an organization’s mission statement is to

a. create a good human relations climate in the organization

b. define the organization’s purpose in society

c. define the operational structure of the

organization ADVERTISEMENTS:

d. generate good public relations for the organization

Ans. b

3. The acronym SWOT stands for

a. Special Weapons for Operations Timeliness

b. Services, Worldwide Optimization, and Transport

c. Strengths Worldwide Overcome Threats

d. Strengths, Weaknesses, Opportunities, and Threats

Ans. d

4. Which of the following is not a characteristic of strategic management that makes it


different from other types of management?

a. It is interdisciplinary.

b. It has an external focus.

c. It has an internal focus.

d. It concerns the present direction of the organization.

Ans. d

5. Which of the following is an issue considered in developing corporate strategies?

a. What business(es) are we in?

b. What direction are we going?

c. What resources do we have to implement our strategies?


d. What businesses are we in and what to do with those businesses?

Ans. c

6. Which of the following is NOT a major element of the strategic management process?

a. Formulating strategy

b. Implementing strategy

c. Evaluating strategy

c. Assigning administrative tasks

Ans. d

7. Competitive advantage can best be described as:

a. increased efficiency.

b. what sets an organization apart.

c. a strength of the organization.

d. intangible resources.

Ans. a

8. is the foundation of blue ocean strategy.

a. Innovation

b. Value creation

c. Value innovation

d. value cost trade-off

Ans. c

9. The various organizational routines and processes that determine how efficiently
and effectively the organization transforms its inputs into outputs are called:

a. strengths.

b. core competencies.

c. capabilities.

d. customer value.

Ans. b

10. When defining strategic management the most important thing to remember is that it is:

a. Not as easy as you think

b. Mainly the province of senior managers


c. A living evolving process

d. More conceptual than practical

e. A way of determining responsibilities

Ans. c

11. An organisation’s strategy:

a. remains set in place longer than the mission and objectives

b. generally forms over a period of time as events unfold

c. tends to be formed at the same time the mission is developed and objectives are formulated

d. is usually conceived at a single time when managers sit down and work out a
comprehensive strategic plan for the next 3-5 years

Ans. b

12. The primary focus of strategic management is:

a. strategic analysis

b. the total organisation

c. strategy formulation

d. strategy implementation.

Ans. b

13. Which of the following is not an advantage of strategic management?

a. It provides organisations with a clearer sense of direction and purpose

b. It helps improve the political, economic, social and technological environment of the organisation

c. It helps orientate management decisions to relevant environmental conditions

d. It helps organisations be proactive rather than reactive

Ans. b

14. Which of the following defines what business or businesses the firm is in or should be in?

a. Business strategy

b. Corporate strategy

c. Functional strategy

d. National strategy

Ans. b

15. Which of the following defines how each individual business unit will attempt to achieve
its mission?
a. Business strategy

b. Corporate strategy

c. Functional strategy

d. National strategy

Ans. a

16. Which of the following focuses on supporting the corporate and business strategies?

a. Competitive strategy

b. Corporate strategy

c. Operational strategy

d. National strategy

e. Mission strategy

Ans. c

17. Which one of the following is not a primary task of strategic managers?

a. Establishing strategic objectives

b. Developing the steps to follow in implementing operational level plans

c. Defining the business and developing a mission

d. Developing a strategy

e. Implementing and evaluating the chosen strategy

Ans. b

18. The task of strategy choice involves:

a. developing plans and activities which will improve the organisation’s performance and
competitive position

b. determining how the organisation can be more market and efficiency oriented

c. monitoring whether the organisation is achieving good financial performance

d. keeping the organisation free of debt

Ans. a

19. Which one of the following is at the core of strategic management?

a. Choosing which organisational objectives to focus on

b. Being alert for opportunities to change work responsibilities

c. Adapting the organisation to a changing external environment


d. Choosing whether to make decisions autocratically or on the basis of participation

Ans. c

20. The corporate level is where top management directs:

a. all employees for orientation

b. its efforts to stabilize recruitment needs

c. overall strategy for the entire organization

d. overall sales projections

Ans. c

21. The three organizational levels are:

a. corporate level, business level, functional level

b. corporate level, business unit level, functional level

c. corporate strategy level, business unit level, functional level

d. corporate strategy level, business level, specialist level

Ans. a

22. Which of the following is an example of competing on quick response?

a. a firm produces its product with less raw material waste than its competitors

b. a firm offers more reliable products than its competitors

c. a firm’s products are introduced into the market faster than its competitors’

d. a firm’s research and development department generates many ideas for new products

Ans. c

23. Which one of the following is NOT included in the Porter’s Five Forces model:

a. Potential development of substitute products

b. Bargaining power of suppliers

c. Rivalry among stockholders

d. Rivalry among competing firms

Ans. c

24. What is meant by the term ‘Stakeholder’?

a. A person who is not related with a business.

b. A person who is related with a business.

c. A person who owns a business.


d. A person who purchases the shares of a business.

Ans. b

25. Of the following, which one would NOT be considered one of the components of a
mission statement?

a. The target market for XYZ is oil and gas producers as well as producers of chemicals.

b. XYZ shall hire only those individuals who have with sufficient educational levels so as to be
of benefit to our customers

c. The customers of XYZ shall include global and local consumers of gas and oil products
and domestic users of nontoxic chemicals

d. The technologies utilized by XYZ shall focus upon development of alternative sources of gas
and oil so as to remain competitive within the industry

Ans. b

26. The strategic management process is

a. a set of activities that will assure a temporary advantage and average returns for the firm.

b. a decision-making activity concerned with a firm’s internal resources, capabilities,


and competencies, independent of the conditions in its external environment.

c. a process directed by top-management with input from other stakeholders that seeks to
achieve above-average returns for investors through effective use of the organization’s resources.

d. the full set of commitments, decisions, and actions required for the firm to achieve above-
average returns and strategic competitiveness..

Ans. d

27. The goal of the organization’s is to capture the hearts and minds of employees,
challenge them, and evoke their emotions and dreams.

a. vision

b. mission

c. culture

d. strategy

Ans. a

28. A firm’s mission

a. is a statement of a firm’s business in which it intends to compete and the customers which
it intends to serve.

b. is an internally-focused affirmation of the organization’s financial, social, and ethical goals.

c. is mainly intended to emotionally inspire employees and other stakeholders.

d. is developed by a firm before the firm develops its vision.

Ans. a
29. The environmental segments that comprise the general environment typically will
NOT include

a. demographic factors.

b. sociocultural factors.

c. substitute products or services.

d. technological factors.

Ans. c

30. An analysis of the economic segment of the external environment would include all of
the following EXCEPT

a. interest rates.

b. international trade.

c. the strength of the U.S. dollar.

d. the move toward a contingent workforce.

Ans. d

31. Product differentiation refers to the:

a. ability of the buyers of a product to negotiate a lower price.

b. response of incumbent firms to new entrants.

c. belief by customers that a product is unique.

d. fact that as more of a product is produced the cheaper it becomes per unit.

Ans. c

32. Which of the following is NOT an entry barrier to an industry?

a. expected competitor retaliation

b. economies of scale

c. customer product loyalty

d. bargaining power of suppliers

Ans. d

33. Switching costs refer to the:

a. cost to a producer to exchange equipment in a facility when new technologies emerge.

b. cost of changing the firm’s strategic group.

c. one-time costs suppliers incur when selling to a different customer.

d. one-time costs customers incur when buying from a different supplier.


Ans. d

34. New entrants to an industry are more likely when (i.e., entry barriers are low when…)

a. it is difficult to gain access to distribution channels.

b. economies of scale in the industry are high.

c. product differentiation in the industry is low.

d. capital requirements in the industry are high.

Ans. c

35. Suppliers are powerful when:

a. satisfactory substitutes are available.

b. they sell a commodity product.

c. they offer a credible threat of forward integration.

d. they are in a highly fragmented industry.

Ans. c

36. The highest amount a firm can charge for its products is most directly affected by

a. expected retaliation from competitors.

b. the cost of substitute products.

c. variable costs of production.

d. customers’ high switching costs.

Ans. b

37. All of the following are forces that create high rivalry within an industry EXCEPT

a. numerous or equally balanced competitors.

b. high fixed costs.

c. fast industry growth.

d. high storage costs.

Ans. c

38. According to the five factors model, an attractive industry would have all of the
following characteristics EXCEPT:

a. low barriers to entry.

b. suppliers with low bargaining power.

c. a moderate degree of rivalry among competitors.


d. few good product substitutes.

Ans. a

39. Internal analysis enables a firm to determine what the firm

a. can do.

b. should do.

c. will do.

d. might do.

Ans. a

40. An external analysis enables a firm to determine what the firm

a. can do.

b. should do.

c. will do.

d. might do.

Ans. d

41. is/are the source of a firm’s , which is/are the source of


the firm’s .

a. Resources, capabilities, core competencies

b. Capabilities, resources, core competencies

c. Capabilities, resources, above average returns

d. Core competencies, resources, competitive advantage

Ans. a

42. In the airline industry, frequent-flyer programs, ticket kiosks, and e-ticketing are
all examples of capabilities that are

a. rare.

b. causally ambiguous.

c. socially complex.

d. valuable.

Ans. d

43. Firms with few competitive resources are more likely

a. to not respond to competitive actions.

b. respond quickly to competitive actions.


c. delay responding to competitive actions.

d. respond to strategic actions, but not to tactical actions.

Ans. c

44. Competitors are more likely to respond to competitive actions that are taken by

a. differentiators.

b. larger companies.

c. first movers.

d. market leaders.

Ans. d

45. What can be defined as the art and science of formulating, implementing and
evaluating cross-functional decisions that enable an organization to achieve its
objectives?

a. Strategy formulation

b. Strategy evaluation

c. Strategy implementation

d. Strategic management

e. Strategic leading

Ans. d

46. Which of the following is not a cultural product?

a. Rites

b. Emotions

c. Rituals

d. Sagas

e. Symbols

Ans. b

47. Which individuals are most responsible for the success and failure of an organization?

a. Strategists

b. Financial planners

c. Personnel directors

d. Stakeholders

e. Human resource managers


Ans. a

48. Which of the following is an element of a firm’s remote external environment?

a. Competition

b. Political agencies

c. Suppliers

d. Trade union

Ans. b

49. Long-term objectives should be all of the following except:

a. measurable.

b. continually changing.

c. reasonable.

d. challenging.

e. consistent.

Ans. b

50. What are guides to decision making?

a. laws

b. rules

c. policies

d. procedures

e. goals

Ans. c

51. According to Greenley, strategic management offers all of these benefits except that

a. it provides an objective view of management problems.

b. it creates a framework for internal communication among personnel.

c. it encourages a favourable attitude toward change.

d. it maximizes the effects of adverse conditions and changes.

e. it gives a degree of discipline and formality to the management of a business.

Ans. d

52. The vision and mission statement can often be found

a. in the SEC report.


b. in annual reports.

c. on customer receipts.

d. on supplier invoices.

e. on community news bulletins

Ans. b

53. Which group would be classified as a stakeholder?

a. Communities

b. Banks

c. Suppliers

d. Employees

e. All of these

Ans. e

54. The process of performing an external audit needs to include:

a. only top level managers, as it’s a planning function.

b. as many managers and employees as possible.

c. primarily front-line supervisors

d. between 15 to 20 managers for it to be valid

e. stockholders and external government agencies

Ans. b

55. Which of the following is not a stage of strategy formulation techniques?

a. Formulation Framework

b. Matching stage

c. External factor evaluation

d. Decision stage

Ans. b

56. ST Strategies is an important strategy to

a. Match weakness with opportunities of the firm

b. Overcome external threats

c. Obtain benefit from its resources

d. Overcome its weakness and reducing threats


Ans. b

57. The immediate external environment includes:

a. Divisions

b. S. B. U. s

c. Competitors

d. Management

Ans. c

58. The comprises economic and social conditions, political priorities and
technological developments, all of which must be anticipated, monitored, assessed
and incorporated into the executive’s decision making.

a. Internal environment

b. Task environment

c. Operating environment

d. Societal environment

Ans. d

59. Strategic management involves the , directing, and controlling of a


company’s strategy-related decisions and actions.

a. Financing; marketing

b. Planning; financing

c. Planning; organizing

d. Marketing; planning

Ans. c

60. A strategy is a company’s

a. Value statement

b. Pricing policy

c. Game Plan to outsmart competitor

d. Long-term objective

Ans. c

61. Strategy-formulation concepts and tools

a. Do not differ greatly for different size and type of organizations

b. Differ greatly for different size and type of organizations


c. Do not differ greatly for profit or nonprofit organizations but differ in small and large organizations

d. None of the mentioned options

Ans. a

62. Annual objectives

a. Are not critical to success

b. Serve as guidelines for action, directing and channeling efforts and activities of
organization members

c. Are not important for employee motivation and identification

d. Do not provide a basis for organizational design

Ans. b

63. Annual objectives

a. Need not to be consistent

b. Should be easily achievable

c. Should be measurable

d. Should be confidential and not to be communicated throughout the organization

Ans. c

64. Which of the following resources is used by all organizations to achieve


desired objectives?

a. Financial resources,

b. Physical resources,

c. Human resources

d. All of the mentioned options

Ans. d

65. Strategic management is

a. A pure science.

b. Based mainly on intuition.

c. Needed mainly when organizational performance falls.

d. Based on the use of quantitative and qualitative information.

Ans. d

66. Large-scale, future-oriented plans, for interacting with the competitive environment
to achieve company objectives refers to its
a. Strategy

b. Goals

c. Competitive analysis

d. Dynamic policies

Ans. a

67. Strategic issues require which level of management decisions?

a. Operative

b. Top

c. Front-line

d. Middle

Ans. b

68. Which of these basic questions should a vision statement answer?

a. What is our business?

b. Who are our employees?

c. Why do we exist?

d. What do we want to become?

Ans. d

69. is not part of an external audit.

a. Analyzing competitors

b. Analyzing financial ratios

c. Analyzing available technologies

d. Studying the political environment

Ans. b

70. Strategic management process activate in the sequence of

a. Environmental scanning, Strategy formulation, Implementation, control and evaluation

b. Strategy formulation, Environmental scanning, Implementation, control and evaluation

c. Environmental scanning, Strategy Implementation, formulation, control and evaluation

d. Strategy formulation, Implementation, control, evaluation, Environmental scanning

Ans. a
71. KAPKAL Power’s interested to achieve a 10 percent return on equity (ROE) in their core
electric utility, 14 percent ROE on water resource operations, and 15 percent ROE on
support businesses. It is

a. Mission

b. Strategy

c. Objective

d. Policy

Ans. c

72. “A possible and desirable future state of an organization” is called:

a. Mission

b. Vision

c. Strategy implementation

d. None of above

Ans. b

73. Strategic decisions are based on what managers , rather than on what
they .

a. Know; forecast

b. React to; anticipate

c. Forecast; know

d. Compromise with; analyze

Ans. c

74. “To improve economic strength of society and function as a good corporate citizen on
a local, state, and national basis in all countries in which we do business”. This is a
mission statement that contains:

a. Self-concept

b. Economic concern

c. Products or Services

d. Concern for Public Image

Ans. d

75. Strategic-management audit is known as:

a. Environmental scanning

b. Strategy formulation
c. Strategy control

d. Strategy evaluation

Ans. a

76. Forecasting tools can be broadly categorized into two groups. Those are:

a. Qualitative, Operational

b. Quantitative, Operational

c. Qualitative, Quantitative

d. Regression and time series analysis

Ans. c

77. identifies a firm’s major competitors and their particular strengths and
weaknesses in relation to a sample firm’s strategic position.

a. Competitive Profile Matrix

b. External Factor Evaluation matrix

c. Internal Factor Evaluation Matrix

d. Boston consulting group matrix

Ans. a

78. Organizing means an identifiable group of people contributing their efforts towards
the attainment of same goal. It is important at the time of:

a. Environmental scanning

b. Strategy formulation

c. Strategy Implementation

d. Strategy evaluation

Ans. c

79. In a turbulent and competitive free enterprise environment, a firm will succeed only if
it takes a(n) stance towards change.

a. Reactive

b. Proactive

c. Anti-regulatory or anti-government

d. Vision and not mission

Ans. b

80. Which statement best describes intuition?


a. It represents the marginal factor in decision-making.

b. It represents a minor factor in decision-making integrated with analysis.

c. It should be coupled with analysis in decision-making.

d. It is better than analysis in decision-making.

e. It is management by ignorance.

Ans. c

81. What are the means by which long-term objectives will be achieved?

a. Strategies.

b. Strengths.

c. Weaknesses.

d. Policies.

e. Opportunities.

Ans. a

82. Which of these basic questions should a vision statement answer?

a. What is our business?

b. Who are our employees?

c. Why do we exist?

d. What do we want to become?

e. Who are our competitors?

Ans. d

83. When an industry relies heavily on government contracts, which forecasts can be the
most important part of an external audit.

a. economic

b. political

c. technological

d. competitive

e. Multinational

Ans. b

84. is not part of an external audit.

a. Analyzing competitors
b. Analyzing financial ratios

q. Analyzing available technologies

d. Studying the political environment

e. Analyzing social, cultural, demographic and geographic forces

Ans. b

85. Which individuals are most responsible for the success and failure of an organization?

a. Strategists

b. Financial planners

c. Personnel directors

d. Stakeholders

e. Human resource managers

Ans. a

86. Long-term objectives should be all of the following except:

a. Measurable.

b. Continually changing.

c. Reasonable.

d. Challenging.

e. Consistent.

Ans. b

87. What are guides to decision making?

a. laws

b. rules

c. policies

d. procedures

e. goals

Ans. c

88. Which group would be classified as a stakeholder?

a. Communities

b. Banks

c. Suppliers
d. Employees

e. All of the given options

Ans. e

89. Typically how many strategic decision levels are in the corporate decision-
making hierarchy?

a. 3

b. 4

c. could be more than 5

d. 2

Ans. a

90. Which type of trend can be exemplified by the increasing numbers of two-
income households in a society?

a. Social

b. Economic

c. Cultural

d. Technological

Ans. b

91. External assessment is performed in which of the strategic management phase?

a. Strategy formulation stage

b. Strategy implementation stage

c. Strategy evaluation stage

d. All of the given options

Ans. a

92. Political variables have a significant effect on

a. Strategy formulation and implementation

b. Strategy formulation and evaluation

c. Strategy implementation and evaluation

d. Strategy formulation, implementation and evaluation

Ans. a

93. Strategic decisions ostensibly commit the firm for

a. 1 -2 years
b. The short term

c. one years

d. A long time, typically five years

Ans. d

94. Social responsibility is a critical consideration for a company’s strategic decision


makers since

a. Stockholders demand it

b. The mission statement must express how the company intends to contribute to the societies
that sustain it

c. It increases a company’s profits

d. It helps make decisions

Ans. b

95. “The perfect search engine would understand exactly what you mean and give back
exactly what you want”, this statement is included in the mission statement of an online
firm and is showing which one of the following components?

a. Self-concept

b. Concern for public image

c. A declaration of attitude

d. Philosophy

Ans. b

96. Which of the following are signs of weakness in a company’s competitive position?

a. A return-on-equity is below 25% and earnings per share of less than Rs. 2.00

b. A price set by the firm higher than its rivals

c. A declining market share, poor product quality and few sales in market

d. Lower revenues and profit margin and narrow product line than the market leader

Ans. c

97. It directs at developing new products before competitors do at improving product


quality or at improving manufacturing processes to reduce costs.

a. Marketing

b. Opportunity analysis

c. Research and development

d. Management
Ans. c

98. “Identifying and evaluating key social, political, economic, technological and
competitive trends and events”. Which of the followings best describes this statement?

a. Developing an effective mission statement

b. Conducting an internal audit

c. Performing an external audit

d. Formulating strategy

Ans. c

99. What is the central purpose of strategic evaluation?

a. Evaluate effectiveness of strategy to achieve organisational objectives.

b. Evaluate effectiveness of control system to measure achievements.

c. Evaluate effectiveness of strategies to be implemented efficiently.

d. Evaluate effectiveness of the strategy implementation process.

Ans. a

100. Strategy evaluation at the level involves using specific performance


measures-qualitative and quantitative-for each functional area.

a. organizational

b. operational

c. functional

d. production

Ans. c
Strategic
Management
Multiple choice
questions

DR. SESHU BABU


Mr. CHUOP Theot Therith

2010
Strategic
Management
PART A: MULTIPLE CHOICE QUESTIONS

CHAPTER ONE
The Nature of Strategic Management

1. Which of these is not a reason why some firms do no strategic planning?


a. Laziness
b. Competitive leadership
c. Honest difference of opinion
d. Poor reward structures

2 .Developing a vision and mission, identifying an organization's external opportunities


and threats, and determining internal strengths and weaknesses are all activities.
a. strategy-formulation
b. strategy-implementation
c. long-range planning
d. short-range planning

3 . The means by which long-term objectives will be achieved are


a. mission statements
b. strategies.
c. vision statements.
d. long-term goals.

CHAPTER TWO
Business Mission

4. The answers the question "What do we want to become?" whereas


answers the question "What is our business?"
a. vision statement; mission statement
b. short-term objectives; long-term objectives
c. objectives; strategies
d. mission; vision

5. What is the recommended length of an effective mission statement?


a. One page
b. Less than 200 words
c. One sentence of 10 to 20 words.
d. There is no recommendation. It can be as long as the management wants.

CHAPTER THREE
External Assessment

6. represents the average score in both EFE and CPM.


a. 2.0
b. 3.0
c. 2.5
d. 4.0

7. All of these, except , are part of Porter's competitive forces in industry


analysis.

Prepared by: CHUOP Theot 1


Therith
a. potential entry of new competitors
b. bargaining power of suppliers
c. development of substitute products
d. bargaining power of union

8. is based on the assumption that the future will be just like the past. [Hint]
a. Delphi forecasts
b. Econometric models
c. Linear regression
d. Scenario forecasts

CHAPTER FOUR
Internal Assessment

9. Shorthand words use to capture a vision or to reinforce old or new values in a


firm's culture are called
a. Metaphors
b. Sagas
c. Rituals
d. Symbols

10. In an IFE Matrix, the weight range is from and the ratings range from
.

a. to 1.0; 1.0 to 4.0


b. to 1.0; 0.0 to 4.0
c. to 3.0; 1.0 to 2.0
d. to 4.0; 0.0 to 1.0

11. An effective information system collects, codes, stores, synthesizes, and


information in such a manner that it answers important operating and strategic questions.
a. Prints
b. Distributes
c. Presents
d. Filters

CHAPTER FIVE
Strategies in Action

12. is adding new, unrelated products or services for present customers.


a. Concentric diversification
b. Horizontal diversification
c. Conglomerate diversification
d. Product development

13. Two reasons for mergers and acquisitions are


a. to increase managerial staff and to minimize economies of scale.
b. to reduce tax obligations and increase managerial staff.
c. to create seasonal trends in sales and to make better use of a new sales force.
d. to provide improved capacity utilization and to gain new technology.

14. Which strategy would be effective when the new products have a counter cyclical sales
pattern compared to an organization's present products?
a. Forward integration
b. Retrenchment
c. Horizontal diversification
d. Market penetration

CHAPTER SIX
Strategy Analysis & Choice

15. A coordinate of in the SPACE Matrix is a defensive profile.


a.+1, +1
b.-4, -2
c. +5, -1
d.-2, +3

16. The first option that should be considered for firms in Quadrant II of the Grand
Strategy Matrix is the strategy.
a. integration
b. intensive
c. defensive
d. diversification

17. The pie slices within the circles of a reveal the percent of corporate profits
contributed by each division.
a.QSPM
b.BCG Matrix
c. SPACE Matrix
d.Grand Strategy Matrix

CHAPTER SEVEN
Implementing Strategies: Management Issues
18. All of the following are stated advantages of a divisional structure except
a. it allows local control of local situations.
b. it leads to a competitive climate within a firm.
c. accountability is clear.
d. it promotes specialization of labor.

19. The average employee performance bonus is percent of pay for individual
performance, percent of pay for group productivity, and percent of
pay for company-wide profitability.
a. 10.5; 5.5; 2.8
b. 6.8; 5.5; 6.4
c. 10.8; 8.5; 12.4
d. 15.4; 12.4; 10.4

20. approach involves delivering parts and materials as needed rather than
being stockpiled
a. JIT
b. MBO
c. PERT
d. CAD-CAM
PART B: ESSAY
Discussion on the applied concepts
in Strategic Management of Nokia Company

I. REVIEW OF NOKIA COMPANY


I.1 GENERATION OF NOKIA

From roots in paper, rubber, and cables, in just over 100 years Nokia becomes a
powerful industrial conglomerate.
The first Nokia century began with Fredrik Idestam's paper mill on the banks of the
Nokianvirta river. Between 1865 and 1967, the company would become a major industrial
force; but it took a merger with a cable company and a rubber firm to set the new Nokia
Corporation on the path to electronics.

The newly formed Nokia Corporation was ideally positioned for a pioneering role in the
early evolution of mobile communications. As European telecommunications markets were
deregulated and mobile networks became global, Nokia led the way with some iconic
products.

As mobile phone use booms, Nokia makes the sector its core business. By the turn of
the century, the company is the world leader. In 1992, Nokia decided to focus on its
telecommunications business. This was probably the most important strategic decision in its
history.
As adoption of the GSM standard grew, new CEO Jorma Ollila put Nokia at the head of
the mobile telephone industry’s global boom – and made it the world leader before the end of
the decade.

Nokia sells its billionth mobile phone as the third generation of mobile technology
emerges. Nokia’s story continues with 3G, mobile multiplayer gaming, multimedia devices
and a look to the future.

I.2. ORGANIZATIONAL STRUCTURE OF NOKIA


Nokia’s organizational structure is designed to position them for a world where the
mobile device, the Internet and the computer are fusing together.
Mobile Solutions is responsible for developing and managing our portfolio of
smartphones and mobile computers. The team is also busy developing a world-class suite of
internet services under the Ovi brand, with a strong focus on maps and navigation, music,
messaging and media.
Mobile Phones is responsible for developing and managing our portfolio of affordable
mobile phones, as well as a range of services that people can access with them.
Markets manage our supply chains, sales channels, brand and marketing activities, and
is responsible for delivering our mobile solutions and mobile phones to the market.
Nokia Siemens Networks, jointly owned by Nokia and Siemens, provides wireless and
fixed network infrastructure, communications and networks service platforms, as well as
professional services to operators and service providers.
NAVTEQ is a leading provider of comprehensive digital map data and related location-
based content and services for automotive navigation systems, mobile navigation devices,
Internet-based mapping applications, and government and business solutions.

II. DISCUSSION ON THE APPLIED CONCEPTS


IN STRATEGIC MANAGEMENT OF NOKIA
The concept of strategic management is refers to (1) Strategy Formulation, (2) Strategy
Implementation and (3) Strategy Evaluation.
The limitation of discussion now is discussed only on strategic formulation: Nokia’s
vision of the future, mission, goals, and strategies.

II.1. VISION OF NOKIA


The Nokia’s vision statement is connecting people to what matters empowers them to
make the most of every moment. Nokia will therefore empower everyone to share and make
the most of their life by offering irresistible personal experiences. Nokia want to become the
leading provider of mobile solutions.
The full power of being connected: By 2015, all people will experience the full power of
being connected everywhere anytime.
Enable people to be wherever they want, whenever they want: Highly personalized and
contextually relevant solutions become passports that take us wherever we want or need to be,
whenever we want or need to be there.
Life becomes more flexible and spontaneous: Freed from the physical constraints of
time and place, our lives become vastly more experiential, flexible and spontaneous.
Innovating, creating and sharing: Innovating, creating and sharing are social activities
where everyone plays. Co-creation is limited only by the willingness to participate.
Technology becomes invisible: Technology becomes invisible, technical literacy
becomes irrelevant, and intuition takes over.
Nokia never miss an opportunity to get the most out of life: Nokia never miss an
opportunity to get the most out of life thanks to ubiquitous connectivity and combinations of
mobile devices and services that learn and anticipate our personal wants and needs. Like
personal assistants sensing where we are, they point and lead us to what’s available and
relevant.

II.2. MISSION OF NOKIA


Nokia’s mission of connecting people is more than just an advertising buzz phrase. It
describes what we do. It says what benefits we bring to our operator and enterprise customers,
to consumers and to society. It says who we are and what we represent. It is our compass; we
use it to drive our business forward and to deliver value to our customers.
As people have different priorities, Nokia offers a wide range of opportunities and the
flexibility to make sure that they are accessible. The recognition of achievement through a
mix of individual, team and company-wide incentives is one aspect. However, equally
important is the support given to personnel in finding a balance between work and free time,
as well as the creation of opportunities for personal and professional growth.
Ultimately, the most important thing in the workplace is that people can feel motivated,
valued and comfortable. Through the Nokia Values: customer satisfaction, achievement,
respect and renewal, we aim to create such an atmosphere, where people can be themselves
and excel at what they do. Our values also help us establish a firm base from which we can
research, create and deliver the quality products and services our customers want, while
building a corporate image, with which our stakeholders are proud to be connected.
Today, Nokia is a world leader in mobile communications, contributing to the growth
and sustainability of the broader mobility industry. Nokia is dedicated to enhancing people’s
lives and productivity by providing easy-to-use products like mobile phones and solutions for
imaging, games, media, mobile network operators and businesses. Today, Nokia comprises
four business groups: Mobile Phones, Multimedia, Enterprise Solutions, and Networks.
With approximately one in every third mobile phone in use made by Nokia, our
operations influence the lives of hundreds of millions of people; as customers, employees,
business partners and investors. As market leader with global operations, Nokia takes its
responsibility seriously and has many established practices and programs to help make sure
our overall impact is positive.
Mobile communications is already an established force for positive change, driving
economic development and improving social wellbeing. It gives people a voice, helps them
build personal and professional networks, and provides a base for more widespread
information sharing. We aim to increase the penetration of mobile phones in countries where
basic communications infrastructure is sparse or non-existent. Nokia is working closely with
operator customers and governments to offer affordable mobile entry products and solutions
to a broader cross-section of society, in a way that is financially viable.
Discussion on Nokia’s missions: Looking to the missions above, it indicates that Nokia
Company has applied the right concept in strategic management to develop its mission
statement corresponding to the nine components.
1. Customers: Nokia’s customers are people around the world, “…Nokia offers a wide
range of opportunities and the flexibility to make sure that they are accessible…”
2. Product and Services: “…Nokia comprises four business groups: Mobile Phones,
Multimedia, Enterprise Solutions, and Networks…”
3. Markets: “…is a global operation market…”
4. Technology: Nokia applies with high mobile and communication technology.
5. Concern for survival, growth, and profitability: “…contributing to the growth and
sustainability of the broader mobility industry…”
6. Philosophy: Nokia beliefs, “…Connecting people to what matters empowers them to
make the most of every moment…”
7. Self-concept: the major competitive advantage is, “…Seamless, delightful and
effortless user experiences. Vibrant partner ecosystem, People and places enriched
solutions, Direct and continuous consumer relationships, and Regain market position
in all markets…”
8. Concern for public image: “…established force for positive change, driving
economic development and improving social wellbeing…”
9. Concern for employee: “…the most important thing in the workplace is that people
can feel motivated, valued and comfortable…”
II.3. GOALS OF NOKIA
To become the leading provider of mobile solutions, because in the mobile converged
internet space consumers expect seamlessly integrated solutions.
To deliver these solutions requires continuous relationships with consumers and vibrant
ecosystem.

II.4. STRATEGIES OF NOKIA


Strategy1: Competitive environment is changing – traditional competitors making an
effort to increase their volume share in the low end. As the mobile telecom, Internet and PC
industries converge, the industry ecosystem is expanding, and new entrants like Apple, RIM
& Google are creating value with mobile solutions.
Strategy2: Consumer needs are changing – voice and design driven devices business
continues to be important and volume growth will resume. Innovative mobile solutions are
delivering significant new value for many consumers.
Strategy3: The nature of consumers’ relationships with companies is changing – from a
monologue to a conversation … into continuous relationship. From a unified to a segmented
… into a dynamic personalized offering together with their ecosystem.
Strategy4: Irresistible solutions & vibrant ecosystem – starting from a consumer need
or want, radically improve the user experience, co-creating value with developers, operators,
partners by building and open ecosystem.
Strategy5: Transforming into a solutions driven company optimizing user experience.
Strategy6: Laying the foundation for an inclusive and sustainable ecosystem – a
sustainable ecosystem where Nokia services will be only one part of the offering 99% comes
from others.
Strategy7: Direct and continuous consumer relationships – Nokia devices strive to
initiate all consumers to the rich world of services, consumer understanding to maximize
Nokia’s value to the consumer, privacy & trust (permission-based), 300M active users by end
2011.
Strategy8: Best devices – three device areas with different ways to create and capture
value (computers, smart phones, phones).
Strategy9: Smart services – focus on four interconnected services under the Ovi brand,
differentiation through context enriched services – people and places.
II.5. STRATEGIES EVOLUTION OF NOKIA

III. CONCLUSION
According to the discussion above, Nokia is a company that well understanding the
nature of strategy formulation, implementation and evaluation activities. Specially, Nokia has
accurately applied the concepts of strategic management in its strategy formulation. It has
clear vision statement, a good mission corresponding to the nine components, objectives that
stated clearly about what results to accomplish by when, and strongly strategies that plan to
achieve the mission and objectives.
Moreover, Nokia Company has program activities needed to accomplish its plan, cost of
the program, and the procedure – sequence of steps needed to do the job in strategy
implementation as well as the process to monitor performance and take corrective action.

IV. REFERENCES
1. Fred R. David, Strategic Management, 9/e, © 2003 by Prentice-Hall, Inc., A Pearson
Education Company, Upper Saddle River, New Jersey 07458
2. Dr. V.V.R. Seshu Babu, Strategic Management hand out, 2010, BBU, Phnom Penh
3. Nokia company web: http://www.nokia.com/about-nokia/company/vision-and-strategy
4. http://www.essayclub.com/term-papers/Analysis-Mission-Vision-Statement-
Nokia/3501.html
5. http://www.docstoc.com/docs/DownloadDoc.aspx?doc_id=1903541
lOMoARcPSD|6004607

Summary Strategic Management -Chapter 1-9

Strategic Management (King's College London)

Downloaded by Imad Quadri (imad3889@yahoo.co.in)


lOMoARcPSD|6004607

StuDocu is not sponsored or endorsed by any college or university

Downloaded by Imad Quadri (imad3889@yahoo.co.in)


STRATEGIC MANAGEMENT

Chapter 1

Developing the competitive strategic process

A strategy is a set of actions that managers take to increase their company’s


performance relative to rivals. If a company’s strategy does result in superior
performance, it is said to have competitive advantage.

Competitive advantage and superior performance

Superior performance = one’s company profitability relative to that of other


companies in the same or a similar kind of business or industry.

Profitability: the return that a company makes on the capital invested in the
enterprise.

A company’s profitability is determined by the strategies its managers adopt.

A company is said to have a competitive advantage over its rivals when its
profitability is greater than the average profitability for all firm in its
industry. A company is said to have a sustained competitive advantage when it
is able to maintain above-average profitability for a number of years.

It is important to note that in addition to strategies, firms’ profitability is


determined by characteristics of the industry.

Strategic Managers

In most companies there are 2 main types of managers:

o General managers: bear responsibility for the overall performance of the


company or one of its subunits or divisions

o Functional managers: responsible for supervising a particular function


(e.g. accounting, marketing, R&D).

1
Company is a collection of functions or departments that work together to bring
a particular product or service to the market.

Figure on the next page shows the organization of multidivisional company – a


company that competes in several different businesses and has created a
separate, self-contained division to manage each of them.

Corporate level managers

Corporate level of management consists of the CEO (chief executive officer),


other senior executives, the BoD and corporate staff.

The role includes:


o Defining the goals of the organization
o Determining what businesses it should be in
o Allocating resources among different businesses
o Formulating and implementing strategies that span individual businesses
o Providing leadership for the entire organization

Corporate level managers also provide a link between the people who oversee
the strategic development of a firm and those who own it (shareholders) => they
can be viewed as agents of the shareholders. = responsibility to ensure that
corporate business and corporate level strategies are consistent with maximizing
profitability.

Business level managers

Business unit – a self-contained division (with its own function – e.g. finance,
marketing) that provides a product or service for a particular market.

The principal general manager of the business unit is the head of the division.

Functional level managers

Functional managers’ sphere of responsibility is generally confined to one


organisational activity, whereas general managers oversee the operation of a
whole company or division.

The model of the strategic planning process:

1) Select the corporate mission and major corporate goals


2) Analyze the organization’s external competitive environment to identify
opportunities and threats
3) Analyze the organization’s internal operating environment to identify the
organization’s strengths and weaknesses
4) Select strategies that build on organization’s strengths and correct its
weaknesses in order to take advantage of external opportunities and

2
counter external threats. They should be congruent and constitute a
viable business model
5) Implement the strategies
6) Progress Review

1) Each round of the planning process begins with a statement of the


corporate mission and major corporate goals
2) Mission statement is followed by external analysis, internal analysis and
strategic choice.
3) Strategy-making process ends up with the organization of the design of
the organisational structure, culture and control systems necessary to
implement the organization’s chosen strategy.

Functional level strategy is directed at improving the effectiveness of


operations within company, such as manufacturing, marketing, materials
management, product development and customer service.

Business-level strategy encompasses the business’s overall competitive


theme, the way it positions itself in the marketplace to gain competitive
advantage: cost leadership, differentiation, niche/segment focus within the
industry, or a combination of all.

Global strategy is addressing how to expand operations outside the home


country to grow and prosper in a world where competitive advantage is
determined at the global level.

Corporate level strategy what business/businesses should the company be to


maximize the long-run profitability and profit growth of the organization and
how should the company enter and increase the presence of the company in
these businesses to gain a competitive advantages.

Strategic planning is ongoing process. Once strategy has been implemented, its
execution must be monitored to determine the extent to which strategic
goals and objectives are actually being achieved and the extent to which
strategic goals and objectives are actually being achieved and to what degree
competitive advantage is being created and sustained.

Strategy As An Emergent Process

Several scholars have critics an assumption that strategic planning is rational


and highly structured, and orchestrated by top management. Main reasons
for criticism:

 The unpredictability of the real world


 The role that lower-level managers can play in the strategic management
process
 The fact that many successful strategies are a result of serendipity, not
rational strategizing

3
Strategy Making in an Unpredictable World

Critics of formal planning systems argue that we live in highly dynamic,


uncertain and globalized world = premium for adopt quickly

Autonomous Action: Strategy Making by Lower-Level Managers

Autonomous Action is an action taken by lower-level managers who, on their own


initiative, formulate new strategies and work to persuade top-level managers to
alter the strategic priorities of a company

Lower-level managers might be less likely to have the same commitment to


status quo, as top managers might have, and thus might be more keen to support
new technologies and strategies within the firm.

Serendipity and Strategy

Some companies have missed out on profitable opportunities because


serendipitous discoveries or events were inconsistent with their prior (planned)
conception of what their strategy should be.

Intended and Emergent Strategies

Henry Mintzberg’s model of Emergent and Deliberate Strategies:

A company’s realized strategy is the product of whatever planned strategies


are actually put into action (the company’s deliberate strategies) and of any
unplanned, or emergent, strategies. In Mintzberg’s view, many planned
strategies are not implemented due to unpredicted changes in the environment
(unrealized).

Emergent strategies are the unplanned responses to unforeseen


circumstances. They arise from autonomous action by individual managers
deep within organization, from serendipitous discoveries or events, or from an
unplanned strategic shift by top-level managers in response to changed
circumstances = not a product of formal top-down planning mechanisms. =
usually successful and may be more appropriate than intended strategies.

Strategic Planning In Practice

Despite criticism, research suggests that formal planning systems do help


managers make better strategic decision. For strategic planning to work,
however, I is important that top-level managers plan not just in the context of the
current competitive environment but also try to find strategy that will best allow
them to achieve a competitive advantage in the future competitive environment.

4
Scenario Planning

Scenario planning is formulating plans that are based upon realization that
the future is inherently unpredictable, and that an organization should plan a
range of possible futures.

Decentralized Planning

The Ivory Tower approach is treating planning as an exclusively top


management responsibility. This approach can result in strategic plans
formulated in a vacuum by top managers who have little understanding or
appreciation of current operating realities. It can also lead to tensions between
corporate-, business-, and functional-level managers.

Finally Ivory tower planning ignores the important strategic role of


autonomous action by lower-level managers and serendipity.

Strategic Decision Making

One important way in which managers can make better use of their knowledge is
to understand and manage their emotions during the course of decision-making.

Cognitive Biases

They seem to arise from a series of cognitive biases in the way that human
decision makers process information and reach decisions – because of which bad
decisions are still made even when good information is at disposal.

Prior Hypothesis Bias is a cognitive bias that occurs when decision-makers who
have strong prior beliefs tend to make decisions on the basis of these beliefs,
even when presented with evidence that their beliefs are wrong – in this way
CEO might continue to pursues the strategy prior to his belief even if he has
evidence that its falling

Escalating Commitment - a cognitive bias that occurs when decision makers,


having already committed significant resources to a project commit even more
resources after receiving feedback that the project is falling – personal
responsibility induce decision-makers to stick with a project despite evidence
that it is falling

Reasoning by analogy – a cognitive bias that involves the use of simple


analogies to make sense out of complex problems – analogy may not be valid to
solve it.

5
Representativeness – bias rooted in the tendency to generalize from a small
sample or even a single vivid anecdote – dot-com boom when entrepreneurs saw
success of Yahoo and Amazon as definite success for any start up leading to many
going bankrupt.

Illusion of control – a cognitive bias rooted in the tendency to overestimate


one’s ability to control events – tendency to attribute their success in life to their
own good decision making and their failures to bad luck. Popular with having too
much confidence to create value by acquiring another company which usually
fails.

Improving Decision Making

Two techniques known to enhance strategic thinking and counteract groupthink


and cognitive biases:

Devil’s Advocacy – a technique in which one member of a decision making


group acts as a devils advocate, brining out all the considerations that might
make the proposal unacceptable = one member of the decision-making group act
as the devils advocate, bringing out all the reasons that might make the proposal
unacceptable.

Dialectic inquiry – is more complex, for it requires the generation of a plan ( a


thesis) and a counterplan (an antithesis) that reflect plausible but conflicting
courses of action = strategic manager listen to debate between the advocates and
then make the decision on which judgment will lead to higher performance. As a
result of this exercise, strategic managers are able to form a new and more
encompassing conceptualization of the problem, which becomes the final plan

Strategic Leadership

One of the key strategic roles of both general and functional managers is to use
all their knowledge, energy, and enthusiasm to provide strategic leadership for
their subordinates and develop a high-performing organisation.

Key Characteristics of good strategic leaders that lead to high performance:

Vision, eloquence and Consistency – key leadership task is to give sense of


direction and strong leaders have the clear vision of where their organisation
should go and are eloquent enough to communicate their visions to others within
their organisation in terms that energize people and consistently articulate their
visions until they become part of the organisations culture. – Example Bill Gates
with his vision of the world in which there would be a Windows based personal
computer on every desk was a driving force at Microsoft.

6
Commitment – strong leaders demonstrate their commitment to their vision
and business model by actions and words, and they often lead by example. Ken
Iverson former Nucor’s CEO had lowest salary, drove an old car etc.

Being Well Informed – effective strategic leaders develop a network of formal


and informal sources who keep them well informed about what is going on
within their company. For example Herb Kelleher at Southwest Airlines was able
to find out a lot about the health of his company by dropping in unannounced on
aircraft maintenance facilities and helping workers there to perform their tasks.

Willingness to Delegate and Empower – high-performance leaders are skilled


at delegation. They recognize that unless they learn how to delegate effectively
they can quickly become overloaded with responsibilities. They also recognize
that empowering subordinates to make decisions is a good motivation tool.
However, its important to keep the most important decisions for themselves.

The Astute Use of power – effective leaders tend to be very wisw in their use of
power. He argued that strategic leaders must often play the power game with
skill and attempt to build consensus for their ideas rather than use their
authority to force ideas through – act as members or democratic members of
coalition rather than as directors.

Emotional Intelligence – emotional intelligence is a term that describe a bundle


of psychological attributes that many strong and effective leaders exhibit such as:

 Self-awareness.

 Self-regulation

Both help elicit the trust and confidence of subordinates

 Motivation

 Empathy

 Social Skills

7
lOMoARcPSD|6004607

STRATEGIC MANAGEMENT
CHAPTER 2
Governing stakeholders and business ethics
Stakeholders
Successful strategies consider their key constituencies that
impact the functioning and ultimate survival of the company.
Corporate governance - mechanisms to monitor managers
making sure they pursue strategies in the interest of
shareholders.
Stakeholders
◦ Individuals or groups with an interest, claim, or stake in
the company and how well it performs

◦ Anyone in an exchange relationship with the company


Why consider stakeholders?
Stakeholders are key to the
success of company  If their claims are not
considered, stakeholders withdraw
support Stakeholders are in an exchange
relationship with the company, therefore the company
would not function as well if they lost the support
Companies cannot satisfy all stakeholders,
therefore companies must make choices. The impact
analysis helps companies decide what stakeholders
are most critical to survival. Steps of an Impact
Analysis:

◦ Identify stakeholders

◦ Identify their interests and concerns

◦ Identify what claims they are likely to make

Downloaded by Imad Quadri (imad3889@yahoo.co.in)


◦ Identify most important stakeholders

◦ Identify the resulting strategic challenges Most companies


find that customers, employees, and stockholders are
most important stakeholders.

The mission statement Purpose: To establish the guiding


principles for strategic decision making. Includes 4 main
elements:

◦ The Mission

◦ The Vision

◦ Values

◦ Goal of the Corporation The mission statement is a key


indicator of how an organization views the claims of
stakeholders. Describes what the company does.
Can be product oriented:

◦ Focus on the product the company produces


Can be customer oriented: ◦ Focus on satisfying
customers’ needs ◦ Example: Kodak focuses on
providing imaging solutions to
customers, instead of focusing on the products that the
company produces

The mission
Defining the business
Product orientation: • Only a physical manifestation
The need to take a customer-oriented view of a company’s
business has often been ignored. It helps to provide customers
with the solutions they need and assists companies to
capitalize on changes in their environment.
• E.g. Kodak – customer-oriented approach that helps
customers to capture, store, process, output and
communicate image.
The vision
The vision tells what the company would like to achieve, lays
out some desired future state.
Intended to stretch a company by articulating its
ambitions. Meant to be an attainable goal that
will motivate employees.
Values
Values tell how managers and employees should conduct
themselves, how they should do business and what kind of
organization they should build to help a company to achieve its
mission.
Values establish the basis of the organizational culture:
◦ Organizational culture is the set of values, norms, and
standards that control how employees work to achieve
an organization’s mission and goals.

◦ Organizational culture is an important source of competitive


advantage. Major goals Having stated the mission,
vision and key values, strategic managers an take the
next step in the formulation of a mission statement:
establishing major goals. Goal – is a precise and
measurable desired future state that a company attempts
to realize. Characteristics of well-constructed goals:
Precise and measurable Address crucial
issues Challenging but realistic Specify a time
period

Well-constructed goals also provide a means by which the


performance of managers can be evaluated.
Most companies operate with goals of profitability and profit
growth (as central goal of most organizations is to maximize
shareholders’ values). Problems arise when managers
overemphasize profit growth.
Some companies tend to cut costs to increase short-term
profits, but the cuts can be detrimental to the company in the
long run.
It is worth noting that pressures to maximize short-term
profitability may result in managers' acting in an unethical
manner (e.g. Enron in 1990s).
Corporate governance and strategy
Stockholders are the legal owners of a company and the
providers of risk capital.
The capital that stockholders provide is seen as risk capital -
stockholder equity with no guarantee of profit or even
recoupment.
Agency relationship - whenever one party delegates decision-
making authority or control over resources to another person.
Stockholders delegate control to agents Stockholders’
agents = Managers
Agency problem
As agents for stockholders, managers should pursue strategies
that maximize long run return for stockholders. However,
managers sometimes pursue strategies that are not in the
interest of stockholders.
Agency Theory offers an explanation for this behavior.
Information asymmetry exists between the agent and the
principal, because the agents almost always have more
information.
To an extent, principal has to trust the agent to do the right
thing. This trust is not blind: principals do put governance
mechanisms in place whose purpose is to monitor agents,
evaluate their performance, and if necessary, take corrective
action.
Confronted with agency problems, the challenge for principals
is to:
฀Shape the behavior of agents so that they act in accordance
with the goals set by principles

฀Reduce the information asymmetry between agents and


principals

• Develop mechanisms for removing agents who do not act


in accordance with the goals of principals, or mislead
principals
Monitoring managers – Governance mechanisms
Purpose is to monitor agents, evaluate their performance, and
take corrective action to reduce the scope and frequency of
agency problems. Goal is to align stockholder and management
interests.
1. The board of directors
Board of directors - members elected by shareholders to make
sure business decisions are in the shareholder’s best interest.
฀It can apply sanctions, such as voting against management
nominations to the board of directors or submitting its
own nominees. In addition, the board has legal authority
to hire, fire, and compensate corporate employees,
including, most importantly, the CEO.

฀ The board is also responsible for making sure that audited


financial statements of the company present a true
picture of its financial situation. The typical board of
directors is composed of a mix of inside and outside
directors.

฀ Inside directors – senior employees of the company, such as


CEO. They are required for the board because they
have valuable information about the company’s
activities.

฀ Outside directors – not full-time employees of the company.


Many of them are full-time professional directors who
hold positions on the boards of several companies.
Outside directors are needed to bring objectivity to the
monitoring and evaluating processes. Inside directors
often dominate the outsiders on the board because of
their superior knowledge and control over information
are sources of power; they consequently may be better
positioned than outsiders to influence boardroom
decision making. In the aftermath of a wave of corporate
scandals in early 2000s, BoD are beginning to play a much
more active role in corporate governance.

• E.g. growing trend on the part of the courts to hold


directors liable for corporate misstatements.
2. Stock-based compensation
Stock-based compensation - pay-for-performance systems to
reward ethical managers. = long-run profitability.
Some research studies suggest that stock-based compensation
schemes for executives, such as stock options, can align
management and stockholder interests.
3. Financial statements and auditors
Financial statements - reports that give consistent, detailed,
and accurate information about how efficiently and effectively
the company is run.
In the US – all publicly traded companies are required to file
reports according to GAAT (generally agreed accounting
principles). To make sure that they are adequate audits would
be done by independent and accredited accounting firms.
BUT: e.g. in the US the system has not been working as
intended => Enron’s auditor – Arthur Anderson.
4. The takeover constraint
The takeover constraint - the risk of being taken over by
another company. The takeover constraint limits the extent to
which managers can pursue strategies to actions that put their
own interests above those of stakeholders. If they ignore
stakeholder interests and the company is acquired, senior
managers typically lose their independence and probably their
jobs as well.

Ethics and strategy


Ethics – refers to accepted principles of right and wrong
that govern the conduct of a person, the members of a
profession, or the actions of an organization.

Business ethics – accepted principles or right and wrong


governing the conduct of business people.
Ethical dilemmas – situations where there is no agreement
over exactly what the accepted principles of right and wrong
are, or where none of the available alternatives seems ethically
acceptable.
It is important to note, however, that behaving ethically goes
beyond staying within the bounds of the law (e.g. in 1990s
managers at Nike contracted out the production of sports
shoes to producers in the developing world; Nike did not
break laws, but there were ethical issues with regard to
‘sweatshop labor’.
.
Ethical Issues:
◦ Self Dealing (occurs when managers find a way to feather
their own nests with corporate money, e.g. Tyco)

◦ Information Manipulation (occurs when managers


use their control over corporate data to distort or
hide information in order to enhance their own
financial situation, or the competitive position of the
firm)

◦ Anti-Competitive Behavior (covers a range of actions aimed


at harming actual or potential competitors, most often by
using monopoly power, thereby enhancing the long-run
prospects of the firm)
◦ Opportunistic Exploitation (occurs when the managers of a
firm seek to unilaterally rewrite the terms of a contract
with suppliers, buyers, or competent providers in a way
that is more favorable to the firm, often using their
power to force the revision through).

◦ Environmental Degradation (occurs when a firm takes


actions that directly or indirectly result in pollution
or other forms of environmental harm).

◦ Corruption (arises in a business context when managers


pay bribes to gain access to lucrative (profitable)
business contracts.

◦ Substandard working conditions (occurs when managers


underinvest in working conditions, or pay employees below
market rates, in order to reduce their costs of production).
The roots of unethical behavior
Business ethics are not divorced from personal ethics.
A lack of separation between one’s business
ethics and personal ethics
Failure to ask relevant questions
Purely economical organizational culture
Pressure from top managers
Unethical behavior by company leaders. It
is not what leaders preach that matters most, rather the
example they set by their actions. Behaving ethically
Hiring and promotions: the act of hiring employees
whose ethical principles are in line with the company’s
ethics.
Organizational culture: the culture within the business that
should explicitly articulate ethical values and place a
strong emphasis on ethical behavior.
Organizational leadership: once these values are
explicated, it is important that leaders give life and
meaning to these words by acting upon them.
Ethical officers: individuals responsible for making sure
employees are trained to be ethically aware and that
ethical considerations enter the business decision making
process.
Strong corporate governance: rules that are established to
make sure that managers adhere to ethical norms, and in
particular, to make sure that senior managers do not
engage in self-dealing or information manipulation.
Moral courage: courage that enables managers to walk away
from a decision that is profitable, but unethical.
Decision making processes: business people must be able
to think through the ethical implications of decisions in a
systematic way. Moral compasses should be in place –
ethical algorithms.
CHAPTER 3
INDENTIFYING OPPORTUNITIES AND THREATS THROUGH
EXTERNAL ANALYSIS
The starting point of strategy formulation is an analysis of
the forces that shape competition in the industry in which a
company is based.
Opportunities – arise when a company can take advantage of
conditions in its environment to formulate and implement
strategies that enable it to become more profitable.
Threats – arise when conditions in the external environment
endanger the integrity and profitability of the company’s
business.
Analyzing industry structure Industry - a group of
companies offering products or services that are
close substitutes for each other.
Company’s closest competitors – are those that serve the
same basic customer needs.
The starting point of external analysis is to identify the
industry that a company competes in. To do this, managers
must begin by looking at the basic customer needs their
company is serving, they must take customer-oriented view.
 It is highly important to define boundaries of
industry correctly.
Once boundaries are defined, the task facing managers is to
analyze competitive forces in the industry environment to
identify opportunities and threats.
Porters 5 forces
Porter argues that the stronger each of these forces, the more
limited the ability of established companies to raise prices and
earn greater profits.
Risk of entry by potential competitors
Potential competitors – are companies that are not currently
competing in an industry but have the capability to do so if
they choose.
Established companies already operating in an industry often
attempt to discourage potential competitors from entering the
industry because the more companies enter, the more difficult
it becomes for established companies to protect their share of
the market and generate profits.
The risk of entry by potential competitors is a function if the
height of barriers to entry – factors that make it costly for
companies to enter an industry.
 High barriers to entry may keep potential competitors
out of an industry. Economies of scale Arise when unit costs
fall as a firm expands its output.

Sources of economies of scale:


฀Cost reductions gained through mass-producing a
standardized output

฀Discountson bulk purchases of raw material inputs and


component parts

฀ Advantages gained by spreading fixed production costs over


a large production volume

฀Cost savings associated with spreading marketing and


advertisement costs over a large volume of output.
Brand loyalty Exists when consumers have a preference
for the products of established companies. A company
can create brand loyalty through:

฀continuous advertisement

฀ patent protection of products

฀ productinnovation achieved through company research and


development programs

฀emphasis on high product quality

฀ good after sales service Absolute cost advantages Absolute


cost advantage - a cost advantage that is enjoyed by
incumbents in an industry and that new entrants cannot
expect to match. They arise from 3 main sources:

฀Superior production operations and processes due to


accumulated experience in an industry, patents, or secret
processes

฀Control of particular inputs required for production, such as


labour, materials, equipment or management skills, that
are limited in their supply

฀ Accessto cheaper funds because exiting companies represent


lower risks than new entrants Customer switching costs
Switching costs arise when it costs a customer time,
energy and money to switch from the products offered by
one established company.

 Faced with high expense of money and time, most


people are unwilling to make the switch unless
competing
product offers a substantial step forward in performance.

Government regulation
฀Historically, government regulation has constituted a major
entry barrier into many industries.

฀Nowadays, institutional context matters in certain industries.

Rivalry among established companies Rivalry refers to the


competitive struggle between companies in an industry to gain
market share from each other. Companies struggle can be
fought using:

฀Price

฀Product design

฀ Advertising and promotion spending

฀Direct selling efforts

฀ After-sales service and support The intensity of rivalry


among existing firms within an industry is largely a
function of 4 factors:

1. Industry competitive structure


Competitive structure of an industry refers to the number and
size distribution of companies in it.
฀ Fragmented industry – consists of a large number of small-
and medium- sized companies, none of which is in a
position to determine industry price.

฀ Consolidated industry – is dominated by a small number of


large companies (an oligopoly), or in extreme cases, by
just one company (a monopoly) in which companies often
are in a position to determine industry prices. Many
fragmented industries are characterized by low entry
barriers and commodity-type products that are hard to
differentiate.

฀ Often flood of new entrants into booming fragmented


industries creates excess capacity and as a result –
price war.

฀ A fragmented industry structure, then, constitutes


a threat rather than an opportunity.

In consolidated industries, companies are interdependent,


because one company’s competitive actions or moves directly
affect the market share of its rivals, and thus their profitability.
฀Risk of dangerous competitive spiral

฀Price fixing is illegal; instead companies set prices by


watching, interpreting, anticipating, and responding to
each other’s behavior. Industry demand Growing
demand from new customers or additional purchases
by exiting customers tend to reasonable competition by
providing greater scope for companies to compete for
customers.  Growing demand tends to reduce
rivalry because all companies can sell more without
taking market share away from other companies.
 Declining demand constitutes a major threat for it
increases the extent between established companies.
Cost conditions In industries where fixed costs are high,
profitability tends to be highly leveraged to sales volume,
and the desire to grow volume can spark off intense
rivalry.

• When fixed costs are high and sales volume is not


sufficient to cover costs, it will lead to intense rivalry.
Exit barriers
Exit barriers – economic, strategic, and emotional factors that
prevent companies from leaving an industry.
If exit barriers are high, companies become locked into an
unprofitable industry where overall demand is static or
declining.
 Result is often excess capacity and intensified rivalry.
Common exit barriers:
฀Investment in assets such as specific machines, equipment,
and operating facilities when it cannot be sold or used
alternatively

฀High fixed costs of exit, such as pensions paid to workers

฀Emotional attachments to the industry

฀Economic dependence on an industry because a company


relies on a single industry for its revenue and profit

• Bankruptcy regulations The bargaining power of buyers


An industry buyers may be the individual customers who
ultimately consume its products or the companies that
distribute an industry’s products to end users, such
retailers and wholesalers.
Bargaining power of buyers – refers to the ability of buyers to
bargain down prices charged by companies in the industry
or to raise the costs of companies in the industry by
demanding better product quality and service.
 Powerful buyers should be viewed as a threat.
Buyers are most powerful in the following circumstances:
฀ When buyers are large and few in numbers, while suppliers
are big in numbers

฀ When buyers purchase is big in quantities

฀ When switching costs are low

฀ When buyers can threaten to enter the industry and produce


the product themselves The bargaining power of
suppliers Bargaining power of suppliers – refers to the
ability of suppliers to raise input prices, or to raise the
costs of the industry in other ways.  Powerful
suppliers are a threat. Suppliers are most powerful in
the following situations:

฀ The product that suppliers sell has few substitutes and is


vital to the companies in an industry.

฀ The profitability of suppliers is not significantly affected by


the purchases of companies in a particular industry.

฀Companies in an industry would experience significant


switching costs if they moved to the product of a different
supplier.

฀Suppliers can threaten to enter their customers industry.


Substitute products Substitute products – products of
different businesses and industries that can satisfy
similar customer needs (e.g. companies in coffee industry
compete indirectly with tea companies).

 The existence of close substitutes is a strong competitive


threat. Strategic groups within industries
Strategic groups - groups of companies within an industry
that follow a similar strategy (e.g. high risk, high return
strategy).
The closest competitors for a company are those in its strategic
group. The most immediate threat to profitability comes from
rivals within the strategic group.
Each strategic group may face a different set of opportunities
and threats.
Varied threats include:
◦ Risk of new entry

◦ Degree of rivalry

◦ Bargaining power of buyers and sellers

◦ Competitive force of substitute and complementary


goods Moving from one strategic group to another
may be
difficult or impossible. Mobility Barriers: Within-
industry factors that inhibit movement between
strategic groups (e.g. forest Labs entering the
pharmaceutical industry: they would encounter barriers
because they lack resources and competencies necessary
to compete).
Pharmaceutical industry:

Industry life cycle analysis


The industry life cycle is important in analyzing the strength of
competition in an industry.
Competition increases as the industry progresses
through the cycle.
Embryonic:
◦ The industry is just beginning to develop

◦ Development is slow as buyers are unfamiliar with product

◦ High prices due to absence of scale economies

◦ Rivalry based not on prices rather on educating customers

◦ Competition might be intense

Growth:

◦ Demand is expanding rapidly

◦ Many new customers

◦ Prices fall with development and higher volume


(scale economies)

◦ Entry barriers are relatively low (as insignificant


scale economies and brand loyalty)

◦ Relatively low competition (as industry is growing new


competitors might be absorbed into industry
without
much loss to profitability of others)

Shakeout:
◦ Rate of growth slows
◦ Demand approaches saturation levels

◦ Few potential first-time buyers

◦ Rivalries become intense

◦ Excess capacity may exist

Mature:

◦ Market is totally saturated

◦ Demand is limited to replacement demand

◦ Growth is low or zero

◦ Barriers increase

◦ Threat of new entries decrease

◦ Competition drives prices down, often resulting in price


wars (e.g. airline industry)

◦ Most industries become consolidated

Decline:

◦ Falling demand = Excess capacity

◦ Growth becomes negative due to: Technology


substitution Demographics 
International competition
The macroenvironment Macroenvironment - the broader
economic, global, technological, demographic,
social, and political contexts of business.
The macro-environment impacts the strength of forces in
Porter’s model and ultimately, the attractiveness of the
industry.
It is important for managers to pay close attention to external
forces on their industry due to their direct impact.
Macroeconomic forces
◦ Forces at the national or regional level

◦ Most important forces to monitor are:  growth


rate of the economy  interest rate 
currency exchange rates  price inflation

Global forces
◦ Many countries experiencing economic growth since
barriers to international trade have tumbled.

◦ Growth in places like Brazil, China, and India is creating


large new markets for goods and services.
Technological
forces

◦ Technological changes are destructive to some companies


(threats) and creative for others (opportunities.)

◦ They can affect the height of barriers of entry and reshape


an industry. Demographic forces

◦ Outcomes of change in characteristics of the population


such as:  Age - Gender  Ethnic
origin
Social class Race Sexual orientation

◦ Currently there is a growing aging population, which is an


opportunity for organizations to cater to an older age
group. Social forces

◦ Changing social mores and values affect industry, such as the


trend toward health conscientiousness that causes customers
to pay attention to different characteristics of products.
Political and legal forces
◦ Changes in laws and regulations that impact managers
and companies.

◦ Result from political and legal developments within


society and significantly affect businesses.

◦ Firms and industries strive to influence the regulations that


government enacts.
lOMoARcPSD|6004607

Strategic Management
Chapter 4
Building Competitive Advantage
Two basic conditions determine a company’s profitability:
 Amount of value customers place on the company’s goods or services
 Value creation is at the heart of competitive advantage
 The greater the value customers place on a product, the more the company can charge.
 A product’s price is usually less than the value placed on it by the average customer.
 This causes customers to capture consumer surplus.

 Company’s costs of production


A company will look for ways to increase productivity of capital and labor through:
 Economies of Scale
 Spread fixed cost over large product volume
 Greater division of labor and specialization

Profit margin is equal to P-C, where P is price, C is Cost

Company makes profit as long as P > C, and its profit rate will be greater the lower is C relative to P

Difference between V (value per customer) is in part determined by the intensity of competitive pressure in
the market place. The lower the intensity of competitive pressure, the higher the price that can be charged
relative to V2

A Company can create more value for customers either by lowering C or by making product more attractive
through superior design, functionality, quality etc, so that customers place greater value on it (V increases)
and thus are willing to pay higher price (P increases)

The concept of value creation lies at the heart of competitive advantage

 Competitive Advantage occurs when a company’s profitability is greater than the industry’s average
profitability
 Competitive advantage over several years is considered Sustained

Resource-Based View of Competitive Advantage


 Suggests that competitive advantage is attained through firm resources that are:
◦ Valuable - they enable a firm to implement strategies that improve its efficiency and
effectiveness
◦ Rare - not available to other competitors
◦ Imperfectly imitable - not easily implemented by others
◦ Non-substitutable - not able to be replaced by some other non-rare resource

Superior value creation require that the gap between perceived value V and costs of production C be greater
than the gap attained by competitors

According to Porter, competitive advantage (and higher profitability) goes to those companies that can
create superior value by driving down costs of production and/or differentiate the product in some way so
that consumers value it more and are prepared to pay a premium price.

Downloaded by Imad Quadri (imad3889@yahoo.co.in)


The Competitive Building Blocks Of Competitive Advantage

 Four Factors:
◦ Efficiency
◦ Quality
◦ Innovation
◦ Customer Responsiveness
These are the generic building blocks of competitive advantage that any company in any industry can adopt.

Efficiency

Efficiency is the quantity of inputs that it takes to produce a given output.

Efficiency = outputs/inputs. The more efficient a company is, the fewer inputs are required to produce a
given output.

The most important components of efficiency for many companies are employee productivity (output per
employee) and capital productivity (output per unit of invested capital)

Quality as Excellence and Reliability

From a Quality as Excellence perspective, the important attributes are things such as a product’s design and
styling, features and functions, and level of service associated with the delivery of a product. When
excellence is built into product offering, consumers have to pay more to own or consume the product

Quality as Reliability

 A product is reliable if it:


◦ Consistently does the job it was designed for
◦ Does the job well
◦ Rarely, if ever, breaks down
 Less time is spent of defective products and fixing mistakes
Reliability increases the value a consumer gets from the product and increases the price that the company
can charge

Providing high quality products increases customers’ perceived value and allows Bs to charge higher prices;
also, the more products are reliable, the greater efficiency and lower unit costs.

Innovation
 The act of creating new products or processes
◦ Innovations give a company something unique that their competitors lack
◦ In the long run, innovation is perhaps the most important part of competitive advantage
 Product Innovation- The development of products that are new to the world or have superior
attributes to existing products.
 Process Innovation-The development of a new process for producing products and delivering them.

Customer Responsiveness
 If a customer’s need is satisfied better by a certain product, the customer will attribute more value to
the product.
 More value creates a differentiation and ultimately a competitive advantage
Customer Response Time- that time it takes for a good to be delivered or a service to be performed.

Functional Strategies and The Generic Building Blocks of Competitive Advantage

Increasing Efficiency
 By simplifying the design of a product, R&D can dramatically decrease the required
assembly time, which translates into higher employee productivity, lower costs, and
higher profitability
 Economies of scale: a major source of EoS is the ability to spread FC over a production
volume; another source is Specialization, which is said to positively impact
productivity because employees become more skilled at performing particular task
 Learning Effects- costs savings that come from learning by doing. Labor productivity
increases over time, and unit costs fall as individuals learn the most efficient way to
perform a particular task
 Lean Production- a range of manufacturing technologies designed to reduce setup times for
complex equipment, increase the use of individual machines through better scheduling, and
improve quality control at all stages of the manufacturing process
 Mass Customization- the ability of companies to use flexible manufacturing technology to
customize output at costs normally associated with mass production
 Marketing Strategy- the position that a company takes with regard to pricing, promotion,
advertising, product design and distribution
 Customer Defection Rate- the percentage of a company’s customers who defect every year to
competitors
 Material Management and Efficiency- for a typical manufacturing company, materials and
transportation costs account for 50-70% of its revenues, so even small reduction in these
costs can have a substantial impact on profitability
 With JIT- major cost saving comes from increasing inventory turnover, which reduces
inventory holding costs, such as warehousing and storage costs, and the company’s need for
working capital
 HR and Efficiency/ Self-Managing Team- a team wherein members coordinate their
own activities
 The effect of introducing self-managing teams- increase in productivity of 30% or more and
also a substantial increase in product quality
 Infrastructure- can foster companywide commitment to efficiency and promote cooperation
among departments

Increasing Quality
Improved quality means:
 Decreased costs due better use of time and materials=> productivity improves=>
better quality leads to higher market share and allows the company to raise prices=>
increase in profitability

In order to improve quality management needs to identify defect, from which process it arises, what causes
this defect, and after, management should make correction

One key to quality improvement is to create a metric to measure quality.

It is important to identify which product attributes are the most valuable to customers and then to design
products and associated services to satisfy customer needs. Also it is essential to remember that competition
also produces continual improvement
Increasing Innovation
Research suggests that only 10-20% of major R&D projects give rise to commercial products.
Reasons for Innovation failures:
 Demand for innovation is inherently uncertain
 Technology is poorly commercialized
 Poor positioning strategy. Positioning Strategy- the specific set of option a company adopts for a
product on four main dimensions: price, distribution, advertising, promotion and product features
Tight cross-functional integration can help a company to successfully implement product innovation:
 Market research=>better understand customer needs
 Design hat allows ease of manufacture
 Control of costs
 Time to market is minimized
Management can form teams of employees form key departments. Team members should have an ability to
contribute functional expertise and willingness to share responsibility for team results and an ability to put
functional advocacy aside. Team members should be co-located and have a clear plan and set of goals

Heavyweight Project Manager-a project manager who has status within the organization and the power to
and authority required to get the financial and HR that a team needs to succeed.

Increasing Customer Responsiveness

Superior efficiency, quality, innovation are all part of achieving superior customer responsiveness.

Product can be differentiated by customization, which refers to varying the features of a good or service to
tailor to unique needs and tastes of groups of customers; and reducing the time it takes to respond to or
satisfy customer needs.
The Value Chain

 Idea that a company is a chain of activities for transforming inputs into outputs that customers value
 Consists of primary and support activities

Primary Activities

Primary Activities are those related to the design, creation, and delivery of the product, its marketing, and its
support activities.
 Research and Development
 Production
 Marketing and Sales- Through brand positioning and advertising, the marketing function can
increase the value customers perceive to be contained in a company’s product
 Customer Service- after-sales service and support

Support Activities
 Provide inputs for primary activities
◦ Material Management (logistics)- controls the transmission of physical materials through the
value chain, form procurement through production into distribution
◦ Human Resources
◦ Information Systems-electronic systems for managing inventory, tracking sales, products,
dealing with customer service inquires
◦ Company Infrastructure – the companywide context within which all the other value creation
activities take place: the organizational structure, control systems, and company culture
Distinctive Competencies and Competitive Advantage

 Distinctive Competency is a unique firm-specific strength that allows a company to better


differentiate or achieve lower cost than rivals
 Arise from Resources and Capabilities

 Resources- Financial, physical, social or human, technological, and organizational factors that create
value to customers.
 Can be Tangible or Intangible
Example:
Tangible = land, building, equipment
Intangible = brand names, reputations

 Capabilities- a company’s skills at coordinating its resources and putting them into productive use.
 Capacities are needed to utilize resources
A company's distinctive competency is the strongest when it possesses both firm specific and valuable
resources and firm specific capabilities to manage those resources

Durability

 When a company has superior profitability, other companies are inclined to imitate the successful
strategy.
 Durability of competitive advantage depends on the ease or difficulty to copy distinctive
competencies.
 Barriers of Imitation are the factors that make copying difficult for competitors.
 Even if a company’s distinctive competencies are protected by higher barriers to imitation, it should
act as if rivals are continually trying to nullify its source of advantage
Strategic Management
Chapter 5
Positioning for Competitive Business-Level Strategy

Business-Level Strategy- The plan of action that managers adopt to use resources and distinctive
competencies to gain a competitive advantage

 Basis of choosing a business level strategy by determining how well a company can compete
◦ What customer need will be satisfied?
◦ Who is to be satisfied?
◦ How will the need be satisfied?

Customer Needs- Desires, wants, or cravings that can be satisfied by means of the characteristics of a
product or service

Product Differentiation- the process of creating a competitive advantage by designing goods or services to
satisfy customers needs. The greater the differentiation, the more money a customer will pay for the product

Customer Groups
Market Segmentation- the way a company decides to group customers, based on important differences in
their needs or preferences

Alternatives to Market Segmentation:


 Choose not to recognize different needs; just aim to serve the average customer.
 Separate markets and create a product to suit each group.
 Concentrate on serving only one segment.

Distinctive Competencies:
 Decide which distinctive competencies to pursue to satisfy customers
 Decide how to organize and combine distinctive competencies to gain a competitive advantage

Choosing a Business-Level Strategy

 Three basic competitive approaches:


◦ Cost Leadership- to outperform competitors by doing everything it can to produce goods or
services at the lowest possible cost.
◦ Differentiation- the differentiated product has the ability to satisfy a customer’s need in a
way that competitors cannot.
◦ Focus- directed toward serving the needs of a limited customer group or segment

Cost-Leadership Strategy

 Goal: Outperform competitors by increasing efficiency at a lower cost


 Cost leader chooses low level of differentiation
 Positions the product to appeal to the average customer
 Does not try to be the industry leader in differentiation; it waits until customers want a feature or
service before providing it
 Usually ignores market segments; engages in only a limited amount of market segmentation, lower
price attracts wider target population

Advantages:
Charge lower price than competitors but make the same level of profit
Withstand competition based on price
Lower costs=>less affected by powerful suppliers and buyers
Large quantity of production=> increasing bargaining power over suppliers
Can reduce price in case substitute products start to enter the market=> barriers to entry

Disadvantages:
Easy to lose sight of changes in customers’ taste
Competitors will try to beat the cost leader at its own game by finding ways of lowering prices such as
imitating the processes. E.g. ability of Dell’s major competitors HP, Acer and Lenovo to imitate Dell’s low-
cost materials management practices has eroded its competitive advantage and Dell is struggling to find new
ways to compete

Differentiation Strategy

Differentiation Strategy- a strategy of trying to achieve a competitive advantage by creating a product that is
perceived by customers as unique in some important way

Broad Differentiator-a company that offers a product designed for each market niche.

 Goal: To achieve a competitive advantage by creating a product that customers perceive as unique in
some important way
 A differentiated company can charge a premium price based on psychological appeal to customers
e.g. status and prestige; innovation, customer responsiveness and quality

Advantages
Customers develop brand loyalty for a product
Less likely to be put under pressure of powerful buyer because company offers unique product and also
moderate price increases of components can be tolerated
Differentiation creates barriers to entry for other companies

Disadvantages
Difficult to maintain uniqueness in the customer’s eye
Threat of substitute products

Cost Leadership and Differentiation

 Flexible manufacturing strategies make the choice between these two strategies less clear-cut
 The new flexible manufacturing technologies makes diversification inexpensive for firms, allowing
firms to obtain benefits of both strategies
 E.g. expanding markets=> differentiated producer can also enjoy economies of scale e.g. iPhone

Focus Strategy
Focus Strategy-a strategy of serving the needs of one or a few customer groups or segments

 Goal: To serve the needs of a limited customer group or segment


 Concentrate on serving a:
◦ Geographic area
◦ Type of customer
◦ Segment of the product line
Advantages
Customer loyalty lessens the threat of substitutes (high barriers to entry created by product uniqueness) and
of powerful buyers
Power over buyers because they cannot get the same product elsewhere

Disadvantages
Suppliers have power over focused firms, making the firms vulnerable to changes
Vulnerable to attack, therefore must define its niche constantly e.g. Ford’s new luxury cars are aimed to
compete within BMW, Mercedes-Benz and Audi
Higher costs involved that might reduce profitability

Stuck in the Middle


Stuck in the Middle – a problem which occurs when company’s strategy fails because it has made
product/market choices in a way that does not lead to a sustained competitive advantage

Competitive Positioning In Different Industry Environments

Strategies in Fragmented and Growing Industries

 Focus strategy stands out as the best choice through:


◦ Chaining allows cost advantage and amazing buying power to promote competitive
advantage e.g. Wal-Mart and Midas International uses this strategy. They establish networks
of linked merchandising outlets that are so interconnected that they function as one large
entity
◦ Franchising solves the problem of maintaining control over each location and retaining
uniqueness.
◦ Horizontal Mergers consolidate an industry to secure a market.
◦ Using the Internet consolidates fragmented industries globally.

Strategy in Mature Industries

 In a mature industry it is crucial to adopt a strategy that will simultaneously preserve competitive
advantages while preserving industry profitability
 Interdependent companies adopt strategies to:
◦ Manage rivalry
◦ Deter entry

Strategies to Deter Entry in Mature Industries

Product Proliferation- a strategy in which leading companies in an industry all make a product in each
segment or niche and compete head-to-head for customers.
This creates a barrier to entry because potential competitors now find it harder to break into an industry in
which all niches are filled. E.g. Toyota

Price Cutting- Most evidence suggests that companies first skim the market and charge high prices during
the growth stage, maximizing short-run profits. Then they can more to increase their market share and
charge a lower price to expand the market rapidly; develop a reputation; and obtain economies of scale,
driving down costs and barring entry. As competitors do enter, incumbent companies reduce prices to retard
entry and give up market share to create a stable industry context- one in which they can use nonprice
competitive tactics, such as product differentiation, to maximize long-run profits.

Maintaining Excess Capacity- producing more of a product that customers currently demand
Strategies to Manage rivalry in Mature Industries

Price Signaling- the process by which companies increase or decrease product prices to convey their
competitive intentions to other companies. Then companies move to Tit-for-Tat strategy if one company
starts to cut prices more aggressively

Tit-for-Tat Strategy- a form of market signaling in which one company starts to cut prices aggressively, and
then competitors respond in a similar way; when this occurs, nobody gains

Price Leadership- the process by which one company informally takes the responsibility for setting industry
prices. Although price leadership can stabilize industry relationships by preventing head-to-head
competition and thus raise profitability within an industry, it ha s a problem of helping companies with
higher costs by allowing them to survive without becoming more productive and more efficient, this can be
dangerous in the long run.

Market Penetration- a strategy in which a company concentrates on expanding market share in its existing
product markets. Involves heavy advertising to promote and build product differentiation. In consumer
goods industries market penetration becomes a way of life for many products (e.g. diapers and deodorants)

Product Development- a strategy involving the constant creation of new or improved products to replace
existing ones

Market Development- a strategy involving a search for a new market segments, and therefore new uses, for a
company’s products

Product Proliferation- a strategy in which leading companies in an industry all make a product in each
segment or niche and compete head-to-head for customers.
This creates a barrier to entry because potential competitors now find it harder to break into an industry in
which all niches are filled. E.g. Toyota

Strategies in Declining Industries


 When market is shrinking, competition tends to intensify and profit rates tend to fall
 Strategies to deal with decline include:
◦ Leadership Strategy- involves a company seeking to become the dominant player in the
industry
◦ Niche Strategy- focuses on pockets of demands that are declining more slowly
◦ Harvest Strategy- optimizes cash flow for a short period of time
◦ Divestment Strategy- occurs when a company sells off its business to others

Strategy Selection in a Declining Industry

Leadership Strategy- a strategy through which a company seeks to become the dominant player in a
declining industry via aggressive advertising and marketing to build market share; acquiring established
companies to consolidate the industry

Niche Strategy- the strategy of focusing on pockets of demand that are declining more slowly than demand
in the industry as a whole

Harvest Strategy- a strategy that optimizes cash flow; requires to cut all investments on R&D, advertising
etc. Company will be most likely to loose market share but it might improve cash flow, however, ultimately
cash flow will decline.
Divestment Strategy- a strategy in which a company sells off its business assets and resources to other
companies. The success of the strategy depends on the ability of a company to foresee industry’s decline
before it becomes too serious and thus to sell out while the company’s assets are still valued by others.

Using SWOT Analysis to Understand Whether a Given Industry is Viable for New Entry
 Identify
◦ Strengths
◦ Weaknesses
◦ Opportunities
◦ Threats

◦ How can we Use each Strength?


◦ How can we Stop each Weakness?
◦ How can we Exploit each Opportunity?
◦ How can we Defend against each Threat?
Strategic Management
Chapter 6
Global Strategy

The Global Environment

Fifty years ago most national markets were isolated from each other by significant
barriers to international trade and investment – managers could only focus on
analyzing just those national markets in which their companies competed. They did
not need to pay much attention to entry by global competitors, as there were few and
entry was difficult + no attention was paid to foreign markets at it was often
expensive. However, from now it has changed with the rise of many Multinationals
i.e. MacDonald’s etc. intensifying rivalry from industry to industry. Now we are
going to look at different ways of how global expansion can benefit companies.

Increasing Profitability Through Global Expansion

There are number of ways in which expanding globally can enable companies to
increase their profitability and grow their profits more rapidly. At the most basic
level, global expansion increase the size of the market a company is addressing
thereby boosting profit growth. Moreover, global expansion offers opportunities for
reducing the cost structure of the enterprise, or adding value through differentiation,
thereby potentially boosting profitability.

Expanding the Market: Leveraging Products and Competencies

A company can increase its growth by taking goods or services developed at home
and selling them internationally – most multinationals do so. Procter and Gamble, for
example, developed most of its best selling products at home, and then sold them
around the World. The returns from such a strategy are likely to be greater if
indigenous (homegrown) competitors in the nations a company enters lack
comparable products. For e.g. Toyota has grown its profits by entering the large
automobile markets of North America and Europe, offering products that are
differentiated from those offered by local rivals

It is important to note that the success of many multinational companies is based not
just upon the goods and services that they sell in foreign nations, but also upon the
distinctive competencies that underlie the production and marketing of those goods
and services. Pushing this further, one could say that since distinctive competencies
are in essence the most valuable aspects of a company’s business, successful global
expansion by manufacturing companies like Toyota and P&G was based upon their
ability to take their distinctive competencies and apply them to foreign markets.

The same can be said of companies engaged in the services sectors of an economy –
such as financial institutions, retailers, restaurant chains and hotels. Expanding the
market for their services often means replicating their basis business model in foreign
nations – Starbucks is expanding rapidly outside of the United States by taking the
basic business model it developed at home.

Realizing Economies of Scale

In addition to above company can expand its sales volume through international
expansion a company can realize cost savings from economies of scale, thereby
boosting profitability – scale economies come from several sources. First, by
spreading the fixed costs associated with developing a product and setting up
production facilities over its global sales volume, a company can lower its average
unit cost. – Microsoft garner significant economies of scale by spreading the $5
billion it cost to develop Windows Vista over global demand.

Second, by serving a global market, a company can potentially utilize its products
facilities more intensively, which leads to higher productivity, lower costs and greater
profitability – Intel able to produce microchips 7 days a week in 2 shifts by expanding
globally in different markets = Capital invested is used more intensively. Third, as
global sales increase the size of the enterprise, so its bargaining power with suppliers
increases, which may allow it to bargain down the cost of key inputs and boost
profitability.

Realizing Location Economies

We already discussed how countries differ from each other along a number of
dimensions, including differences in the cost and quality of factors of production.
These differences imply that some locations are more suited than others for producing
certain goods and services. Location economies – are the economic benefits that arise
from performing a value creation activity in the optimal location for that activity,
wherever in the world that might be. Locating a value creation activity in the optimal
location for that activity cab have one of two effects: (1) it can lower the costs of
value creation, helping the company achieve a low-cost position, (2) it can enable a
company to differentiate its product offering, which gives it the option of charging a
premium price or keeping price low and using differentiation as a means of increasing
sales volume. Thus, efforts to realize location economies are consistent with the
business-level strategies of low cost and differentiation. For example IMB think pad –
designed in US, keyboard in South Korea, wireless card built in Malaysia.

Leveraging the Skills of Global Subsidiaries

Initially, many multinationals companies develop the valuable competencies and


skills that underpin their business in their home nation then expand internationally,
primarily by selling product and services based on those competencies. Thus,
Walmart honed its retailing skills in the United States before transferring them to
foreign locations. However, for more mature multinationals enterprises that have
already established a network of subsidiary operations in foreign markets, the
development of valuable skills can just as well occur in foreign subsidiaries. –
McDonalds find that its foreign franchisees are a source of valuable new ideas.
Cost Pressures and Pressures for Local Responsiveness

Companies that compete in the global marketplace typically face two types of
competitive pressures: pressure for cost reductions and pressures to be locally
responsive.

Pressures for Cost Reductions

In competitive global markets, international business often face pressures for cost
reductions. Responding to pressures for cost reduction requires a firm to try to lower
the costs of value creation. A manufacturer, for example, might mass-producing a
standardized product at the optimal location in the world, wherever that might be, to
realize economies of scale and locations economies. Alternatively, they might
outsource certain functions to low cost foreign suppliers in an attempt to reduce costs.
– Banks outsourcing telephone service etc. Cost-reduction pressures can be
particularly intense in industries producing commodity-type products where
meaningful differentiation on nonprice factors is difficult and price is the main
competitive weapon. This tends to be the case for products that serve Universal needs
– needs arising from the similar, if not identical tastes and preferences of consumers
in different nations.

Pressures for Local Responsiveness

Pressures for local responsiveness arise from differences in consumer tastes and
preferences, infrastructure and traditional practices, distribution channels and host
government demands. Responding to pressures to be locally responsive requires that a
company differentiate its products and marketing strategy from country to country to
accommodate these factors, all of which tends to raise a company’s cost structure.

Differences in consumer tastes – when customer tastes and preferences differ


significantly between countries for historic or cultural reasons – in such cases a
multinational company’ products and marketing message have to customized to
appeal to the tastes and preferences of local customers hence putting pressure of
production and marketing responsibilities to a company’s overseas subsidiaries. –
Example is the differences of car industry around the globe – for example somewhere
Trucks are purchased as family cars where as elsewhere it may strictly be firms
vehicle.

Differences in Infrastructure and Traditional Practices – arise from differences in


infrastructure or traditional practices among countries, creating a need to customize
product accordingly. Fulfilling this need may require the delegation of manufacturing
and production functions to foreign subsidiaries. – Example: Left-or-right hand wheel
varies from country to country

Differences in Distribution Channels – a company’s marketing strategies may have


to be responsive to differences in distribution channels among countries, which may
necessitate the delegation of marketing functions to national subsidiaries.
Differences in Host Government Demands – Economic and political demands
imposed by host country governments may require local responsiveness. For example,
pharmaceutical companies are subject to local clinical testing, registration procedures,
and pricing restrictions all of which make it necessary that the manufacturing and
marketing of a drug should meet local requirements.

Choosing a Global Strategy

Pressures for local responsiveness imply that it may not be possible for a firm to
realize the full benefits from economies of scale and location economies. It may not
be possible to serve the global marketplace from a single low cost location, producing
a globally standardized product, and marketing it worldwide to achieve economies of
scale – in practice, the need to customize the product offering to local conditions may
work against the implementation of such strategy. For example, automobile firms
have found that Japanese, American, and European consumers demand different kinds
of cars, and this necessitates producing products that are customized for local
markets. – hence Automobile industries use top-to-bottom design and production
facilities in each of these regions so that they can better serve local demands

Global Standardization Strategy

Companies that pursue a global standardization strategy focus on increasing


profitability by reaping the cost reductions that come from economies of scale and
location economies – strategy is to pursue a low-cost strategy on a global scale.

 Production, marketing and R&D are concentrated in a few favorable locations

 Companies pursuing a global standardization strategy try not to customize


their product offering and marketing strategy to local conditions because
customization, which involves shorter production runs and the duplication of
functions, can raise costs - hence some companies choose to market
standardized product worldwide so that they can reap the maximum benefits
from economies of scale.

 This Strategy makes more sense when there are strong pressures for cost
reductions and demand for local responsiveness is minimal – increasingly
these conditions prevail in many industrial good industries, whose products
often serve universal needs.

Localization Strategy

Localization strategy is a strategy that focuses on increasing profitability by


customizing the company’s goods or services so that they provide a good
match to tastes and preferences in different national markets.

 Most appropriate strategy when there are substantial differences across nations
with regard to consumer tastes and preferences, and where cost pressures are
not too intense
 By customizing product to local demand the company increases the value of
that product to local market

 However its more costly to customize as mass production is not possible

 MTV is the best example - each country own channel.

Transnational Strategy

Transnational Strategy – a strategy in which firms try to simultaneously achieve low


costs, differentiate the product offering across geographic markets, and foster a flow
of skills among different subsidiaries the company’s global network of operations.

 Not an easy strategy to pursue since it places conflicting demands on the


company

 Differentiating the product to respond to local demand in different geographic


markers raises costs, which runs counter to the goal of reducing costs. – Ford
found it difficult to implement

International Strategy

Sometimes its possible to identify multinational companies that find themselves that
find themselves in the fortunate position of being confronted with low cost pressures
and low pressures for local responsiveness. Typically these are enterprises that are
selling a product that serves universal needs, but who do not face significant
competitors, and thus are not confronted with pressured to reduce their cost structure.
– Xerox

Companies pursuing an international strategy tend to centralize product


development functions such as R&D at home. However, they also tend to establish
manufacturing and marketing functions in each major country or geographic region in
which they do business – although they may undertake some local customizations of
product offering and marketing strategy, this tends to be rather limited in scope. =
head office usually retains control of marketing an product strategy.

Chancing in Strategy Over Time

The problem with international strategy is that over time, competitors inevitably
emerge, and if managers do not take proactive steps to reduce their cost structure,
their company may be rapidly outflanked by efficient global competitors. Exactly
what happened to Xerox ad other companies invented ways of going around their
patents hence. Therefore strategies may change from time to time.
Choices of Entry Mode

Another key strategic issue confronting managers in a multinational enterprise is


deciding upon the best strategy for entering a market. 5 main choice:

Exporting

Most manufacturing companies begin their global expansion as exporters and only
later switch to one of the other modes for serving a foreign market. Exporting has to
distinct advantages:
 It avoids the costs of establishing manufacturing operations in the host
country, which are often substantial

 and it may be consistent with scale economies and location economies. By


manufacturing the product in a centralized location and then exporting it to
other national markets, the company may be able to realize substantial scale
economies from its global sales volume

There are also a number of drawbacks to exporting:

 First, exporting from the company’s home base may not be appropriate if there
are lower-cost locations for manufacturing the product abroad. Thus,
particularly in the case of a company pursuing a global standardization or
transnational strategy, it may pay to manufacture in a location where
conditions are most favorable from a value creation perspective and then
export from that location to the rest of the globe – argument against exporting
from the company’s home country

 Another drawback is that high transport costs can make exporting


uneconomical, particularly in the case of bulk products – way of getting
around it to manufacture bulk products on a regional basis, realizing some
economies from large-scale production while limiting transport costs.

 Tariff barriers – can make exporting uneconomical and a government threat to


impose tariff barriers can make the strategy very risk

 Finally, exporting through agents – delegate marketing activities in each


country in which it does business to a local agent however no guarantee that
that agent will act in the best interest of the company.

Licensing

International Licensing is an arrangement whereby a foreign licensee buys the rights


to produce a company’s product in the licensee’s country for a negotiated fee. –the
license then puts up most of the capital necessary to get the overseas operation going.

Advantage of licensing is that the company does not have to bear the development
costs and risks associated with opening up a foreign market. Licensing therefore can
be a very attractive option companies that lack the capital to develop operations
overseas. It can also be an attractive option for companies that are unwilling to
commit substantial financial resources to an unfamiliar or politically volatile foreign
market where political risks are particular high

Licensing has three serious drawbacks:

 1. It does not give a company the tight control over manufacturing, marketing,
and strategic functions in foreign countries that it need to have in order to
realize scale economies and location economies – as companies pursuing both
global standardization and transnational strategies try to do. Typically, each
licensee sets up its own manufacturing operations – hence company stands
little chance of realizing scale economies by manufacturing its product in a
centralized location.

 Second, competing in a global marketplace may make it necessary for a


company to coordinate strategic moves across countries so that the profits
earned in one country can be used to support competitive attacks in another.
Licensing, by its very nature limits a company’s ability to coordinate strategy
in this way. A license is unlikely to let a multinational company take its profits
and use them to support an entirely different licensee operating in another
country

 A third problem with licensing is the risk associated with licensing


technological know-how to foreign companies. – by licensing its technology a
company can quickly lose control over it

Franchising

Franchising – a specialized from of licensing in which the franchiser sell the


franchisee intangible property and insists that the franchisee agree to abide by strict
rules about how it does business. – Franchiser will also often assist the franchisee to
run the business on an ongoing basis

Advantages: franchiser does not have to bear the development costs and risks of
opening up a foreign market on its own, for the franchisee typically assumes those
costs and risks. Thus, using a franchising strategy, a service company can build up a
global presence quickly and at a low cost.

Disadvantages: lack of quality control.

Joint Ventures

Joint Venture – a separate corporate entity I which two or more companies have an
ownership stake.

Advantages:

 A company may feel that it can benefit from a local partners knowledge of a
host country’s competitive conditions, culture, language, political systems,
and business systems.
 Second, when the development costs and risks of opening up a foreign market
are high, a company might gain by sharing these costs and risks with local
partner.

 Third, in some countries political considerations make joint ventures the only
feasible entry mode.

Drawbacks:
 A Company that enters into a joint venture risks losing control over its
technology to its revenue partner.

 Joint venture does not give a company the tight control over it subsidiaries that
it might need in order to realize scale economies or location economies.

Wholly Owned Subsidiaries

Wholly Owned Subsidiary – a subsidiary where the parent company owns 100% of
the subsidiary stock

3 advantages:

 When company’s competitive advantage is based on its control of a


technological competency, a wholly owned subsidiary will normally be
preferred entry mode, since it reduces the company’s risk of losing this control
– high tech companies prefer this

 Gives a company the kind of tight control over operations in different in


different countries that it needs if it is going to engage in global strategic
coordination – taking profits from one country to support competitive attacks
in another

 May also be the best choice if a company wants to realize location economies
and the scale economies that flow from producing a standardized output from
a single or limited number if manufacturing plants. = Different subsidiaries
producing different parts of eventual product.

Disadvantage:

 Most costly method of serving a foreign market. Parent company must bear all
the costs and risks of setting up overseas operations. Risk of learning to do
business in a new culture diminish if the company acquires an established host
country enterprise. – however problems with different corporate cultures arise.
STRATEGIC MANAGEMENT

Chapter 7

Long-Run Profitability Through Corporate-Level Strategy

Concentration On a Single Industry

For many companies this appropriate choice-level strategy entails


concentration on a single industry, whereby a company focuses its resources
and capabilities on competing successfully within the confines of a particular
product market – for e.g. McDonalds.

Advantage:

 Doing so enables a company to focus all its managerial, financial,


technological, and functional resources and capabilities on developing
strategies to strengthen its competitive position in just one business – this
strategy is important in fast-growing industries that make strong
demands on company’s resources and capabilities but also offer the
prospect of substantial long-term profits if a company can sustain its
competitive advantage.

Disadvantage:

 Concentrating on just one market or industry can result in disadvantages


emerging over time. – certain amount of vertical integration may be
necessary to strengthen a company’s competitive advantage within its
core industry.

 Moreover, companies that concentrate on just one industry may miss out
on opportunities to create more value and increase their profitability by
using their resources and capabilities to make and sell products in other
markets or industries

Horizontal Integration

Horizontal Integration – acquiring or merging with industry competitors to


achieve the competitive advantages that come with large size. Acquisition
occurs when one company uses its capital resources to purchase another
company, and a merger is an agreement between two companies to pool their
resources in a combined operation. – For e.g. Rupert Murdock buying different
newspapers .
Benefits and Costs of Horizontal Integration

Lower Operating Costs

 HI lowers a companies operating costs when it results increasing


economies of scale – for example buying a competitor and closing his
manufacturing plan and use it for own production

 Achieving economies of scale is very important in industries that have


high fixed costs, because large-scale production allows a company to
spread its fixed costs over a large volume, which drives down average
operating costs.

Increased Product Differentiation

 Horizontal integration may also boost profitability when it increases


product differentiation, for example, by allowing a company to combine
the product lines of merged companies in order to offer customers a
wider range of products that can be bundled together. Product bundling
is the strategy of offering customers the opportunity to buy a complete
range of products at a single combined price

 From this value is added as usually price discount is obtained and they
get used to dealing with just one company – hence competitive advantage
may be achieved

 Example – Microsoft Office

Reduced Industry Rivalry

 Rivalry through horizontal integration can be reduced in 2 ways: 1 st –


acquiring or merging with a competitor helps to eliminate excess capacity
in an industry

 2nd Less competitors = easier to agree on prices agreement hence no price


wars

Increased Bargaining Power

 Final reason to be horizontal is to achieve more bargaining power over


suppliers or buyers, which strengthens its competitive position and
increases its profitability at their expense.

 Company becomes a much larger buyer of a supplier’s product; it can use


this buying power as leverage to bargain down the price it pays for inputs,
and this also lowers its costs.
Disadvantages

◦ Problems with merging cultures, managers and operations – high


management turnover possible, tendency to overestimate benefits
of the merger etc.

◦ Problems with the Federal Trade Commission if a company grows


too large – against monopoly’s’!

Outsourcing Functional Activities

 To improve its competitive position in an industry company can


outsource one or more of its own value creation functions and contract
with another company to perform that activity on its behalf

 Virtual corporation – has been coined to describe companies that


outsource most of their functional activities and focus on one or a few
core value chain

Advantages and Disadvantages of Outsourcing

Advantages:

 Outsourcing a particular noncore activity to a specialist company that is


more efficient at performing that activity than the company itself lowers a
company’s operating costs.

 A specialist often has a distinctive competency in a particular functional


activity, so the specialist can help the company better differentiate its
products.

 Third advantage is that it enables a company to concentrate scarce human


, financial, and physical resources on further strengthening its core
competencies.

Disadvantages:

 Company loses both the ability to learn from that activity and the
opportunity to transform that activity into one of its distinctive
competencies

 Outsourcing may go to far, hence potential value creation can be


outsourced and loss of competitive advantage.
 Company may become too dependent on the subcontractor, which may
start to demand prices etc.

Vertical Integration

Vertical Integration – a strategy in which a company expands its operations


either backward into industries that produce inputs for its core products
(backward vertical integration) or forward into industries that use, distribute or
sell its products (forward vertical integration). To enter a new industry,
company may establish its own operations and create the set of value chain
functions it needs to compete that is already in the industry. For e.g. Apple and
its apple stores

Advantages:

◦ Enables company to build barriers to new competition – limits


competition in company’s industry.

◦ Facilitates investments in specialized assets – a value creation tool


that is designed to perform a specific set of activities and whose
value creation potential is significantly lower in its next-best use.
Companies invest in specialized assets because these assets allow
them to lower the costs of value creation and differentiate their
products from those of competitors – permitting premium pricing

◦ Protects product quality – enables a company to become a


differentiated player in its core business – McDonalds in Russia
setting up its own farms to protect meat quality.

◦ Results in improved scheduling

Disadvantages:

◦ May actually increase cost of inputs when a company has to


purchase high-cost inputs from company owned suppliers despite
the existence of low-cost external sources of supply

◦ Suppliers have less incentive to be efficient – managers of supply


divisions may be tempted to pass on any cost increases to other
company divisions in the form of higher prices for components,
rather than looking for ways to lower costs.

◦ Ties a company into old, obsolescent, and high cost technology –


too much investments may be needed

Thus, on the one hand, vertical integration may create value and increase
profitability when it lowers operating costs or increases differentiation. On the
other hand, it can reduce profitability if a lack of cost-cutting incentive on the
part of company owned suppliers increases operating costs, or if he inability to
change its technology quickly results in lower quality and reduced
differentiation.

In general, company should pursue vertical integration only if the extra value
created by entering a new industry in the value chain exceeds the extra costs
involved in managing its new operations when it decides to perform additional
upstream or downstream value creation activities. Not all vertical integration
opportunities have the same potential the same potential for value creation.

Entering new Industries Through Diversification

Diversification – the process of entering into one or more industries that are
distinct or different from a company’s core or original industry to find ways to
use the company’s distinctive competencies to increase the value to customers of
the products it offers in those industries. A diversified company is the one that
operates in two or more different or distinct industries to find ways to increase
its long-run profitability.

 Diversification can help a company create value in 3 main ways:

◦ Permitting superior internal governance – refer to the manner in


which the top executive of a company manage its business units,
divisions, and functions. In a diversified company, effective or
superior governance resolves around how well top managers can
develop strategies that improve the competitive positioning of its
business units in the industries where they compete.

◦ Transferring competencies among businesses – a second way for a


company to create value from diversification is to transfer its
existing distinctive competencies in one or more value creation
functions to other industries. Top managers seek out companies in
new industries where they believe they can apply these
competencies to create value and increase profitability

◦ Realizing economies of scope – phrase “two can live cheaper than


one” expresses the idea behind economies of scope. When two or
more business units or capabilities such as manufacturing
facilities, distribution channels, advertising campaigns, and R&D
costs, total operating costs fall because of economies of scope.

Restructuring

Restructuring- implementing strategies for reducing the scope of the company


by removing exiting business areas

Why restructure?
◦ Because the stock of highly diversified companies is often assigned
a lower valuation relative to earnings than stocks of less
diversified enterprises

◦ In an attempt to boost returns to shareholders

Restructuring can be beneficial due to diminished advantages of vertical


integration or diversification

Restructuring can be a reaction to:

◦ Managers pursuing too much diversification


◦ Diversification for the wrong reasons
◦ Failed Acquisition

Exit Strategies

Divestment – the sale a business unit to the highest bidder. Three types of
buyers are independent investors, other companies, and the management of the
unit to be divested. Selling off a business makes goo sense when the unit to be
sold is profitable and when the stock market has an appetite for net stock is sues.
However, spinoffs do not work if the unit to be spun off is unprofitable and
unattractive to independent investors or if the stock market is slumping and
unresponsive to new issues.

Harvest Strategy – involves halting investment in a unit in order to maximize


short-to-medium term cash flow from that unit. Although this strategy seems
fine in theory, it is often a poor one to apply in practice. Once it becomes
apparent that the unit is pursuing a harvest strategy, the morale of the units’
employees, as well as the confidence of the units’ customers and suppliers in its
continuing operation, can sink very quickly. If this occurs, as it often does, the
rapid decline in the unit’s revenues can make the strategy untenable

Liquidation Strategy – involves shutting down the operations of a business unit.


A pure liquidation strategy is the least attractive of all to pursue, because it
requires that the company write offs its investment in a business unit, often at
considerable cost.
PART FOUR – STRATEGY IMPLEMENTATION
Chapter 8: Strategic Change (Managing Change)

Firstly this chapter examines the nature of strategic change and the obstacles that may
hinder managers’ attempts to change the company’s strategy and structure, and ultimately,
the steps that managers can take to overcome these obstacles (8.1). Secondly, it will provide
an overview of an important technique used to identify a company’s desired future state:
analyzing the company as a portfolio of core competencies, as opposed to a simple portfolio
of businesses (8.2). Finally an analysis of the different methods that managers can use to
enter new business or industries, will be provided (8.3, 8.4 & 8.5).

8.1 Strategic Change

Strategic change is the movement of a company away from its present state toward some
desired future state to increase its competitive advantage and profitability. More often than
not, the push for strategic change comes from the changing external competitive
environment, rather than internal operating environment.

One way of changing the company to enable it to operate more effectively is by


reengineering, a process in which managers focus not on the company’s functional
activities, but on the business processes underlying the business value creation process. A
business process is any activity (such as order processing, inventory control, or product
design) that is vital to delivering products to customers quickly, or that promotes high
quality or low costs. Business processes are not the responsibility of any one function in the
firm’s value chain, but they cut across functions.

Another way is to pursue restructuring, the process through which managers simplify
organizational structure by eliminating divisions, departments, or levels in the hierarchy, and
downsize by terminating employees; thereby lowering operating costs. Restructuring may
also involve outsourcing the company’s non‐core functional activities.

In order to understand issues involved in implementing strategic change, we focus on the


series of distinct steps that strategic managers must follow if the change process is to
succeed (shown below):

The first step in the change process is for strategic managers to recognize the need for
change. Sometimes problems can be obvious, but more often they are unnoticeable,
because problems develop silently/gradually, and organizational performance may slip for
quite some time, before the decline becomes obvious. Once, managers realize that there is
something wrong, they conduct external and internal (i.e. SWOT) analysis discussed in
chapters 3 & 4. Ultimately, when the source of the problem has been identified, strategic
managers must determine the desired future state of the company, i.e. how it should
change its strategy to achieve the newly set goals.

Given that the decisions to reengineering and restructuring requires the establishment of
new set of roles, rewards and authority relationships among managers/employees in
different
1

functions/division, there will be resistance to change due to inherent organizational inertia.


Therefore strategic managers must identify potential obstacles to change as they design
and implement new strategies. These obstacles can be found in all levels of the organization:
corporate, divisional, functional and individual.

Generally a company can take one of two main approaches to implementing and managing
change: top‐down change or bottom‐up change. With top‐down change, a strong CEO or
top management team (TMT) analyses what strategies need to be pursued, recommends a
course of action, and then moves quickly to implementing the changes. The emphasis is on
speed of response to problems as they emerge. Bottom‐up change is much more gradual in
that TMT consults with managers/employees at all levels in the organization. Then over time
as the strategic change plan develops, it is gradually implemented. The emphasis is on
participation and keeping people informed. The advantage of bottom‐up change is that it
removes some of the obstacles to change by including them in the strategic planning
process. It could also reveal more hidden potential problems, because the whole
organization is being consulted. However, when speedier response is required, top‐down
change should be utilized.

The last step in the change process is to evaluate the effects of strategic change on
organizational performance. The company must compare the way it operates after
implementing the changes vis‐à‐ vis the way it operated before, which could be done using
proxies, such as changes in stock market price, market share, and higher revenues. The
company’s performance can also be benchmarked against the market leaders, to see if
further improvement is needed.

8.2 Analyzing a Company as a Portfolio of Core Competencies

According to Hamel and Prahalad, the company must be analyzed as a portfolio of core
competencies, as opposed to a portfolio of actual businesses (e.g. the BCG Matrix). After all,
the goal of most companies is to achieve long‐run growth and profitability, the key to which
are the companies’ core competencies. Hence the corporate development should be
oriented towards maintaining and building new competencies, as well as leveraging existing
competencies by applying them to new business opportunities.

(1) Fill in the blanks refers to the opportunity to improve the company’s competitive
position in its existing markets by leveraging existing core competencies. For example,
Canon was able to improve the position of its camera business by leveraging
microelectronics skills from its copier business to support the development of cameras
with electronic features, such as autofocus capabilities.

(2) Premier plus 10 raises an important question for the company: “What new core
competencies must be built today to ensure that the company remains a premier provider
of its existing products in 10 years’ time?”. For example, Canon decided that in order to
maintain a competitive edge in its copier business, it was going to have to build a new
competency in digital imaging. This new competency subsequently helped Cannon to extend
its product range to include laser copiers, colour copiers, and digital cameras.

(3) White spaces raises the question: “How best to fill the white space by creatively
redeploying or recombining our current core competencies. For example, Canon has been
able to recombine its established core competencies in precision mechanics, fine optics,
microelectronics and digital imaging, to enter the market for computer printers and
scanners.

(4) Mega‐opportunities do not overlap with the company’s current market position or with
its current endowment of core competencies. Nonetheless, the company may choose to
pursue such opportunities, because they are particularly attractive or relevant to the
company’s existing business opportunities. For example, in 1979 Monsanto, manufacturer of
chemicals, saw enormous opportunities in the emerging field of biotechnology. It made an
investment of around $1.0bil, believing that it could produce genetically engineered crops,
which would produce their own organic pesticides. Ultimately, the investment earned the
company hundreds of billions of dollars in profit.

The biggest advantage of Hamel & Prahald’s framework is that it recognizes the
interdependencies among the company’s businesses and focuses on opportunities to create
value by building and leveraging competencies, unlike traditional portfolio tools that treat
businesses as independent.

8.3 Implementing Strategy through Internal New Ventures

Internal new ventures involve creating the value chain functions necessary to start a new
business scratch. Internal new venturing is typically used to execute corporate‐level
strategy, when the company possesses a set of valuable competencies (resources &
capabilities) in its existing businesses that can be leveraged or recombined to enter new
business areas. In general, science‐based companies tend to favour internal new venturing
as an entry strategy.

Despite the popularity of internal new venturing, there is a high risk of failure (33% ‐ 60%).
Three main reasons are often put forward to explain this relatively high failure rate: (1)
market entry on too small a scale; (2) poor commercialization of the new‐venture product;
(3) poor corporate management of the new‐venture division.

(1) Vis‐à‐vis the less risky small‐scale entry, large‐scale entry can more rapidly realize scale
economies, build brand loyalty and good distribution channels in the new industry, all of
which would increase the probability of the new venture’s success. In contrast, small‐scale
investments are more prone to failure, because they lack the capacity to achieve these
advantages, which are especially important when entering an established/mature industry.
However, because of the high costs and risks associated with large‐scale entry, many
companies make the mistake of choosing the small‐scale entry strategy, which often results
in failure to build the market share necessary for long‐term success (see diagram below).

(2) To be commercially successful, science‐based innovations must be developed with


market requirements in mind. Many internal new ventures fail, because the company gets
fixated on the innovative and technological myopia, which leads it the improper analysis of
market opportunities. Thus a new venture can fail, because of a lack of commercialization,
or because it is marketing a technology for which there is no demand (e.g. Motorola’s failure
with satellite phone Iridium).

(3) Poor corporate management examples include: the company stretching its resources
and management attention too thin over too many new ventures; failure to establish the
strategic context within which new‐venture projects should be developed (e.g. the
company
cannot simply allow R&D team to do research on whatever they find interesting); failure to
anticipate the time and costs involved in the new‐venture process (e.g. killing the project too
early).

To avoid pitfalls of internal new ventures, the company should adopt a structured approach
to managing internal new venturing. Firstly, the entry into new industry should be on a large
scale to increase the probability of long‐run profitability. Secondly, to improve the
commercialization of the new venture idea, the company should foster close links between
R&D and market, and R&D and manufacturing. Finally, the company must employ a
structured approach to project selection (with the greatest chance of commercial success)
and set clear targets for that new venture (preferably market share and not profit targets).

8.4 Implementing Strategy through Acquisitions

Acquisitions involve one company purchasing another company. The company may use
acquisitions in two ways: to strengthen its competitive position in an existing business by
purchasing a competitor (i.e. horizontal integration), and to enter a new business/industry
(i.e. vertical integration & diversification).

Companies have a preference for acquisitions as an entry mode, when they feel the need to
move in fast. Acquisitions are also somewhat less risky than internal new ventures, because
they involve less commercial uncertainty. Acquisitions also allow the company to
circumvent the barriers to entry.

Nevertheless acquisitions often fail. There are appears to be 4 major reasons: (1) difficulties
with integrating divergent corporate cultures; (2) managers overestimate the potential
economic benefits from an acquisition; (3) acquisitions tend to be very expensive; (4)
companies often do not adequately screen their acquisition targets.

(1) Post‐acquisition integration can be very challenging, because the companies might have
very different corporate cultures, which makes it difficult to adopt a common financial
control system and to join the companies’ operations. What’s more high management
turnover stymies the benefits of integration.

(2) Companies often overestimate the strategic advantages that can be derived from the
acquisition, because top managers tend to overestimate their abilities to create value from
an acquisition (e.g. Time Warner acquisition of AOL in 2001, and spinning it off in 2009).

(3) Acquisitions also tend be to very expensive, because the stock price of the target
company usually get bid up in the acquisition process. Oftentimes, the company has to pay a
premium of 50% ‐ 100%, meaning that the company has to improve the performance of the
acquired unit by just as much, if it is to reap a positive return on its investment! This can be
particularly hard.

(4) Sometimes the management does an inadequate job of pre‐acquisition screening, and
decide to acquire other firms without thoroughly analyzing the potential benefits and costs.
After the acquisition has been completed, the management might discover that instead of
buying a well‐run business, they have purchased a troubled organization.
To avoid pitfalls, and make successful acquisitions, companies need to take a structured
approach with three main components:

(1) Prudent target identification and careful pre‐acquisition screening; (2) Competitive
bidding strategy with the right timing

(3) Integration centred on the source of the potential strategic advantages of the acquisition
(e.g. share functional activities and eliminate duplications of facilities/functions).

8.5 Implementing Strategy through Strategic Alliances

Strategic alliances are cooperative agreements between two or more companies to work
together and share resources to achieve a common business objective. A joint venture is a
formal type of strategic alliance in which two companies jointly create a new, separate
company to enter a new business area.

Strategic alliances can take the form of short‐term informal agreements (e.g. to share know‐
how), or the long‐term contractual agreement, such as long‐term outsourcing or JTs.
Strategic alliances may be the preferred entry strategy when (1) the risks and costs
associated with setting a new venture/business unit are more than the company is willing to
assume on its own, and (2) the company can increase the probability of success of the new
business by teaming up with another company that has resources and capabilities
complementing its own.

Strategic alliances can: (1) facilitate entry into markets (e.g. Motorola forming alliance with
Toshiba to overcome the Japanese trade barriers, i.e. barrier to entry); (2) enable partners
to share the fixed costs and risk associated with the development of new products and
processes; (3) facilitate the transfer of complementary skills and assets that neither
company could easily develop on its own (e.g. Microsoft & Toshiba alliance to develop
microprocessors = software engineering + microprocessors expertise).

The drawbacks of formal strategic alliances (particularly JTs) include: (1) the risk that the
company may give away technological know‐how and its market access to the alliance
partner, without getting much in return; (2) sharing profits, if the alliance proves to be very
successful; (3) sharing control meaning harder global strategic coordination of retained
profits (see chapter 6).

The success of a strategic alliance seems to be a function of three main factors: (1) partner
selection; (2) alliance structure; (3) the manner in which the alliance is managed.

(1) A good partner has three principal characteristics. Firstly, the partner must have
resources and capabilities that the company lacks and values. Secondly, the partner shares
the firm’s long‐term vision for the purpose of the alliance. Thirdly, the partner is unlikely to
try to exploit the alliance opportunistically (e.g. steal technological know‐how); for example,
IBM has to keep its good reputation, in order to further attract important alliance partners
in the future.

(2) Once the partner has been selected, the alliance should be structured so that the
company’s risk of giving too much away to partner is reduced to an acceptable level. The
figure below depicts the four safeguards against opportunism of cheating by alliance
partners.

Establishing contractual safeguards can include agreements that prohibits the partner to
enter the company’s home market and utilize the shared technology and managerial know‐
how to compete against it.

Seeking credible commitments can include factors, such as pushing the bigger partner to buy
equity stake in the smaller partner, or making the bigger partner take the minority
ownership position in the JT.

(3) Ultimately managing the alliance effectively requires sensitivity to cultural


differences, building trust between partners and learning from each other. The alliance
also needs relational capital, i.e. building interpersonal relationships between the firms’
managers.
lOMoARcPSD|6004607

Chapter 9: Implementing Strategy through


Organizational Design
Strategy implementation refers to the way a company creates the
organizational arrangements/design that enable it to pursue its
strategy most effectively. Organizational design is the process through
which managers select the combination of organizational structure
and control systems that allows it to effectively create and sustain a
competitive advantage. Therefore the primary role of organizational
structure and control is to motivate employees to achieve superior
efficiency, quality, innovation, and customer responsiveness (the 4
building blocks of competitive advantage). Ultimately, organizational
structure and control shape the way people behave and act in the
organizational setting.

9.1 The Role of Organizational Structure


Once the company’s strategy has been formulated, management must
design the organizational structure. The role of organization structure
is to provide the vehicle through which managers can coordinate the
activities of the company’s various functions, division and business
units to take advantage of their skills and competencies (to develop
and sustain competitive advantage).

The basic building blocks of organizational structure are differentiation


and integration.

Differentiation is the way in which the company allocates people and


resources to organizational tasks, in order to create value. Generally,
the greater the number of different functions/division in the
organization and the more skilled/specialized they are, the higher is
the level of differentiation (e.g. GE). Firstly, strategic managers must
choose how to distribute decision‐making authority in the
organization to control value creation activities best, i.e. vertical
differentiation choices (e.g. how much authority to delegate to
managers at the divisional and functional level). Secondly, corporate

Downloaded by Imad Quadri (imad3889@yahoo.co.in)


managers must choose how to group people and tasks into
functions/divisions to increase their value‐ creation ability, i.e.
horizontal differentiation choices (e.g. should there be separate sales
and marketing departments?).

Integration is the means by which the company seeks to coordinate


people and functions to accomplish organizational tasks. When
separate/distinct value creation functions/division exist, they tend to
pursue their own goals and objectives; therefore the company has to
create an organizational structure that encourages different
functions/divisions to coordinate their activities, by use of
integrating mechanisms and control systems.

Ultimately, differentiation consists of the way the company divides


itself into parts (functions/divisions), whereas integration consists of
the way those parts are then combined for enhanced value creation.
Together, the two processes determine how an organizational
structure will operate, and how successfully strategic managers will be
able to create value through their chosen strategies.

9.2 Vertical Differentiation


The aim of vertical differentiation is to specify the reporting
relationships that link people, tasks, and functions at all levels within
the company. This entails choosing the appropriate number of
hierarchical levels and the correct span of control for implementing
the company’s strategy most effectively.

The organizational hierarchy establishes the authority structure from


the top to the bottom of the organization. The span of control is
defined as the number of subordinates a manager directly manages.
The basic choice is whether to aim for a flat structure, with a few
hierarchical levels and thus relatively wide span of control; or a tall
structure, with many levels and thus a relatively narrow

1
span of control. The average number of hierarchical levels for a
company employing 3,000 people is 7 – 8.

As the company grows, the number of levels in its hierarchy of


authority increases, to allow it to monitor and coordinate employee
activities more efficiently. Though managers should try to keep the
organization as flat as possible and follow the principle of the
minimum chain of command, which states that an organization
should choose a hierarchy with minimum number of levels of
authority necessary to achieve its strategy (demonstrated below with
the levelling of the curve).

Managers try to keep the hierarchy as flat as possible, because when


companies become too tall, several problems arise that make strategy
more difficult to implement.

Coordination problems. Having too many hierarchical levels impedes


communication and coordination between employees and functions,
and also raises costs, as communication between the top and the
bottom of the hierarchy takes much longer.

Information distortion. The greater the number of hierarchical levels,


the more scope employees and managers have, to distort facts; as a
consequence, the costs of managing the hierarchy increases. The
managers at different levels (functional, divisional & corporate) may
misinterpret information, either accidentally or on purpose, to suit
their own interests.

Motivational problems. As the number of levels in the hierarchy


increases, the amount of authority and autonomy possessed by
managers at each hierarchical levels diminishes, while the
promotional steps up the corporate ladder increases. Ultimately, the
managers tend to refuse to take the risks that are often necessary
when new strategies are pursued. This further increases the costs of
coordination, because more corporate/strategic managerial time must
be spent coordinating tasks.
Too many middle managers = high costs. Having many hierarchical
levels implies having many middle managers, and employing managers
is expensive.

Decentralization can help overcome some of these problems.


Authority is centralized when managers at the upper levels of the
organizational hierarchy retain the authority to make the most
important decisions. When authority is decentralized, it is delegated
to divisions, functions, managers and workers at lower levels in the
organization. Decentralization has 3 main advantages:

(1) By delegating day‐to‐day operational decision‐making authority to


middle managers, strategic managers reduce information overload,
which enables them to spend more time on strategic decision making,
which = more effective decisions.

(2) When middle managers become responsible for adapting the


organizational structure to local conditions, their motivation and
accountability increase, which also promotes organizational flexibility
necessary for anticipation of changes in the external competitive
environment.

(3) When lower‐level employees are given more authority, fewer


managers are needed = lower costs.

Notwithstanding, centralization has its advantages too. Firstly, it


makes coordination of the organizational activities needed to pursue
the company’s strategy, less chaotic than when the strategic planning
is decentralized to include everybody’s opinion. Secondly,
centralization also means that decisions fit the broad organization
objectives without any division pursuing its own objectives.

9.3 Horizontal Differentiation


As the company grows, even decentralization becomes less effective
(e.g. Walmart with 2.2 million employees). The company will firstly
have to resort to horizontal differentiation, and secondly to
integration mechanisms and control systems (discussed in the next
parts – 9.4 & 9.5).

The challenge facing the company is choosing the appropriate form


of horizontal differentiation – that is deciding how best to group
organizational tasks and activities to meet the objectives of the
company’s strategies. The different grouping/structures are discussed
below.

(1) Functional structures group people on the basis of their common


expertise or because they use the same resources (e.g. engineers are
grouped into R&D function of the firm’s value chain).

Functional structures have several advantages. Firstly, if people who


perform similar tasks are grouped together, they can learn from each
other, and become more specialized and productive. Secondly, due to
similar expertise, they can monitor each other to make sure that
everybody is performing their tasks effectively and no one is free
riding. Thirdly, it gives corporate managers greater control of
organizational activities, because each function of the firm’s value
chain will have its own manager overseeing the hierarchy within a
firm’s function (i.e. managing the business is much easier, when
different groups specialize in different organizational tasks, and are
managed separately).

However, when the company becomes geographically diverse or starts


producing a wide range of products, several coordination problems
arise that undermine the company’s ability to coordinate its activities
and reduce costs:

Communication problems. As separate functional hierarchies evolve,


functions grow more remote from one another, making it increasingly
difficult to communicate across function to coordinate activities,
because distinct functions develop different orientations toward the
problems (and hence strategies) facing the organization (e.g. R&D vs.
manufacturing perspective on strategy).
Measurement problems. As the number of its products grows, the
company may find it difficult to measure the contribution of one or a
few products to its overall profitability. Consequently, the company
may turn out some unprofitable products without realizing it (e.g.
Dell’s explosive growth in the 1990s made it lose control of its
inventory management systems).

Location problems. If the company makes and sells products in many


different regions, then the centralized system of control provided by
the functional structure no longer suits it, because managers in the
various regions must have the flexibility to respond to the needs of
their customers.

Strategic problems. Sometimes the combined effect of all these


problems is that long‐term strategic considerations are frequently
ignored, because management is preoccupied with solving
communication and coordination problems. As a result the company
may lose direction and fail to take advantage of new opportunities,
while costs escalate.

(2) In the product structure, activities are grouped by product line.


The manufacturing function is broken down into different product
lines based on the similarities and differences among the products.
Seeing as, there might exist three different product grouping, the
degree of horizontal differentiation in the product structure is higher
than that in the functional structure. Moreover, the specialized
support functions (e.g. accounting, sales and R&D) are centralized at
the top of the organization, but each support function is divided in
such a way that personnel tends to specialize in one of the (three)
different product categories to avoid communication problems; thus
there may be three groups of accountants, one for each of three
product categories. Furthermore, the use of the product structure
reduces the problems of control and coordination associated with
the functional structure: it pushes aside barriers among
functions/divisions, because the product line (rather than
each individual function/division), becomes the focus of attention. In
addition, the profit contribution of each product line can be clearly
identified, and hence resources can be allocated more efficiently.
Ultimately, the product structure has one more level in the hierarchy
than the functional structure due to the addition of product line
managers. This increase in vertical differentiation allows managers at
the production level to focus on day‐to‐day operations, which gives
top strategic managers more time to develop the long‐term
strategy for the company’s competitive advantage. Although the
operating costs are higher than in the functional structure, this
expense is warranted by the extra coordination that the structure
provides.
Below is the product structure typical of an imaging firm.

(3) In the product‐team structure, as in the product structure, task


activities are divided along product lines to reduce costs and increase
management’s ability to monitor and control the manufacturing
process. The difference is that specialists are taken from the various
support function and combined into cross‐functional teams to serve
the needs of the product/project. When all functions have direct input
from the beginning, design costs and subsequent manufacturing costs
can be kept low. In addition, the use of cross‐functional teams can
speed innovation and responsiveness to customers, because when
authority is decentralized to the team, decisions can be made more
quickly.

(4) When the company is geographically spread, geographic regions


become the basis for the grouping of organizational activities, i.e.
geographic structure. For example, the company may divide up its
manufacturing operations and establish manufacturing plants in
different regions of the country. This allows it to be responsive to the
needs of its regional customers and reduces transportation costs.
Similar to the product structure, the geographic structure provides
more control than the functional structure, because there are
several
regional hierarchies carrying out the work previously performed by a
single centralized hierarchy (e.g. FedEx & Walmart). Moreover,
because the purchasing function remains centralized, this gives the
company a greater bargaining power over suppliers.

(4) The multidivisional structure has two main advantages over the
functional structure: innovations that let the company grow, yet
overcome problems that stem from loss of control. The main
characteristics of a multidivisional structure are as follows.

Firstly, each distinct product line or business is placed in its own


self‐ contained unit/division, with all the necessary support
functions.
Secondly, the office of corporate headquarters staff is created to
monitor divisional activities and exercise financial control over each
division. Seeing as this constitutes an additional level in the
organizational hierarchy, the multidivisional structure has a higher
level of vertical differentiation than the functional structure. Thirdly,
there is flexibility in that each division is able to adopt the structure
that best suits its needs (e.g. functional structure, product‐team
structure, matrix structure – a structure in which functional managers
work with project managers in temporary teams to develop new
products). Ultimately, in the multidivisional structure, the day‐to‐day
operations of a division are the responsibility of divisional managers,
i.e. divisional management has operating responsibility. Corporate
managers are responsible for overseeing long‐term strategic plans
and providing guidance for interdivisional projects, i.e. they have
strategic responsibility. Such a combination of self‐contained
divisions with the centralized corporate management represents a
higher level of both vertical and horizontal differentiation, which
provides the extra control necessary to coordinate growth and
diversification. Although, this structure has the highest operating
costs, it can help the company organize its value‐creation activities to
achieve even greater performance.

The multidivisional structure offers several advantages, and enable


the company to operate more complex kinds of corporate‐level
strategies.

Enhanced corporate financial control. Seeing as, the profitability of


different divisions is clearly visible and each division is its own profit
centre, financial controls can be applied to each business (e.g. on the
basis of profit criteria) to monitor performance. The corporate
managers can then use this information to identify the divisions that
are worth investing in for long‐term profits. Essentially the corporate
managers act as internal investors.

Enhanced strategic control. The multidivisional structure frees


corporate managers from operating responsibilities, which gives them
more time for contemplating wider long‐term strategies in response
to environmental changes. In addition, the structure provides
corporate managers with the required information for strategic
planning.

Growth. By reducing information overload at the corporate level,


corporate managers can handle a greater number of businesses, i.e.
they can consider more opportunities for further growth and
diversification. Additionally, communication problems are reduced,
because the same set of standardized accounting and financial control
techniques can be used to evaluate all divisions.

Stronger pursuit of internal efficiency. Unlike the functional structure


with its measurement problem, in a multidivisional structure the
individual efficiency of each autonomous division can be directly
observed and measured in terms of profitability. Autonomy makes
divisional managers accountable; therefore they have no excuses for
poor performance.

However, the multidivisional structure has its disadvantages too.

Establishing the appropriate divisional‐corporate authority


relationship. The corporate managers have to decide how much
authority and control to assign to the operating divisions, and how
much authority to retain at the corporate headquarters (i.e. how
much authority to centralize and how much to decentralize). If too
much authority is retained, the managers of the operating divisions
lack the

sufficient autonomy to develop the business‐level strategy that might


best meet the needs of the division. On the other hand, when too
much authority is delegated, manager may start to pursue strategies
that benefit their own divisional objectives, but add little value to the
corporation as a whole.

Distortion of information. If corporate management puts too much


emphasis on divisional ROI, divisional managers may choose to distort
the information they supply to the TMT to paint a rosier picture of
their division (at the expense of long‐term profitability). That is,
divisions may start to pursue strategies that increase short‐run
profitability but reduce future long‐run profitability.

Competition for resources. Given a fixed amount of resources for


allocation, divisions may destructively compete for them, which
prevents synergy gains or economies of scope from emerging.

Transfer pricing. Divisional competition may also lead to battles over


transfer pricing. One of the main challenges that vertical integration
and related diversification imposes is the need to set fair prices, at
which products are transferred between divisions. This will be
problematic, when each supplying division tries to set the highest
price for its outputs to maximize its own profitability.

Focus on short‐term R&D. Again, if extremely high profitability targets


are set by corporate headquarters, the danger arises that divisions will
cut back on R&D to improve the short‐term financial performance of
their divisions at the expense of long‐term profitability.

High operating costs. Seeing as, in the multidivisional structure, each


division has its own support functions, the operating costs are very
high, because of the duplication of functions. Though the duplication
is not a problem, if the gains from having separate support functions
outweigh the costs.

9.4 Integration and Organizational Control


As was pointed out above, choosing the type of differentiation is only
the first step in organizational design. The second decision concerns
the level and type of integration and control necessary to make the
organizational structure work effectively.

The company’s level of integration is the extent to which it seeks to


coordinate its value creation activities and make them
interdependent. The higher the company’s level of differentiation, the
higher the level of integration needed to make organizational
structure work effectively. There is a series of integrating mechanisms
the company can use to increase its level of integration, as its level of
differentiation increases.

Direct contact. The aim behind establishing direct contact among


managers it to set up a context within which managers from different
division/functions can work together. Though more often than not,
managers tend to compete rather than cooperate, which wastes the
top corporate managers’ time, as they working on resolving these
issues.

Interdepartmental liaison roles. Another way of improving


interdepartmental coordination is giving one manager in each
division/function the responsibility for coordinating with the other
division/function. The responsibility for coordination is part of the
manager’s full time job, which helps form a more permanent
relationship between the divisions/functions.

Temporary task forces. When more than two divisions/functions


share a common problem, the solution is to adopt a temporary task
force. One member of each division/function is assigned to the task
force created to solve that specific problem. Essentially, task forces
are ad hoc committees, and members are responsible for reporting
back to their departments on the issues addressed and the solutions
recommended. Task force members also have to perform their normal
duties, while serving on the task force.

Permanent teams. When issues addressed by the task force often


reoccur, it is sensible to form a permanent team for problems
that have a great deal of integration between functions.
Essentially,
permanent teams are the organization’s standing committees (not ad
hoc), and much of the strategic direction of the organization is
formulated in their meetings. In the product‐team differentiation
structure, the cross‐functional teams are essentially the standing
committees (i.e. permanent teams).

Integrating roles. The only function of the integrating role is to prompt


integration among division/departments, which is a full‐time job
requiring an independent expert who, is normally a senior manager
with a great deal of experience in the joint needs of the two
departments.

Note that just as too much differentiation and not enough integration
lead to the failure of implementation, the converse is also true. The
combination of low differentiation and high integration leads to an
over‐controlled, bureaucratized organizations, in which flexibility and
speed of response are reduced by the level of integration. Besides,
differentiation and integration are both very costly; therefore the
management goal is to decide on the optimum amount of integration
(and differentiation), i.e. the simplest structure consistent with
implementing its strategy effectively.

9.5 The Nature of Organizational Control


In practice, integrating mechanisms are only the first means through
which the company seeks to increase its ability to coordinate its
activities. Control systems are the second. Organizational control is
the process by which managers monitor the ongoing activities of an
organization and its members, to evaluate whether activities are being
performed efficiently and effectively, and to take corrective action to
improve performance if necessary. Organizational control does not
just mean reacting to events after they have occurred; it also means
keeping an organization on track, anticipating events that might occur,
and responding swiftly to new opportunities that present themselves.

Strategic control systems are the formal target‐setting, measurement,


and feedback systems that allow strategic managers to evaluate
whether the company is achieving superior efficiency, quality,
innovation and customer responsiveness, and is implementing its
strategy successfully. An effective control system should have three
characteristics: (1) it should be flexible enough to allow managers to
respond quickly to unexpected events; (2) it should provide accurate
information, giving true picture of organizational performance; (3) it
should supply managers with the information in a timely manner,
because making decisions on the basis of outdated information is a
recipe for failure.

As shown below, designing an effective strategic control system


requires 4 steps.

The table below shows the various types of strategic control systems
that manager can use to monitor and coordinate organizational
activities (the unobvious ones will be discussed below).

In output control, strategic managers estimate/forecast appropriate


performance goals for each division, department, and individual
employees, and then measure actual performance vis‐à‐vis these set‐
out goals.

Behaviour control is control through establishment of a


comprehensive system of rules and procedures, to direct the
actions/behaviour of divisions, functions and individuals.

An operating budget is a blueprint that states how managers intend to


use organizational resources to achieve organizational goals most
efficiently. Oftentimes, managers at one level allocate to managers at
a lower level a specific amount of resources for production of
goods/services.

Standardization is the degree to which the company specifies how


decisions are to be made so that employees’ behaviour becomes
predictable (e.g. JIT inventory systems help standardize the flow of
inputs).

One important kind of behavioural control is the organizational


culture, which is the specific collection of values and norms that are
shared by people and groups in the organization, and that controls the
way people interact with each other and with stakeholders outside
the organization. Organizational values are beliefs and ideas about
what kinds of goals, members of the organization should pursue, and
what kinds or standards of behaviour employees should use to
achieve these goals. From organizational values develop
organizational norms, the guidelines or expectations that prescribe
appropriate kinds of behaviour by employees in particular situations,
and control the behaviour of organizational members toward one
another. Ultimately, let’s not forget that organizational culture is the
product of strategic leadership provided by the organization’s founder
and TMT; the organization’s founder is particular important in shaping
the culture (e.g. Walmart or Nucor Steel Corporation).

You might also like